societatea de ȘtiinȚe - ssmrmh.ro · s.s.m.romÂnia - filiala mehedinți 2016 6 revista de...

83
S.S.M.ROMÂNIA - Filiala Mehedinți 2016 1 REVISTA DE MATEMATICĂ MEHEDINȚEANĂ NR. 17 SOCIETATEA DE ȘTIINȚE MATEMATICE DIN ROMÂNIA Filiala Mehedinți REVISTA DE MATEMATICĂ MEHEDINȚEANĂ R.M.M. Nr.17-2016

Upload: others

Post on 03-Sep-2019

7 views

Category:

Documents


1 download

TRANSCRIPT

Page 1: SOCIETATEA DE ȘTIINȚE - ssmrmh.ro · S.S.M.ROMÂNIA - Filiala Mehedinți 2016 6 REVISTA DE MATEMATICĂ MEHEDINȚEANĂ NR. 17 PRINCIPIUL INCLUDERII ȘI EXCLUDERII Daniel Stretcu

S.S.M.ROMÂNIA - Filiala Mehedinți 2016

1 REVISTA DE MATEMATICĂ MEHEDINȚEANĂ NR. 17

SOCIETATEA DE ȘTIINȚE MATEMATICE DIN ROMÂNIA

Filiala Mehedinți

REVISTA DE MATEMATICĂ MEHEDINȚEANĂ

R.M.M.

Nr.17-2016

Page 2: SOCIETATEA DE ȘTIINȚE - ssmrmh.ro · S.S.M.ROMÂNIA - Filiala Mehedinți 2016 6 REVISTA DE MATEMATICĂ MEHEDINȚEANĂ NR. 17 PRINCIPIUL INCLUDERII ȘI EXCLUDERII Daniel Stretcu

S.S.M.ROMÂNIA - Filiala Mehedinți 2016

2 REVISTA DE MATEMATICĂ MEHEDINȚEANĂ NR. 17

SOCIETATEA DE ȘTIINȚE MATEMATICE DIN ROMÂNIA

Filiala Mehedinți

COLECTIVUL DE REDACȚIE REVISTA DE MATEMATICĂ MEHEDINȚEANĂ

1 DANIEL SITARU Redactor șef ; Machetare grafică ; Copertă 2 GHEORGHE CĂINICEANU Redactor principal 3 DAN NĂNUȚI Redactor principal 4 EMILIA RĂDUCAN Redactor ediție electronică RMM 5 MIHAI OCTAVIAN UNGUREANU Redactor coordonator primar 6 DANA PAPONIU Redactor coordonator gimnaziu 7 LEONARD GIUGIUC Redactor coordonator liceu 8 IULIANA GIMOIU Redactor clasa a V-a 9 ELENA RÎMNICEANU Redactor clasa a VI-a

10 DRAGA TĂTUCU MARIANA Redactor clasa a VII-a 11 DANIEL STRETCU Redactor clasa a VIII-a 12 CLAUDIA NĂNUȚI Redactor clasa a IX-a 13 DAN NEDEIANU Redactor clasa a X-a 14 GABRIELA BONDOC Redactor clasa a XI-a 15 OVIDIU TICUȘI Redactor clasa a XII-a

Page 3: SOCIETATEA DE ȘTIINȚE - ssmrmh.ro · S.S.M.ROMÂNIA - Filiala Mehedinți 2016 6 REVISTA DE MATEMATICĂ MEHEDINȚEANĂ NR. 17 PRINCIPIUL INCLUDERII ȘI EXCLUDERII Daniel Stretcu

S.S.M.ROMÂNIA - Filiala Mehedinți 2016

3 REVISTA DE MATEMATICĂ MEHEDINȚEANĂ NR. 17

CUPRINS Suma cifrelor unui număr natural- Daniel Stretcu.....................................................................................4

Asupra unei ecuații diofantice-Dan Nedeianu..........................................................................5

Principiul includerii și excluderii- Daniel Stretcu.........................................................................................6

Punctul și teorema lui Gergonne-Radu-Cătălin Șeitan…..........................................................8

Perpendicularitate și relații generate de aceasta într-un triunghi dreptunghic- Carmen – Victorița Chirfot…………………..………………………………………………………………………………………...….11

An inequality from Gazeta Matematica-March 2016-Daniel Sitaru,Leonard Giugiuc...........14

An inequality from Gazeta Matematica-II-March 2016-Daniel Sitaru,Leonard Giugiuc........16

Magia numerelor complexe -Leonard Giugiuc, Diana Trăilescu.............................................17

A special inequality for triangle -Mihaly Bencze.........................................................................................21

Solutions and generalizations of some problems from Crux Mathematicorum –D.M. Bătinețu Giurgiu, Neculai Stanciu.................................................................................................................22

An Application of Schur’s Inequality- Daniel Sitaru,Leonard Giugiuc............................................25

Some new inequalities –Mihaly Bencze......................................................................................................26 An Angle Inequality in Simple Polygons-Daniel Sitaru……………………………………………...……..30

An Inequality via Young’s Theorem- Daniel Sitaru……………………………………………………...…..31

Equation in Permutation Be on Lookout- Daniel Sitaru……………………………………………….….32

Solved Problems I- Nguyen Viet Hung,Titu Zvonaru,Neculai Stanciu............................................32

Solved Problems II- Nguyen Viet Hung, Abnushav Mishra,Kunihiko Chikaia.......................................................................................................................................................................34 Solved Problems III- D.M. Bătinețu Giurgiu, Neculai Stanciu..............................................................41

Some applications of the convexity and concavity-Marian Dincă..................................................43

Triangle solutions and inequalities of Alexandru Szoros-Van Le.....................................................44

Probleme propuse………………………………………..…………………………………………………………………...50 Rezultate la Olimpiade și Concursuri…………………………………………………………………………………………77

Autorii materialelor din RMM-17………………………………………………………………………………………………83

Page 4: SOCIETATEA DE ȘTIINȚE - ssmrmh.ro · S.S.M.ROMÂNIA - Filiala Mehedinți 2016 6 REVISTA DE MATEMATICĂ MEHEDINȚEANĂ NR. 17 PRINCIPIUL INCLUDERII ȘI EXCLUDERII Daniel Stretcu

S.S.M.ROMÂNIA - Filiala Mehedinți 2016

4 REVISTA DE MATEMATICĂ MEHEDINȚEANĂ NR. 17

SUMA CIFRELOR UNUI NUMĂR NATURAL

Daniel Stretcu

Definiție: Dacă n = 풂풌풂풌 ퟏ. . .풂ퟏ풂ퟎ este scris în baza 10, notăm s(n) = ∑ 풂풋풌풋 ퟎ și se numește

suma cifrelor lui n. PROPRIETĂȚI:

1) n – s(n) ⋮ 9, (∀) n ∈ ℕ 2) s(m + n) ≤ s(m) + s(n), (∀) n,m ∈ ℕ 3) s(m ∙ n ) ≤ n ∙ s(m), (∀) m,n ∈ ℕ 4) s(m ∙ n) ≤ s(m) ∙ s(n), (∀) m,n ∈ ℕ.

Probleme propuse:

1. Fie s(n) suma cifrelor lui n ∈ ℕ. Rezolvați ecuația: 풏+ 풔(풏) + 풔 풔(풏) = ퟐퟎퟏퟕ. Soluție: Transformarea c lasă restul mod 3 neschimbat. Așadar ecuația, modulo 3, are forma 0 = 1. Nu există soluții. 2. Se poate ca suma cifrelor unui pătrat perfect să fie a) 3; b) 2037 ? Soluție: a) Nu, un pătrat perfect divizibil cu 3 este divizibil cu 9; b) Același argument. 3. Fie D(n) suma cifrelor unui număr natural n. a) Există n astfel incat n + D(n) = 1980 ? b) Să se demonstreze că cel puțin unul din două numere naturale consecutive este de

forma: 푬풏 = 풏+ 푫(풏) (OMUS 1980)

Soluție: a) 1962 + D(1962) = 1980 b) Dacă n se termină cu 9, atunci 퐸 < 퐸 , dacă nu 퐸 = 퐸 + 2. Pentru orice m >2, m

∈ ℕ, alegem pe cel mai mare N pentru care 퐸 < 푚. Atunci 퐸 ≥ 푚 și ultima cifră a lui N nu este 9. Prin urmare, fie 퐸 = 푚 sau 퐸 = 푚 + 1.

4. Pentru fiecare număr natural n notăm cu s(n) suma cifrelor sale. Fie a un număr natural cu 2012 cifre, care este divizibil cu 9. Arătați că numărul s(s(s(a))) este pătrat perfect.

OJ. 2012, clasa a VI-a Soluție: Numărul s(a) este divizibil cu 9 și este cel mult egal cu 2012 ∙ 9 = 18108. Cum s(a) are

cel mult 5 cifre, rezultă că numărul s(s(a)) este divizibil cu 9 și este cel mult egal cu 45. Rezultă că s(s(s(a))) = 9, care este pătrat perfect. 5. Primul termen al unui șir este egal cu 1, iar fiecare termen următor, începând cu al

doilea, este egal cu suma dintre numărul precedent și suma cifrelor lui. Putem obține astfel numărul 765432 ?

Soluție: Răspunsul este negativ. Restul la împărțirea oricărui număr din șir cu 9 poate fi egal numai cu anumite valori, anume 1,2,4,8,7 sau 5. Într-adevăr, dacă numărul m dă restul r, atunci suma cifrelor lui dă tot restul r, deci suma dintre el și suma cifrelor lui este congruentă cu 2r modulo 9. Rezultă că șirul resturilor este periodic, având perioada egală cu secvența enunțată la început. Dar numărul 765432 are restul egal cu 0.

6. Pentru numărul n notăm s(n) suma cifrelor sale. Să se arate că ecuația: s(풙ퟐ) + s(풚ퟐ) = s(풛ퟐ) are o infinitate de soluții. Soluție: Pentru x = 10 − 1 avem 푥 = 99 … 98

00 … 01

și deci s(푥 ) = 9n. Numerele

x = 10 − 1, y = 10 − 1 și z = 10 − 1 verifică ecuația pentru orice m,n ∈ ℕ∗.

Page 5: SOCIETATEA DE ȘTIINȚE - ssmrmh.ro · S.S.M.ROMÂNIA - Filiala Mehedinți 2016 6 REVISTA DE MATEMATICĂ MEHEDINȚEANĂ NR. 17 PRINCIPIUL INCLUDERII ȘI EXCLUDERII Daniel Stretcu

S.S.M.ROMÂNIA - Filiala Mehedinți 2016

5 REVISTA DE MATEMATICĂ MEHEDINȚEANĂ NR. 17

7. Un număr natural n are cinci cifre 2 și cinci cifre 5, restul cifrelor fiind 0. Să se arate că n nu este pătrat perfect. (Probleme de aritmetica și teoria numerelor,autor

Laurențiu Panaitopol ,2006,pr.24/pg 38) Soluție: Suma cifrelor este s(n) = 35 = 3k + 2. Deoarece n ≡ s(n)(mod 9), rezultă că n ≡ s(n)(mod

3), adică n = 3k + 2. Pătratele perfecte sunt însă de forma 3k sau 3k + 1. Deci n nu este pătrat perfect. 8. Dacă p este prim și 풔풑(풏) este suma cifrelor numărului n scris în baza p, atunci funcția

Legendre 풆풑(풏) verifică relația: 풆풑(풏) = 풏 풔풑(풏)풑 ퟏ

.

Soluție: Reamintim: se notează 푒 (푛) exponentul lui p din descompunerea în factori primi a lui n! Și aceasta se numește funcția lui Legendre, calculându-se cu formula:

푒 (푛) = 푛푝

Dacă n = ∑ 푎 푝 ⇒ 푒 (푛) = 푎 푝 + 푎 푝 +⋯+ 푎 푝+ 푎 + (푎 푝 +

+푎 푝 +... + 푎 푝 + 푎 ) + ... + (푎 푝 + 푎 ) + 푎 = ⋯ ( )

=

= ( ).

9.Să se arate că s(8n) ≥ ퟏퟖ풔(풏).

Soluție: Avem s(n) = s(1000n) = s(125∙ 8푛) ≤ s(125)∙ s(8n) ≤ 8푠(8푛). Am folosit

s(mn) ≤ s(m) ∙ s(n).

ASUPRA UNEI ECUAȚII DIOFANTICE Nedeianu Dan

În cele ce urmează vom expune o metodă alternativă de rezolvare a unei probleme dată la Olimpiada Județeană de Matematică (19 Martie 2016), la concursul aferent clasei a VII – a. Enunțul problemei este următorul: „Determinați numerele naturale nenule 풙 și 풚 care verifică relația

풙 + 풚 = √풙 + 풚 + 풙풚.” Cele două variante oficiale de soluții pot fi vizualizate accesând link-ul „ssmr.ro/onm2016”. O altă soluție poate fi abordată folosind metoda completării la un pătrat.

Astfel, ecuația se mai scrie:

푥 − √푥 − 푥푦 + 푦 − 푦 = 0 ⇔ 푥 − √푥 1 + 푦 = 푦 − 푦. Completăm membrul stâng la un

pătrat în sensul că adunăm ( √ )

. Se va obține:

푥 − √푥 1 + 푦 +1 + 푦

4= 푦 − 푦 +

1 + 푦4

⇔ 0 ≤ √푥 −1 + 푦

2=

4 푦 − 4푦+ 1 + 2 푦 + 푦4

⇒−3푦 + 6 푦 + 1

4≥ 0 ⇒ 1 ≥ 3푦 − 6 푦 ⇔ 1 ≥ 3 푦 − 2 푦 + 1 − 3 ⇔

⇔ 4 ≥ 3 푦 − 1 și cum 푦 ∈ ℕ ⇒ 푦 ∈ {1,2,3,4}. Prin simetrie avem și 푥 ∈ {1,2,3,4}. Convin cazurile (1,4), (4,1), (4,4), care reprezintă soluțiile ecuației.

Page 6: SOCIETATEA DE ȘTIINȚE - ssmrmh.ro · S.S.M.ROMÂNIA - Filiala Mehedinți 2016 6 REVISTA DE MATEMATICĂ MEHEDINȚEANĂ NR. 17 PRINCIPIUL INCLUDERII ȘI EXCLUDERII Daniel Stretcu

S.S.M.ROMÂNIA - Filiala Mehedinți 2016

6 REVISTA DE MATEMATICĂ MEHEDINȚEANĂ NR. 17

PRINCIPIUL INCLUDERII ȘI EXCLUDERII

Daniel Stretcu

Definiție: Numim cardinalul unei mulțimi numărul de elemente al mulțimii. Cardinalul unei mulțimi 퐴 se notează card퐴 sau |퐴|.

Regula sumei: Dacă 퐴,퐵 sunt două mulțimi finite disjuncte, atunci: |퐴 ∪ 퐵| = |퐴| + |퐵|

Exemple:

1. Pe un raft sunt 7 cărți diferite de matematică și 8 cărți diferite de literatură. O carte poate fi aleasă în 7 + 8 = 15 moduri.

2. Un elev trebuie să aleagă o problemă din 3 liste diferite. Prima listă conține 12 probleme, a doua 8 probleme iar a treia 15. Nici o problemă nu apare în mai multe liste. Câte probleme posibile există?

Soluție: Aplicând regula sumei avem: 12 + 8 + 15 = 35 de probleme propuse.

Principiul includerii și excluderii: Este o generalizare a regulei sumei. El dă o relație prin care se poate calcula cardinalul reuniunii unor mulțimi finite, nu neapărat disjuncte.În cazul a două mulțimi, avem:

Teorema 1: Dacă 퐴 și 퐵 sunt două mulțimi finite, atunci: c

card(퐴 ∪ 퐵) = card퐴 + card퐵 − card(퐴 ∩ 퐵)sau |퐴 ∪ 퐵| = |퐴| + |퐵| − |퐴 ∩ 퐵|

Aplicația 1: Câte numere naturale mai mici sau egale cu 2016 sunt divizibile cu 2 sau cu 3?

Soluție: Fie 퐴 = {2푛|푛 ∈ 푁, 2푛 ≤ 2016} și 퐵 = {3푛|푛 ∈ 푁, 3푛 ≤ 2016}. Cum (2,3) = 1, avem că: 퐴 ∩ 퐵 = {6푛|푛 ∈ 푁, 6푛 ≤ 2016}. Evident, numărul căutat este:

|퐴 ∪ 퐵| = |퐴| + |퐵|− |퐴 ∩ 퐵| = 1009 + 673− 337 = 1345

Aplicația 2: Locuitorii unui sat vorbesc fie limba română, fie limba sârbă. Dacă 72% vorbesc româna și 61% sârba, câți vorbesc ambele limbi?

Soluție: Deoarece datele din ipoteză sunt exprimate procentual și nu cunoaștem numărul locuitorilor satului, este recomandat să luăm ca număr de locuitori ai satului numărul natural 100. Notăm cu 퐴 mulțimea locuitorilor satului, cu 퐵 mulțimea vorbitorilor de limba română și cu 퐶 mulțimea vorbitorilor de limba sârbă. Atunci: |퐴| = |퐵 ∪ 퐶| = |퐵| + |퐶| − |퐵 ∩ 퐶| și avem:

100 = 72 + 61− |퐵 ∩ 퐶|, deci |퐵 ∩ 퐶| = 33. Așadar, 33% dintre locuitori vorbesc ambele limbi.

Pentru trei mulțimi principiul includerii și excluderii se formulează astfel:

Teorema 2: Dacă 퐴,퐵 și 퐶 sunt trei mulțimi finite, avem:

card(퐴 ∪ 퐵 ∪ 퐶) = card퐴 + card퐵 + card퐶 − card(퐴 ∩ 퐵) − card(퐴 ∩ 퐶) − card(퐵 ∩ 퐶) + card(퐴 ∩ 퐵 ∩ 퐶) sau scris altfel:

|퐴 ∪ 퐵 ∪ 퐶| = |퐴| + |퐵| + |퐶| − |퐴 ∩ 퐵| − |퐴 ∩ 퐶|− |퐵 ∩ 퐶| + |퐴 ∩ 퐵 ∩ 퐶|

Aplicația 1: Câte numere naturale de la 1 la 2016 se divid sau cu 2, sau cu 3, sau cu 5?

Soluție: Fie 퐴 = {2푛|푛 ∈ 푁∗, 2푛 ≤ 2016} și 퐵 = {3푛|푛 ∈ 푁∗, 3푛 ≤ 2016} și

Page 7: SOCIETATEA DE ȘTIINȚE - ssmrmh.ro · S.S.M.ROMÂNIA - Filiala Mehedinți 2016 6 REVISTA DE MATEMATICĂ MEHEDINȚEANĂ NR. 17 PRINCIPIUL INCLUDERII ȘI EXCLUDERII Daniel Stretcu

S.S.M.ROMÂNIA - Filiala Mehedinți 2016

7 REVISTA DE MATEMATICĂ MEHEDINȚEANĂ NR. 17

퐶 = {5푛|푛 ∈ 푁∗, 5푛 ≤ 2016}. Cum (2,3) = 1, (2,5) = 1 și (3,5) = 1, avem:

퐴 ∩ 퐵 = {6푛|푛 ∈ 푁∗, 6푛 ≤ 2016},퐴 ∩ 퐶 = {10푛|푛 ∈ 푁∗, 10푛 ≤ 2016},

퐵 ∩ 퐶 = {15푛|푛 ∈ 푁∗, 15푛 ≤ 2016} și 퐴 ∩ 퐵 ∩ 퐶 = {30푛|푛 ∈ 푁∗, 30푛 ≤ 2016}.

Din principiul includerii și excluderii avem:

|퐴 ∪ 퐵 ∪ 퐶| = |퐴| + |퐵| + |퐶|− |퐴 ∩ 퐵| − |퐴 ∩ 퐶| − |퐵 ∩ 퐶| + |퐴 ∩ 퐵 ∩ 퐶| =

=2016

2+

20163

+2016

5−

20166

−2016

10−

201615

−2016

30=

= 1008 + 672 + 403− 336 − 201− 134 + 67 = 2083 − 366 − 201− 134 + 67 = 1479 de numere divizibile cu 2 sau cu 3 sau cu 5.

Aplicația 2: Într-o clasă cu 27 elevi, 12 elevi joacă fotbal, 13 elevi joacă baschet și 10 elevi joacă volei. Se știe că 4 elevi joacă și fotbal și baschet, 3 elevi joacă și baschet și volei, iar 2 elevi joacă și fotbal și volei. Aflați câți elevi joacă și fotbal și baschet și volei.

Soluție: Vom nota cu 퐴 mulțimea elevilor care joacă fotbal, 퐵 mulțimea elevilor care joacă baschet și 퐶 mulțimea elevilor care joacă volei. Atunci, mulțimea elevilor din clasă este 퐴 ∪ 퐵 ∪ 퐶, mulțimea elevilor care joacă și fotbal și baschet este 퐴 ∩ 퐵, mulțimea elevilor care joacă și baschet și volei este 퐵 ∩ 퐶, iar mulțimea elevilor care joacă fotbal și volei este 퐴 ∩ 퐶.Mulțimea căutată este 퐴 ∩ 퐵 ∩ 퐶 a elevilor care joacă și fotbal și baschet. Din principiul includerii și excluderii avem:

card(퐴 ∪ 퐵 ∪ 퐶) = card퐴 + card퐵 + card퐶 − card(퐴 ∩ 퐵) − card(퐴 ∩ 퐶) − card(퐵 ∩ 퐶) + card(퐴 ∩ 퐵 ∩ 퐶) și cum:

card(퐴 ∪ 퐵 ∪ 퐶) = 27, card퐴 = 12, card퐵 = 13, card퐶 = 10, card(퐴 ∩ 퐵) = 4, card(퐴 ∩ 퐶) =2 și card (퐵 ∩ 퐶) = 3, înlocuind obținem: 27 = 12 + 13 + 10− 4 − 3 − 2 + card(퐴 ∩ 퐵 ∩ 퐶) de unde: card(퐴 ∩ 퐵 ∩ 퐶) = 1, deci un singur elev joacă și fotbal și baschet și volei.

Aplicația 3: Printre cei 100 de elevi ai unui liceu s-a stabilit că preferințele pentru înghețată cu vanilie, ciocolată și zmeură sunt: 50 elevi consumă înghețată cu vanilie, 43 înghețată cu ciocolată, 28 înghețată cu zmeură, 13 cu vanilie și ciocolată, 11 cu ciocolată și zmeură, 12 cu zmeură și vanilie, iar 5 toate cele trei feluri.Determinați numărul de elevi care consumă următoarele feluri de înghețată:

1. cu vanilie sau ciocolată sau zmeură; 2. cu ciocolată dar nu cu zmeură; 3. cu ciocolată și cu zmeură;

4. cu vanilie sau ciocolată, dar nu cu zmeură.

Soluție: Fie 푉,푍,퐶 mulțimile de elevi care consumă înghețată cu vanilie, zmeură și respectiv ciocolată:

|푉| = 50, |퐶| = 43, |푍| = 28, |푉 ∩ 퐶| = 13, |퐶 ∩ 푍| = 11, |푍 ∩ 푉| = 12, |푉 ∪ 푍 ∪ 퐶| = 5

1. |푉 ∩ 퐶 ∩ 푍| = 90, 2. 24 + 8 = 32, 3. 6 , 4. 30 + 8 + 24 = 62

Probleme propuse:

1. Dintre 100 consumatori de cafea, 70 o consumă cu zahăr, 60 cu frișcă, iar 50 cu zahăr și frișcă.

a. Câți consumatori de cafea cu zahăr sau frișcă există?

b. Câți dintre ei consumă fără zahăr și fără frișcă?

Page 8: SOCIETATEA DE ȘTIINȚE - ssmrmh.ro · S.S.M.ROMÂNIA - Filiala Mehedinți 2016 6 REVISTA DE MATEMATICĂ MEHEDINȚEANĂ NR. 17 PRINCIPIUL INCLUDERII ȘI EXCLUDERII Daniel Stretcu

S.S.M.ROMÂNIA - Filiala Mehedinți 2016

8 REVISTA DE MATEMATICĂ MEHEDINȚEANĂ NR. 17

2. Câte numere naturale mai mici sau egale cu 100 sunt divizibile sau cu 3 sau cu 5?

3. Toți locuitorii dintr-un oraș vorbesc fie franceza, fie germana. Dacă 63% vorbesc franceza și 59% germana, câți vorbesc ambele limbi?

4. Câte numere naturale mai mici sau egale cu 1000 nu se divid nici cu 2, nici cu 3, nici cu 5?

5. Dintre participanții la o conferință, 21 cunosc limba franceză, 18 cunosc limba franceză, 11 cunosc limba germană, 5 cunosc și franceza și engleza, 6 cunosc engleza și germana, 3 cunosc franceza și germana, 4 cunosc toate cele trei limbi, iar unul nu cunoaște niciuna dintre aceste limbi. Câți participanți sunt la conferință?

Bibliografie:

1. Dan Schwarz, Gabriel Popa – Probleme de numărare, Gil, Zalău, 2007

2. Mircea Ganga, Manual pentru clasa a 푋 – a , Mathpress, Ploiești, 2008

3. ViitoriOlimpici.ro și Concursul Gazeta Matematică

PUNCTUL ȘI TEOREMA LUI GERGONNE Radu-Cătălin Șeitan

Radu-Cătălin Șeitan-este elev în clasa a IX-a la Colegiul Național „Traian” din Drobeta Turnu-Severin-profesor Dana Paponiu. În anul 2016 a obținut premiul al doilea la Concursurile Interjudețene „Ion Ciolac” și „Modus Vivendi” și o medalie de argint la Olimpiada de Matematică-Etapa Națională. Membrii Filialei Mehedinți a SSMR îi ureaza succes în continuare!

( Președinte SSMR-MH Daniel Sitaru)

1. Teorema lui Gergonne: Această teoremă este puțin cunoscută, dar poate fi aplicată în rezolvarea multor probleme, uneori conducând la găsirea unor soluții mai rapide decât cele rezultate prin metode „clasice”. Fie un triunghi 훥퐴퐵퐶 ,퐷 ∈ (퐵퐶),퐸 ∈ (퐴퐶),퐹 ∈ (퐴퐵). Dacă 퐴퐷,퐵퐸 și 퐶퐹 sunt concurente în punctul 푀, atunci: + + = 1. Demonstrație:

Fie 퐴푋 ⊥ 퐵퐶 și fie 푀푌 perpendiculara dusă din 푀 pe 퐵퐶, unde 푋,푌 ∈ (퐵퐶).

⇒ 퐴 = ⋅ și 퐴 = ⋅ ⇒ = (1) . Cum dreptele 퐴푋 și 푀푌 sunt paralele (sunt

perpendiculare pe aceeași dreaptă) ⇒ 훥푀퐷푌~훥푀푋퐴 ⇒ = (2) .Din (1) și (2) ⇒ = .

Obținem în mod analog că = și = . Adunând aceste ultime trei relații obținem că :

Page 9: SOCIETATEA DE ȘTIINȚE - ssmrmh.ro · S.S.M.ROMÂNIA - Filiala Mehedinți 2016 6 REVISTA DE MATEMATICĂ MEHEDINȚEANĂ NR. 17 PRINCIPIUL INCLUDERII ȘI EXCLUDERII Daniel Stretcu

S.S.M.ROMÂNIA - Filiala Mehedinți 2016

9 REVISTA DE MATEMATICĂ MEHEDINȚEANĂ NR. 17

+ + = = = 1, ceea ce trebuia demonstrat. 1.1. Probleme

rezolvate:

Fie un triunghi 훥퐴퐵퐶,퐷 ∈ (퐵퐶) și 푀 ∈ (퐴퐷) astfel încât 퐴푀 = 푀퐷. Fie 퐹 ∈ (퐴퐵) și 퐸 ∈ (퐴퐶) astfel încât 퐴퐷,퐵퐸 și 퐶퐹 sunt concurente în punctul 푀. Dacă 퐵푀 = 2푀퐸, demonstrați că 퐶푀 = 5푀퐹.

Demonstrație: Aplicăm teorema lui Gergonne și obținem imediat că + + = 1. Cum

퐴푀 = 푀퐷 ⇒ = 1 ⇒ = . Cum 퐴푀 = 푀퐷 ⇒ = 1 ⇒ = . Cum 퐵푀 = 2푀퐸 ⇒ = .

După ce înlocuim în suma de rapoarte, obținem că + + = 1 ⇒ = 1 − − = ⇒ 퐶퐹 =6퐹푀 ⇒ 퐶푀 = 5푀퐹.

1.2. Probleme propuse: 1. Fie un triunghi 훥퐴퐵퐶 și 푃 ∈ (퐵퐶),푀 ∈ (퐴퐷),푅 ∈ (퐴퐵) și 푄 ∈ (퐴퐶). Știind că 퐴푀 = 푛 ⋅ 푀푃,푄푀 = .퐵푀 și că 퐴푃 ⋅ 푅푀 ⋅ 푛 = 푀푃 ⋅ 푅퐶 ⋅ (푛 + 2), stabiliți dacă 퐴푃,퐵푄

și 퐶푅 sunt concurente în 푀, unde 푛 ∈ ℕ ∖ {0,1}. Valorile rapoartelor , , , exprimate în

centimetri, pot reprezenta lungimile laturilor unui triunghi?

2. Fie un triunghi 훥퐴퐵퐶 și 퐸 ∈ (퐵퐶),퐺 ∈ (퐴퐵) și 퐹 ∈ (퐴퐶). Dacă 퐴퐸,퐶퐺 și 퐵퐹 sunt concurente în 푀 și dacă = , cu 푀퐹 ⋅ 퐶푀 = 퐵푀 ⋅ 푀퐺 = 3푀퐹 ⋅ 푀퐺, arătați că 푀 este mijlocul lui [퐴퐸].

2. Punctul lui Gergonne:

Acest punct al lui Joseph Gergonne (matematician francez) este unul puțin cunoscut, dar în legătură cu care pot fi făcute mai multe observații interesante.Fie un cerc și 훥퐴퐵퐶 circumscris acestuia. Fie 푀,푁 și 푃 punctele de tangentă ale cercului la laturile 퐵퐶,퐴퐵, respectiv 퐴퐶. Atunci 퐴푀,퐶푁 și 퐵푃 sunt concurente în 퐺푒, punctul lui Gergonne.

Demonstrație: Tangentele duse dintr-un punct exterior la un cerc sunt congruente ⇒ 푀퐶 = 퐶푃 = 푥. 퐵푀 = 퐵푁 = 푦,퐴푁 = 퐴푃 = 푧 (cu 푥,푦 și 푧 reale pozitive).

⇒ ⋅ ⋅ = ⋅ ⋅ = 1. Din reciproca teoremei lui Ceva, rezultă că 퐴푀,퐵푃 și 퐶푁 sunt

concurente în 퐺푒.

2.1. Probleme rezolvate: 1. Fie un cerc și un triunghi 훥퐴퐵퐶 circumscris acestuia. Fie 푀,푁 și 푃 punctele de tangență ale cercului la laturile 퐵퐶,퐴퐵, respectiv 퐴퐶 și 퐺푒 punctul lui Gergonne corespunzător triunghiului. Demonstrați că = ( )

( )( ) , = ( )( )( ) și că = ( )

( )( ),

Page 10: SOCIETATEA DE ȘTIINȚE - ssmrmh.ro · S.S.M.ROMÂNIA - Filiala Mehedinți 2016 6 REVISTA DE MATEMATICĂ MEHEDINȚEANĂ NR. 17 PRINCIPIUL INCLUDERII ȘI EXCLUDERII Daniel Stretcu

S.S.M.ROMÂNIA - Filiala Mehedinți 2016

10 REVISTA DE MATEMATICĂ MEHEDINȚEANĂ NR. 17

unde 푝 este semiperimetrul triunghiului, 푎 este lungimea [퐵퐶],푏 este lungimea [퐴퐶] și 푐 este lungimea [퐴퐵].

Demonstrație: Folosim desenul de mai sus. Tangentele duse dintr-un punct exterior la un cerc sunt congruente. ⇒ 푀퐶 = 퐶푃 = 푥,퐵푀 = 퐵푁 = 푦,퐴푁 = 퐴푃 = 푧 (cu 푥, 푦 și 푧 reale pozitive). Aplicăm teorema Van Aubel și obținem că = + ⇒ = + = ( )

Cum perimetrul triunghiului este egal cu 푃 = 2푥 + 2푦 + 2푧 ⇒ 푝 = 푥 + 푦 + 푧 ⇒ 푧 = 푝 − 푥 − 푦. Cum 푥 + 푦 = 푎 = 퐵퐶 ⇒ 푧 = 푝 − 푎. Obținem în mod analog că 푥 = 푝 − 푐 și 푦 = 푝 − 푏. Înlocuind în relația de mai sus obținem că = ( )

( )( ). Procecdăm în mod analog și obținem și celelalte relații:

= ( )( )( ) și = ( )

( )( ).

2. Fie un triunghi 훥퐴퐵퐶 și cercul înscris în el. Fie 푃,푄 și 푅 punctele de tangență ale acestui cerc la laturile 퐵퐶,퐴퐶 și 퐴퐵 ale triunghiului. Prin 퐴 ducem o dreaptă 푑 paralelă cu 퐵퐶, prin 퐵 ducem o dreaptă 푑 paralelă cu 퐴퐶 și prin 퐶 ducem o dreaptă 푑 paralelă cu 퐴퐵. Fie 푃푅 ∩ 푑 = {퐸},푃푄 ∩푑 = {퐹},푅푄 ∩ 푑 = {퐺},

푅푃 ∩ 푑 = {퐻}.푄푃 ∩ 푑 = {퐽},푄푅 ∩ 푑 = {퐾}. Acum fie 퐸푄 ∩ 퐹푅 = {푆 },

퐻푄 ∩ 퐺푃 = {푆 } și 퐽푅 ∩ 퐾푃 = {푆 }. Demonstați că punctele 푆 ,푄, 푆 ,푃 și 푅 sunt conciclice. Demonstrație: Începem rezolvarea problemei cu o lemă.

Lemă: Fie 푀 ∈ 퐵퐶 ,푁 ∈ 퐶퐴 și 푃 ∈ 퐴퐵 trei puncte pe laturile triunghiului 훥퐴퐵퐶 și dreapta 푑 paralelă cu 퐵퐶, cu 퐴 ∈ 푑. Notăm 푀푃 ∩ 푑 = {푇} și 푀푁 ∩ 푑 = {푆}. Dreptele 퐴푀,퐵푀 și 퐶푃 sunt concurente ⇔ 퐴푇 = 퐴푆. Demonstrație lemă:

Cum 푑 este paralelă cu 퐵퐶, obținem din teorema fundamentală a asemănării că 훥푀푁퐶~훥푆푁퐴 și că 훥퐵푃푀~훥퐴푃푇.⇒ = și = . Din teorema lui Ceva obținem că 퐴푀,퐵푁 și 퐶푃 sunt

concurente ⇔ ⋅ ⋅ = 1 ⇔ ⋅ ⋅ = 1 ⇔ = 1 ⇔ 퐴푇 = 퐴푆. Revenim la problemă.

Vom demonstra mai întâi că 푆 aparține cercului înscris al 훥퐴퐵퐶. Aplicăm lema de mai sus. Cum 퐴푃,퐵푄 și 퐶푅 sunt concurente în punctul lui Gergonne ⇒ 퐴퐹 = 퐴퐸. Tangentele duse dintr-un punct exterior la un cerc sunt congruente. ⇒ 훥퐶푃푄,훥퐵푅푃 și 훥퐴푄푅 sunt isoscele. ⇒ ∢퐶푄푃 ≡ ∢퐶푃푄. Dar 퐵퐶 este paralelă cu 푑 ⇒ ∢퐶푃푄 ≡ ∢퐴퐹푄 și ∢퐴푄퐹 ≡ ∢퐶푄푃 (opuse la vârf) ⇒ ∢퐴푄퐹 ≡ ∢퐴퐹푄 ⇒훥퐴푄퐹 este isoscel ⇒ 퐴퐹 = 퐴푄. Se arată analog că 퐴퐸 = 퐴푅 ⇒ 퐴퐸 = 퐴퐹 = 퐴푄 = 퐴푅. În

Page 11: SOCIETATEA DE ȘTIINȚE - ssmrmh.ro · S.S.M.ROMÂNIA - Filiala Mehedinți 2016 6 REVISTA DE MATEMATICĂ MEHEDINȚEANĂ NR. 17 PRINCIPIUL INCLUDERII ȘI EXCLUDERII Daniel Stretcu

S.S.M.ROMÂNIA - Filiala Mehedinți 2016

11 REVISTA DE MATEMATICĂ MEHEDINȚEANĂ NR. 17

훥푄퐹퐸[푄퐴] este mediană și 푄퐴 = 퐴퐸 = 퐴퐹. Deci 푄퐴 = ⇒ 훥푄퐹퐸 este dreptunghic în 푄 ⇒ 퐸푄 este perpendiculară pe 퐹푃. Se arată în mod analog că 퐹푅 este perpendiculară pe 퐸푃. Cum 퐸푄 ∩ 퐹푅 = {푆 } ⇒ 푆 este ortocentrul triunghiului 훥푃퐹퐸(퐹푅 și 퐸푄 sunt înălțimi). Cum 푚(∢푆 푄푃) = 푚(∢푆 푅푃) = 90° ⇒ 푆 푅푃푄 este inscriptibil (unghiurile opuse sunt suplementare). Cum deja 푅,푃 și 푄 aparțin cercului înscris în 훥퐴퐵퐶 ⇒ 푆 aparține cercului înscris în 훥퐴퐵퐶.Apoi se arată în mod analog că și 푆 și 푆 aparțin cercului înscris în 훥퐴퐵퐶. Cum 푆 ,푄, 푆 ,푃, 푆 și 푅 aparțin toate acestui cerc ⇒ ele sunt conciclice. Notă! Doar într-un triunghi dreptunghic mediana corespunzătoare ipotenuzei este egală cu jumătate din lungime ipotenuzei.

2.2. Probleme propuse:

1. Fie un cerc și un triunghi 훥퐴퐵퐶 circumscris acestuia. Fie 푀,푁 și 푃 punctele de tangență ale cercului la laturile 퐵퐶,퐴퐵, respectiv 퐴퐶 și fie 퐺푒 punctul lui Gergonne corespunzător triunghiului. Demonstrați că ⋅ ⋅ = unde 푅 este raza cercului circumscris și 푟 raza cercului înscris ale 훥퐴퐵퐶. Indicație: Aveți în vedere prima problemă rezolvată de la 2.1.

2. Fie un triunghi neisoscel 훥퐴퐵퐶 și cercul înscris în el. Fie 푃,푄 și 푅 punctele de tangență ale acestui cerc la laturile 퐵퐶,퐴퐶 și 퐴퐵 ale triunghiului. Fie 푅푄 ∩ 퐵퐶 = {퐴 },푃푅 ∩ 퐴퐶 = {퐵 } și 푃푄 ∩ 퐴퐵 ={퐶 }. Demonstrați că 퐴 ,퐵 și 퐶 sunt coliniare (dreapta lui Gergonne).

3. Fie un triunghi echilateral 훥퐴퐵퐶. Demonstrați că punctul lui Gergonne coincide cu centru cercului înscris în acest triunghi.

Bibliografie:

[1] Mihailescu C, Geometria elementelor remarcabile, Societatea de Științe Matematice din România, București, 2007. [2] Nicula V, Geometria plană (sintetică, vectorială, analitică). Culegere de probleme, Editura Gil Zalău, 2002. [3]ro.math.wikia.com

PERPENDICULARITATE ȘI RELAȚII GENERATE DE ACEASTA ÎNTR-UN TRIUNGHI DREPTUNGHIC

Carmen – Victorița Chirfot

Fie triunghiul 푨푩푪, dreptunghi în 푨 cu 푨푩 = 풄, 풄 > 0 și 푨푪 = 풃,풃 > 0. Atunci 푩푪 = 풃ퟐ + 풄ퟐ. Deducem 푨푷ퟏ ⊥ 푩푪,푷ퟏ ∈ (푩푪). Fie apoi 푷ퟏ푷ퟐ ⊥ 푨푩,

h5

h1

h2 h3

A B

C

P2 P4 P6 P8

P1

P3

P5

P7 P92h4

Page 12: SOCIETATEA DE ȘTIINȚE - ssmrmh.ro · S.S.M.ROMÂNIA - Filiala Mehedinți 2016 6 REVISTA DE MATEMATICĂ MEHEDINȚEANĂ NR. 17 PRINCIPIUL INCLUDERII ȘI EXCLUDERII Daniel Stretcu

S.S.M.ROMÂNIA - Filiala Mehedinți 2016

12 REVISTA DE MATEMATICĂ MEHEDINȚEANĂ NR. 17

푷ퟐ ∈ (푨푩); 푷ퟐ푷ퟑ ⊥ 푩푪,푷ퟑ ∈ (푩푪); 푷ퟑ푷ퟒ ⊥ 푨푩,푷ퟒ ∈ (푨푩) și așa mai departe generăm șirul perpendicularelor analoage. Notăm 푨푷ퟏ = 풉ퟏ,푷ퟏ푷ퟐ = 풉ퟐ,푷ퟐ푷ퟑ = 풉ퟑ,푷ퟑ푷ퟒ = 풉ퟒ,푷ퟒ푷ퟓ = 풉ퟓ etc. Ne propunem să calculăm fiecare dintre lungimile 풉ퟏ,풉ퟐ,풉ퟑ,풉ퟒ,풉ퟓ și ceilalți termeni ai șirului astfel generat, dar în două moduri diferite, pentru a găsi o formulă de calcul pentru termenul general al șirului astfel creat. Ca înălțime în ∆푩푨푪,풉ퟏ = 풃풄

풃ퟐ 풄ퟐ . În triunghiul 푨푷ퟏ푩 dreptunghic

în 푷, înălțimea corespunzătoare ipotenuzei este 풉ퟐ = 풉ퟏ⋅푷ퟏ푩풄

=풉ퟏ⋅ 풄ퟐ 풉ퟏ

풄.

În mod analog, 풉ퟑ = 풉ퟐ⋅푷ퟐ푩푷ퟏ푩

=풉ퟐ⋅ 풄ퟐ 풉ퟏ

ퟐ 풉ퟐퟐ

풄ퟐ 풉ퟏퟐ

,풉ퟒ = 풉ퟑ⋅푷ퟑ푩푷ퟐ푩

=풉ퟑ⋅ 풄ퟐ 풉ퟏ

ퟐ 풉ퟐퟐ 풉ퟑ

풄ퟐ 풉ퟏퟐ 풉ퟐ

ퟐ, …,

풉풏 ퟏ =풉풏⋅ 풄ퟐ 풉ퟏ

ퟐ 풉ퟐퟐ ⋯ 풉풏ퟐ

풄ퟐ 풉ퟏퟐ 풉ퟐ

ퟐ ⋯ 풉풏 ퟏퟐ

adică 풉풏 ퟏ = 풉풏 ⋅풄ퟐ 풉ퟏ

ퟐ 풉ퟐퟐ ⋯ 풉풏ퟐ

풄ퟐ 풉ퟏퟐ 풉ퟐ

ퟐ ⋯ 풉풏 ퟏퟐ =

= 풉풏 ⋅ ퟏ − 풉풏ퟐ

풄ퟐ 풉ퟏퟐ 풉ퟐ

ퟐ ⋯ 풉풏 ퟏퟐ ,풏 ∈ 푵∗ − {ퟏ} (1)

Pe de altă parte, în triunghiul 푨푷ퟏ푩 dreptunghic în 푷, 퐬퐢퐧푩 = 풉ퟏ풄⇒ 풉ퟏ = 풄 ⋅ 퐬퐢퐧 푩. Analog,

풉ퟐ = 풄ퟐ − 풉ퟏퟐ ⋅ 퐬퐢퐧푩 ,풉ퟑ = 풄ퟐ − 풉ퟏퟐ − 풉ퟐퟐ ⋅ 퐬퐢퐧푩 , … ,풉풏 = 풄ퟐ − 풉ퟏퟐ −풉ퟐퟐ −⋯−풉풏 ퟏퟐ ⋅

퐬퐢퐧 푩 ,풏 ∈ 푵∗ − {ퟏ}, adică 풉풏ퟐ

풄ퟐ 풉ퟏퟐ 풉ퟐ

ퟐ ⋯ 풉풏 ퟏퟐ = 퐬퐢퐧푩. Revenim în relația (1) și obținem 풉풏 ퟏ = 풉풏 ⋅

√ퟏ − 퐬퐢퐧ퟐ 푩 = 풉풏 ⋅ 퐜퐨퐬푩 ,∀풏 ∈ 푵∗, deoarece 푩 ∈ ퟎ, 흅ퟐ

.

Deci, 풉풏 ퟏ = 풉풏 ⋅ 퐜퐨퐬푩 ,∀풏 ∈ 푵∗ cu 풉ퟏ = 풄 ⋅ 퐬퐢퐧푩 = 풃풄

풃ퟐ 풄ퟐ. Avem așadar,

풉ퟐ = 풉ퟏ ⋅ 푩,풉ퟑ = 풉ퟐ ⋅ 퐜퐨퐬푩 ,풉ퟒ = 풉ퟑ ⋅ 퐜퐨퐬푩 , … ,풉풏 ퟏ = 풉풏 ⋅ 퐜퐨퐬푩, 풏 ∈ 푵∗. Înmulțim ultimele 풏 relații (toți membrii sunt diferiți de ퟎ pentru că suntem în triunghiuri dreptunghice), rezultă că 풉풏 ퟏ = 풉ퟏ ⋅ 퐜퐨퐬풏 푩 = 풄 ⋅ 퐬퐢퐧푩 ⋅ 퐜퐨퐬풏 푩, ∀풏 ∈ 푵∗ cu 풉ퟏ = 풄 ⋅ 퐬퐢퐧 푩 =

풃풄

풃ퟐ 풄ퟐ.

Observație: Dacă considerăm triunghiul 퐴퐵퐶 într-un sistem de coordonate 푥푂푦 cu 퐴 = 푂(0,0),퐵(푐, 0),퐶(0, 푏), atunci

푃 ℎ − ℎ ,ℎ = 푃 ℎ − ℎ cos 퐵 ,ℎ cos퐵 =

= 푃 (ℎ sin퐵 ,ℎ cos퐵) = 푃√

sin퐵 ,√

cos퐵 . De asemenea,

푃 ℎ − ℎ ,ℎ = 푃 ℎ − ℎ cos 퐵 ,ℎ cos퐵 = 푃 (ℎ sin퐵 ,ℎ cos퐵) =

= 푃 (ℎ cos 퐵 sin퐵 ,ℎ cos 퐵) = 푃 (ℎ cos 퐵 sin퐵 ,ℎ cos 퐵) = = 푃

√cos 퐵 sin퐵 ,

√cos 퐵 . Prin inducție matematică, obținem că

= 푃푏푐

√푎 + 푏cos 퐵 sin퐵 ,

푏푐√푎 + 푏

cos 퐵 .

Pentru punctele 푃 de indice par, nu avem înălțime pe axa 푂푦, dar 푥 = 푥 , 푥 = 푥 , … , 푥 = 푥 ,푛 ∈ 푁. Deci, 푃

√sin퐵 , 0 ,

푃√

cos 퐵 sin퐵 , 0 și 푃√

cos 퐵 sin퐵 , 0 ,∀푛 ∈ 푁. Astfel, putem calcula distanța între oricare două puncte alese din figură, deci putem calcula ușor și celelalte diagonalele ale trapezelor formate. Probleme propuse: 1. Să se calculeze lungimea drumului 푨푷ퟏ푷ퟐ푷ퟑ …푷풏,풏 ∈ 푵∗ − {ퟏ}.

Page 13: SOCIETATEA DE ȘTIINȚE - ssmrmh.ro · S.S.M.ROMÂNIA - Filiala Mehedinți 2016 6 REVISTA DE MATEMATICĂ MEHEDINȚEANĂ NR. 17 PRINCIPIUL INCLUDERII ȘI EXCLUDERII Daniel Stretcu

S.S.M.ROMÂNIA - Filiala Mehedinți 2016

13 REVISTA DE MATEMATICĂ MEHEDINȚEANĂ NR. 17

Indicație: 퐴푃 + 푃 푃 + 푃 푃 +⋯+ 푃 푃 = ℎ + ℎ + ℎ + ⋯+ ℎ = = 푐 sin퐵 + 푐 sin퐵 cos퐵 + 푐 sin퐵 cos 퐵 +⋯+ 푐 sin퐵 cos 퐵 =

= 푐 sin퐵 (1 + cos퐵 + cos 퐵 + ⋯+ cos 퐵) =

= 푐 sin퐵 ⋅1− cos 퐵1− cos퐵

= 푐 ⋅ 2 sin퐵2

cos퐵2⋅

1− cos 퐵2 sin

= 푐 ⋅ cos퐵2⋅

1− cos 퐵sin

= 푐 ⋅ ctg퐵2

(1− cos 퐵).

Dacă 푛 → ∞ ⇒ 퐴푃 푃 푃 …푃 → 푐 ⋅ ctg . 2. Să se calculeze lungimea drumului 푨푷ퟐ푷ퟒ푷ퟔ …푷ퟐ풏,풏 ∈ 푵∗. Indicație:

퐴푃 + 푃 푃 + 푃 푃 + ⋯+ 푃 푃 = ℎ − ℎ + ℎ − ℎ + ⋯+ ℎ − ℎ =

= ℎ − ℎ cos 퐵 + ℎ − ℎ cos 퐵 + ⋯+ ℎ − ℎ cos 퐵 =

= ℎ sin퐵 + ℎ sin퐵 + ⋯+ ℎ sin퐵 = sin퐵 (ℎ + ℎ +⋯+ ℎ ) = = (푐 ⋅ sin퐵 + 푐 ⋅ sin퐵 ⋅ cos 퐵 + ⋯+ 푐 ⋅ sin퐵 ⋅ cos 퐵) sin퐵 =

= (푐 + 푐 ⋅ cos 퐵 + ⋯+ 푐 ⋅ cos 퐵) sin 퐵 = 푐 sin 퐵 ⋅1− (cos 퐵)

1− cos 퐵=

= 푐 sin 퐵 ⋅1− (cos 퐵)

1− sin 퐵= 푐(1 − cos 퐵) → 푐

3. Să se demonstreze că 퐜퐭퐠 푩ퟐ

> 1 + 퐜퐨퐬풏 푩, unde 푩 ∈ ퟎ, 흅ퟐ

și 풏 ∈ 푵∗. 4. Să se calculeze lungimea drumului 푷ퟏ푷ퟑ푷ퟓ …푷ퟐ풏 ퟏ,풏 ∈ 푵∗. Indicație: 푃 푃 + 푃 푃 + 푃 푃 + ⋯+ 푃 푃 =

= ℎ − ℎ + ℎ − ℎ + ⋯+ ℎ − ℎ =

= ℎ − ℎ cos 퐵 + ℎ − ℎ cos 퐵 +⋯+ ℎ − ℎ cos 퐵 =

= ℎ sin퐵 + ℎ sin퐵 + ⋯+ ℎ sin퐵 = sin퐵 (ℎ + ℎ +⋯+ ℎ ) = = (푐 ⋅ sin퐵 ⋅ cos퐵 + 푐 ⋅ sin퐵 ⋅ cos 퐵 + ⋯+ 푐 ⋅ sin퐵 ⋅ cos 퐵) sin퐵 =

= (푐 + 푐 ⋅ cos 퐵 +⋯+ 푐 ⋅ cos 퐵) cos퐵 sin 퐵 = 푐 ⋅ cos퐵 sin 퐵 ⋅1− (cos 퐵)

1 − cos 퐵=

= 푐 ⋅ cos퐵 ⋅ (1 − cos 퐵) → 푐 ⋅ cos퐵 5. Să se demonstreze că suma ariilor trapezelor formate aproximează aria triunghiului dreptunghic 푨푷ퟏ푩. 6. Să se calculeze distanțele 푷ퟏ푷ퟒ,푷ퟏ푷ퟔ,푨푷ퟐ풏 ퟏ,풏 ∈ 푵. 7. Să se calculeze : 퐥퐢퐦풏→ (푨푷ퟐ풏 ퟏ − 푨푷ퟐ풏 ퟏ) Bibliografie: [1]. Colecția Revistei de Matematică din Mehedinți. [2]. Colecția Revistei de Matematică din Timișoara.

AN INEQUALITY FROM GAZETA MATEMATICA- March 2016

Daniel Sitaru, Leonard Giugiuc, Alexander Bogomolny

Problem: An inequality with two solutions by Dan Sitaru and Leo Giugiuc has been just published in Gazeta Matematica (March 2016).

Let 퐚,퐛, 퐜 be positive numbers such that 퐚ퟐ + 퐛ퟐ + 퐜ퟐ = ퟑ. Prove that (퐚 + 퐜)(ퟏ + 퐛) ≤ ퟒ.

Page 14: SOCIETATEA DE ȘTIINȚE - ssmrmh.ro · S.S.M.ROMÂNIA - Filiala Mehedinți 2016 6 REVISTA DE MATEMATICĂ MEHEDINȚEANĂ NR. 17 PRINCIPIUL INCLUDERII ȘI EXCLUDERII Daniel Stretcu

S.S.M.ROMÂNIA - Filiala Mehedinți 2016

14 REVISTA DE MATEMATICĂ MEHEDINȚEANĂ NR. 17

Proof 1: Define matrix 퐴 =

1 푎푎 푏푏 푐푐 1

. We have:

퐴 ⋅ 퐴 = 1 푎 푏 푐푎 푏 푐 1 ⋅

1 푎푎 푏푏 푐푐 1

= 푎 + 푏 + 푐 + 1 푎 + 푎푏 + 푏푐 + 푐푎 + 푎푏 + 푏푐 + 푐 푎 + 푏 + 푐 + 1

=

= 4 (푎 + 푐)(1 + 푏)(푎 + 푐)(1 + 푏) 4

By Cauchy – Binet theorem, det(퐴 ⋅ 퐴) ≥ 0.

Therefore, [(푎 + 푐)(1 + 푏)] ≤ 16, or (푎 + 푐)(1 + 푏) ≤ 4.

Proof 2: We use spherical coordinates. Let 푏 = √3 cos 푡 ,푎 = √3 sin 푡 cos 푢, and

푐 = √3 sin 푡 sin푢, where 0 < 푡 < . We need to prove that:

√3(cos푢 + sin 푢) 1 + √3 cos 푡 sin 푡 ≤ 4.

Observe that 1 < sin 푢 + cos 푢 ≤ √2. Thus, suffice it to prove that √6 1 + √3 cos 푡 sin 푡 ≤ 4.

Consider the function 푓; 0, → ℝ, defined by 푓(푡) = 1 + √3 cos 푡 sin 푡. We have 푓 ʹ(푡) =

2√3 cos 푡 + cos 푡 − √3 sin 푡 which implies max 푓 arccos√

= 2 . Therefore,

√6 1 + √3 cos 푡 sin 푡 ≤ √6 ⋅ 2 = 4.

Proof 3: (푎 − 푏) + (푏 − 푐) + (푎 − 1) + (푐 − 1) ≥ 0 which simplifies to

(푎 + 푏 + 푐 ) + 1− 푎푏 − 푏푐 − 푎 − 푐 ≥ 0. This is exactly (푎 + 푐)(1 + 푏) ≤ 4

Proof 4: By the AM-QM inequalty, ≤ = 1. Further, by the AM-GM inequalty,

(푎 + 푐)(푏 + 1) ≤푎 + 푐 + 푏 + 1

2≤

3 + 12

= 4.

Proof 5: From 푎 + 푏 + 푐 = 3, 3∑푎 ≥ (∑푎) , implying (∑푎) ≤ 9, or 푎 + 푏 + 푐 ≤ 3.

푎 + 푏 + 푐 + 1 ≤ 4. Therefore, 4 ≥ (푎 + 푐) + (푏 + 1) ≥ 2 (푎 + 푐)(푏 + 1), i.e., 2 ≥ (푎 + 푐)(푏 + 1), or 4 ≥ (푎 + 푐)(푏 + 1).Equality is attained when 푎 + 푐 = 푏 + 1, which, with 푎 + 푏 + 푐 = 3 implies 푎 = 푏 = 푐 = 1.

Proof 6: From (푥 − 푦) ≥ 0 we have 2푥푦 ≤ 푥 + 푦 . We use this with the couples (푎, 푏), (푏, 푐), (1,푎), (1, 푐): 2푎푏 ≤ 푎 + 푏 , 2푏푐 ≤ 푏 + 푐 , 2푎 ≤ 1 + 푎 , 2푐 ≤ 1 + 푐

adding which gives 2(푎 + 푐 + 푎푏 + 푏푐) ≤ 2 + 2(푎 + 푏 + 푐 ) = 8, and this is exactly

Page 15: SOCIETATEA DE ȘTIINȚE - ssmrmh.ro · S.S.M.ROMÂNIA - Filiala Mehedinți 2016 6 REVISTA DE MATEMATICĂ MEHEDINȚEANĂ NR. 17 PRINCIPIUL INCLUDERII ȘI EXCLUDERII Daniel Stretcu

S.S.M.ROMÂNIA - Filiala Mehedinți 2016

15 REVISTA DE MATEMATICĂ MEHEDINȚEANĂ NR. 17

(푎 + 푐)(푏 + 1) ≤ 4.

Proof 7: To continue: (푎 + 푐)(1 + 푏) ≤ √2√3 − 푏 (1 + 푏) ≤ √2√4− 2푏(1 + 푏) = √2 (4− 2푏)(1 + 푏) =

= √2 (1 + 푏)(1 + 푏)(4− 2푏) ≤ √21 + 푏 + 1 + 푏 + 4− 2푏

3= 4.

The equality is achieved for 1 + 푏 = 1 + 푏 = 4− 2푏, making 푏 = 1 which, by the way, satisfies

3 − 푏 ≥ 0.푎 = 푐 = , i.e. 푎 = 푏 = 푐 = 1.

Proof 8: To continue: (푎 + 푐)(1 + 푏) ≤ √2√3 − 푏 (1 + 푏) ≤ √2 √

(1 + 푏) ≤ = 4.

Equality is achieved for 3 − 푏 = 1 + 푏 and 푎 = 푐 = , i.e., 푎 = 푏 = 푐 = 1.

Proof 9: Observe that (푎 + 푐)(1 + 푏) = 푎 ⋅ 1 + 푎 ⋅ 푏 + 푐 ⋅ 1 + 푏 ⋅ 푐 such that by the Cauchy – Schwarz inequality, (푎 ⋅ 1 + 푎 ⋅ 푏 + 푐 ⋅ 1 + 푏 ⋅ 푐) ≤ (푎 + 푏 + 푐 + 푏 )(1 + 푎 + 1 + 푐 )

which leads to a chain of inequalities:

(푎 + 푎푏 + 푐 + 푏푐) ≤ (3 + 푏 )(2 + 푎 + 푐 ), [(푎 + 푐)(1 + 푏)] ≤ (3 + 푏 )(2 + 3 − 푏 ) =

= (3 + 푏 )(5 − 푏 ) ≤3 + 푏 + 5 − 푏

2=

82

,

and, therefore, (푎 + 푐)(1 + 푏) ≤ 4.

Proof 10: From (푎 + 푐) ≤ 2(푎 + 푐 ) and (1 + 푏) ≤ 2(1 + 푏 ) we obtain a sequence of

inequalities: (푎 + 푐) (1 + 푏) ≤ 4(푎 + 푐 )(1 + 푏 ) ≤ 4 = 4 = 4 ⋅ 4.

and, therefore, (푎 + 푐)(1 + 푏) ≤ 4.

Proof 11: We prove (푎 + 푏)(1 + 푐) ≤ 16 when the point (푎, 푏, 푐) lies on the sphere of radius √3 centered at the origin. At height 푐 the sphere is a circle of radius 푟 = √3− 푐 and the maximum of 푎 + 푏 is 푟√2 (consider the line with slope −1 tangent to this circle in the first quadrant of the plane). We want thus the maximum of 2(1 + 푐 )(3− 푐 ) for 0 ≤ 푐 ≤ √3. The value 16 is attained for 푐 =1. But 16 − 2(1 + 푐) (3− 푐 ) = 2(푐 − 1) (5 + 4푐 + 푐 ) ≥ 0, for all real 푐.

Proof 12: Use Lagrange multipliers to prove that:

(1)max (푎 + 푐)(1 + 푏) = 4.Let 퐽 = (푎 + 푐)(1 + 푏) + 휆(푎 + 푏 + 푐 − 3).

Taking = = = = 0 yields: (2)1 + 푏 + 2푎휆 = 0, (3)1 + 푐 + 2푏휆 = 0,

(4)1 + 푏 + 2푐휆 = 0, (5)푎 + 푏 + 푐 = 3, (6)푎 = 푐 (from (2) and (4))

(7)2푎 + 2푎푏휆 = 0 (from (3) and (6)) ,(8)푏 + 푏 + 2푎푏휆 = 0 (from (2))

Page 16: SOCIETATEA DE ȘTIINȚE - ssmrmh.ro · S.S.M.ROMÂNIA - Filiala Mehedinți 2016 6 REVISTA DE MATEMATICĂ MEHEDINȚEANĂ NR. 17 PRINCIPIUL INCLUDERII ȘI EXCLUDERII Daniel Stretcu

S.S.M.ROMÂNIA - Filiala Mehedinți 2016

16 REVISTA DE MATEMATICĂ MEHEDINȚEANĂ NR. 17

(9)푏 + 푏 = 2푎 (from (7) and (8)), (10)2푎 + 푏 = 3 (from (5) and (6))

(11)2푏 + 푏 − 3 = 0, 푏 = 1,−3 2⁄ (from (9) and (10)), Hence, 푏 = 1 and from (10),푎 = ±1, implying 푎 = 푐 = 1 and 푏 = 1, and (1) follows.Remark : It is clear that the equality is attained for 풂 = 풃 = 풄 = ퟏ - a symmetric condition whereas the inequality itself is asymmetric. In analogy with the above derivation, we can show (풃+ 풂)(ퟏ+ 풄) ≤ ퟒ and (풃 + 풄)(ퟏ + 풂) ≤ ퟒ. The sum of the three gives (풂 + 풃 + 풄) + (풂풃+ 풃풄 + 풄풂) ≤ ퟔ which is just more symmetric.

Acknoledgment: Proofs 1 and 2 are by Dan Sitaru and Leo Giugiuc ; Proof 3 is by Nevena Sybeva; Proof 4 is by Augustini Moraru; Proof 5 is by Imad Zak and independently by Rahim Shabazov; Proof 6 is by Robet Kosova; Proof 7 and 8 are by Kunihiko Chikaya; Proofs 9 and 10 are by Sk Rejuan; Proof 11 is by Grégoire Nicollier; Proof 12 is by Michalos Nikolau.

Bibliography: http://www.cut-the-knot.org/

AN INEQUALITY FROM GAZETA MATEMATICA-II- March 2016

Daniel Sitaru, Leonard Giugiuc, Alexander Bogomolny

Problem: Several inequalities with solution by Dan Sitaru and Leo Giugiuc have been just published in Gazeta Matematica (March 2016). The simplest of these is discussed on a separate page. So far it gathered more than ten solutions. I’ll try to partially reproduce some of that experience.

Let 풙,풚,풛, 풕 be pozitive numbers such that 풙ퟐ + 풚ퟐ + 풛ퟐ + 풕ퟐ = ퟏ. Prove that (풙 + 풛)(풚 + 풕) ≤ ퟏ

Proof 1: Define matrix 퐴 =푥 푦 푧 푡푡 푥 푦 푧 . We have

퐴 ⋅ 퐴 =푥 푦 푧 푡푡 푥 푦 푧 ⋅

푥 푡푦 푥푧 푦푡 푧

= 푥 + 푦 + 푧 + 푡 푥푡 + 푥푦 + 푧푦 + 푡푧푥푡 + 푥푦 + 푧푦 + 푡푧 푥 + 푦 + 푧 + 푡

=

= 1 (푥 + 푧)(푦 + 푡)(푥 + 푧)(푦 + 푡) 1

By Cauchy – Binet theorem, det(퐴 ⋅ 퐴 ) ≥ 0. Therefore, [(푥 + 푦)(푦 + 푦)] ≤ 1 ,

or (푥 + 푧)(푦 + 푦) ≤ 1.

Proof 2: (푥 − 푦) + (푦 − 푧) + (푥 − 푡) + (푦 − 푡) ≥ 0 which simplifies to

푥 + 푦 + 푧 푡 − 푥푦 − 푦푧 − 푥푡 − 푦푡 ≥ 0. This is exactly (푥 + 푧)(푦 + 푡) ≤ 1.

Proof 3: By the AM-QM inequality, ≤ = so that 푥 + 푦 + 푧 + 푡 ≤ 2. Further,

by the AM-QM inequality,

(푥 + 푦)(푦 + 푡) ≤푥 + 푦 + 푧 + 푡

2≤

22

= 1.

Page 17: SOCIETATEA DE ȘTIINȚE - ssmrmh.ro · S.S.M.ROMÂNIA - Filiala Mehedinți 2016 6 REVISTA DE MATEMATICĂ MEHEDINȚEANĂ NR. 17 PRINCIPIUL INCLUDERII ȘI EXCLUDERII Daniel Stretcu

S.S.M.ROMÂNIA - Filiala Mehedinți 2016

17 REVISTA DE MATEMATICĂ MEHEDINȚEANĂ NR. 17

Proof 4: From (푥 + 푧) ≤ 2(푥 + 푧 ) and (푦 + 푡) ≤ 2(푦 + 푡 ) we obtain a sequence of inequalities:

(푥 + 푧) (푦+ 푡) ≤ 4(푥 + 푧 )(푦 + 푡 ) ≤ 4푥 + 푧 + 푦 + 푡

2= 4

12

= 4 ⋅14

.

and, therefore, (푥 + 푧)(푦 + 푡) ≤ 1.

Proof 5: Use Lagrange multipliers to prove that: max (푥 + 푧)(푦 + 푡) = 1.

Let 퐽 = (푥 + 푧)(푦 + 푡) + 휆(푥 + 푦 + 푧 + 푡 − 1). Taking = = = = = 0 yields

푦+ 푡 + 2푥휆 = 0, 푥 + 푧 + 2푦휆 = 0,푦 + 푡 + 2푧휆 = 0,푥 + 푧 + 2푡휆 = 0,푥 + 푦 + 푧 + 푡 = 1.

It follows that 푥 = 푧 and 푦 = 푡. So that 2(푥 + 푦 ) = 1 and we need to find max 2푥 ⋅ 2푦. But, since 2푥푦 ≤ 푥 + 푦 , 4푥푦 ≤ 2(푥 + 푧 ) = 1, with the equality only when 푥 = 푦.

To sum up, max (푥 + 푧)(푦 + 푡) = 1,

whith the maximum achieved for 푥 = 푦 = 푧 = 푡 = .

Proof 6: Sk Rejuan used, as before, the Cauchy-Schwarz inequality:

[(푥 + 푧)(푦 + 푡)] = [푥푦 + 푦푧 + 푧푡 + 푡푥] ≤ (푥 + 푦 + 푧 + 푡 )(푦 + 푧 + 푡 + 푥 ) = 1.

Proof 7: Daniel Liuminati put the AM-QM inequality into the fast lane:

(푥 + 푧)(푦 + 푡) ≤(푥 + 푦 + 푧 + 푡)

4≤ 푥 + 푦 + 푧 + 푡 = 1.

Bibliography: http://www.cut-the-knot.org/

MAGIA NUMERELOR COMPLEXE Leonard Giugiuc, Diana Trăilescu

În cele ce urmează autorii își propun să aducă unele completări la capitolul de parte aplicativă a numerelor complexe în geometrie, considerând că pe lângă frumusețea acestui capitol, rolul numerelor complexe în studiul geometriei plane este unul major. Unele dintre teoremele sau observațiile care urmează, lipsesc ori sunt incomplete în majoritatea manualelor și culegerilor școlare. Definiția 1: Spunem că triunghiul 퐴퐵퐶 este pozitiv orientat dacă vârfurile 퐴,퐵 și 퐶 sunt dispuse în sens trigonometric (adică sensul de parcurgere 퐴,퐵,퐶 este cel direct trigonometric). Definiția 2: Spunem că triunghiul 퐴퐵퐶 este negativ orientat dacă vârfurile 퐴,퐵 și 퐶 sunt dispuse în sens invers trigonometric. Definiția 3: Spunem că ∆퐴퐵퐶 și ∆푀푁푃 sunt la fel orientate dacă ambele sunt pozitiv orientate ori ambele sunt negativ orientate. Definiția 4: Spunem că ∆퐴퐵퐶 și ∆푀푁푃 sunt invers orientate dacă unul este pozitiv orientat, iar celălalt este negativ orientat.

Page 18: SOCIETATEA DE ȘTIINȚE - ssmrmh.ro · S.S.M.ROMÂNIA - Filiala Mehedinți 2016 6 REVISTA DE MATEMATICĂ MEHEDINȚEANĂ NR. 17 PRINCIPIUL INCLUDERII ȘI EXCLUDERII Daniel Stretcu

S.S.M.ROMÂNIA - Filiala Mehedinți 2016

18 REVISTA DE MATEMATICĂ MEHEDINȚEANĂ NR. 17

Definiția 5: Spunem că ∆퐴퐵퐶 și ∆푀푁푃 sunt direct asemenea, dacă: i) ∆퐴퐵퐶~∆푀푁푃 și ii) ∆퐴퐵퐶 și ∆푀푁푃 sunt la fel orientate. Definiția 6: Spunem că ∆퐴퐵퐶 și ∆푀푁푃 sunt invers asemenea, dacă: i) ∆퐴퐵퐶~∆푀푁푃 și ii)∆퐴퐵퐶 și ∆푀푁푃 sunt invers orientate. În continuare vom enunța teoremele de bază ale geometriei numerelor complexe. Unele sunt intuitive, deci nu vor fi demonstrate. De asemenea, convenim să utilizăm notația cos 푡 + 푖 sin 푡 = 푒 , pentru orice 푡 real și să identificăm orice punct cu afixul său. Teorema 1: Fie ∆퐴퐵퐶 și 푡 = ∡(퐵퐴퐶). Dacă ∆퐴퐵퐶 este pozitiv orientat, atunci = ⋅ 푒 ; dacă

∆퐴퐵퐶 este negativ orientat atunci = ∙ 푒 .

Consecința 1: Fie ∆퐴퐵퐶. Atunci 푡 = ↔ este număr imaginar.

Consecința 2: Fie punctele distincte 퐴,퐵,퐶 și 퐷. Atunci 퐴퐵 ⊥ 퐶퐷 ↔ ∈ 푖ℝ. Teorema 2 : (condiția de asemănare directă a două triunghiuri): ∆퐴퐵퐶 și ∆푀푁푃 sunt direct asemenea ↔ = .

Demonstrație: Dacă ∆퐴퐵퐶 și ∆푀푁푃 sunt direct asemenea, atunci = ,

∡(퐵퐴퐶) = ∡(푁푀푃) = 푡 și = ∙ 푒± , iar ∙ 푒± . De aici obținem ușor că = .

Reciproc, dacă = atunci = și 퐴푟푔 =

= 퐴푟푔 , deci ∡(퐵퐴퐶) = ∡(푁푀푃), de unde ∆퐴퐵퐶~∆푀푁푃 în mod direct. Teorema 3: (condiția de asemănare inversă a două triunghiuri):

∆퐴퐵퐶 și ∆푀푁푃 sunt invers asemenea ↔ = . Aceste teoreme au drept consecințe unele dintre cele mai spectaculoase rezultate ale geometriei complexe, anume: Teorema 4: Fie triunghiul 퐴퐵퐶 pozitiv orientat. Atunci el este echilateral d.n.d.

퐴 + 퐵휀 + 퐶휀 = 0, unde 휀 = 푒 . Teorema 5: Fie triunghiul 퐴퐵퐶 negativ orientat. Atunci el este echilateral d.n.d.

퐴 + 퐵휀 + 퐶휀 = 0, unde 휀 = 푒 . Consecință: ∆퐴퐵퐶 este echilateral d.n.d. 퐴 + 퐵 + 퐶 = 퐴퐵 + 퐵퐶 + 퐶퐴. În continuare vom vorbi despre coliniaritate și paralelism. Teoreama 6: Fie punctele distincte 퐴,퐵 și 퐶. Cele trei puncte sunt coliniare d.n.d. ∈ ℝ. Consecința: Fie punctele distincte 퐴,퐵,퐶 și 퐷. Atunci dreptele 퐴퐵 și 퐶퐷 sunt paralele (sau coincid) d.n.d. numărul este real. Vom încheia incursiunea teoretică cu condiția de conciclicitate a 4 puncte distincte. Teorema 7: Fie un patrulater convex având vârfurile în 퐴,퐵,퐶 și 퐷. Patrulaterul este inscriptibil d.n.d. biraportul armonic : este număr real. În cele ce urmează vom concretiza acest aparat teoretic prin rezolvarea unor probleme cu un grad foarte ridicat de dificultate, unele dintre ele depășind cu mult nivelul concursurilor naționale și internaționale, dar care au soluții surprinzător de elegante dacă folosim numerele complexe. Aplicația 1 (Problema 4 IMO 2014): Considerăm ∆퐴퐵퐶 în care măsura unghiului din vârful 퐴 este strict mai mare decât măsurile celorlalte două unghiuri ale triunghiului. Fie punctele 푃,푄 ∈ (퐵퐶) astfel ca ∡(푃퐴퐵) = ∡(퐵퐶퐴) și ∡(퐶퐴푄) = ∡(퐴퐵퐶). Fie punctele 푀,푁 pe dreptele 퐴푃 și 퐴푄 respectiv astfel ca 푃 este mijlocul lui 퐴푀 și 푄 este mijlocul lui 퐴푁. Demonstrați că dreptele 퐵푀 și 퐶푁 se intersectează pe cercul circumscris ∆퐴퐵퐶 ∡퐵퐴퐶 < .

Page 19: SOCIETATEA DE ȘTIINȚE - ssmrmh.ro · S.S.M.ROMÂNIA - Filiala Mehedinți 2016 6 REVISTA DE MATEMATICĂ MEHEDINȚEANĂ NR. 17 PRINCIPIUL INCLUDERII ȘI EXCLUDERII Daniel Stretcu

S.S.M.ROMÂNIA - Filiala Mehedinți 2016

19 REVISTA DE MATEMATICĂ MEHEDINȚEANĂ NR. 17

Soluție (Leonard Giugiuc): Evident, △퐴퐵퐶 și △ 푃퐵퐴 sunt invers asemenea, deci

= (1). Analog △퐴퐶퐵 și △푄퐶퐴 sunt invers asemenea, deci = (2).

푃 este mijlocul lui 퐴푀, deci 퐴 +푀 = 2푃, de unde 푀− 푃 = −(퐴 − 푃) și analog 푁 −푄 = −(퐴 − 푄). (3) Din relațiile (1), (2) și (3) obținem că = ⟹△푀푃퐵 ∼△ 퐶푄푁.

În △푀푃퐵,∡(푀푃퐵) = 휋 − ∡퐵퐴퐶.푁표푡∡(푀퐵푃) = 훼. Dar △푀푃퐵 ∼△ 퐶푄푁 ⟹ ∡(푁퐶푄) = ∡퐵퐴퐶 − 훼. În consecință, ∡(퐴퐵푀) + ∡(퐴퐶푁) = 훼 + ∡(퐴퐵퐶) + ∡퐵퐴퐶 − 훼 + ∡(퐵퐶퐴) = 휋. Deci, dacă 퐷 este punctul de intersecție al dreptelor 퐵푀 și 퐶푄, atunci patrulaterul 퐴퐵퐷퐶 are două unghiuri opuse suplementare, de unde obținem ca el este inscriptibl. Aplicația 2 (Generalizarea teoremei lui Bottema de către Leonard Giugiuc): Fie △퐴퐵퐶 fixat în plan și 퐷 un punct variabil situat în exteriorul triunghiului. Considerăm punctele 푇,퐸 și 퐹 pentru care △퐵퐸푇~ △ 퐷퐴푇~ △ 퐷퐹퐶, în configurația:

Dacă 훼,훽 sunt constante, atunci mijlocul lui 퐸퐹 nu depinde de poziția lui 퐷. Demonstrație: Din configurația de mai sus obținem ca triunghiurile sunt direct asemenea, deci

= = ≔ 푢 = constant. Aplicând propoziții derivate rezultă că ( ) ( )( ) ( ) = 푢 ⟹ ( ) ( )

( ) ( ) = 푢 ⟹ 퐸 + 퐹 este constant ⟹ este constant, adică mijlocul lui 퐸퐹 nu depinde de poziția lui 퐷. Acest rezultat a fost supranumit de către matematicianul american Alexander Bogomolny teorema Leo – Bottema. Aplicația 3 ( Dao Thanh Oai, Vietnam): Fie △퐴퐵퐶 și 푃 un punct arbitrar situat pe cercul circumscris triunghiului. Notăm cu 푃 ,푃 și 푃 simetricele punctului 푃 față de 퐵퐶,퐶퐴 și 퐴퐵 respectiv. Notăm cu 퐴 ,퐵 ,퐶 mijloacele segmentelor 퐴푃 ,퐵푃 și respectiv 퐶푃 ; de asemenea, notăm cu 퐴 ,퐵 ,퐶 simetricele punctelor 푃 ,푃 și 푃 față de mijloacele laturilor 퐵퐶,퐶퐴,퐴퐵. a) Demonstrați că punctele 퐴 ,퐵 ,퐶 se află pe cercul Euler al △ 퐴퐵퐶. b) Demostrați că 퐴 ,퐵 ,퐶 sunt pe cercul circumscris △퐴퐵퐶. c) Demonstrați că triunghiurile 퐴퐵퐶,퐴 퐵 퐶 și 퐴 퐵 퐶 sunt asemenea. Soluție (Diana Trăilescu): Alegem pe cercul unitar punctele 퐴,퐵,퐶 având afixele 푎, 푏, 푐, 1 respectiv.

Page 20: SOCIETATEA DE ȘTIINȚE - ssmrmh.ro · S.S.M.ROMÂNIA - Filiala Mehedinți 2016 6 REVISTA DE MATEMATICĂ MEHEDINȚEANĂ NR. 17 PRINCIPIUL INCLUDERII ȘI EXCLUDERII Daniel Stretcu

S.S.M.ROMÂNIA - Filiala Mehedinți 2016

20 REVISTA DE MATEMATICĂ MEHEDINȚEANĂ NR. 17

△ 푃퐴퐵 ≡△ 푃 퐴퐵 și sunt invers orientate ⟹ = ⟹ = ⟹

푃 = 푎 + 푏 − 푎푏; analog 푃 = 푎 + 푐 − 푎푐 și 푃 = 푏 + 푐 − 푏푐. De aici ne rezultă că 퐴 = ,퐵 = și 퐶 = . Patrulaterul 퐴 퐵푃 퐶 este paralelogram ⟹ 퐴 + 푃 = 푏 + 푐 ⟹ 퐴 = 푏푐. Analog 퐵 = 푎푐 și 퐶 = 푎푏. a) Știm că cercul Euler are centrul în mijlocul lui 푂퐻 și raza egală cu , cu 퐻 ortocentrul △ 퐴퐵퐶. Via

Sylvester, 퐻 = 푎 + 푏 + 푐 ⟹ 퐸: 푧 − = . Este evident că punctele 퐴 ,퐵 ,퐶 se află pe cercul lui Euler. b) Numerele 푏푐,푎푐 și 푎푏 cu modulul unitar, ceea ce demonstrează cerința.

c) = ; = = și analog = . Deci cele trei triunghiuri sunt într-

adevăr asemenea. Aplicația 4 (autor Francisco Javier Garcia, Spania): Fie △ 퐴퐵퐶 ascuțitunghic și un punct 푆 în interiorul său. Proiecțiile lui 푆 pe dreptele 퐵퐶,퐶퐴 și 퐴퐵 respectiv sunt situate pe laturile triunghiului și le notăm cu 푀,푁 și 푃. Considerăm punctul 퐷 astfel încât 퐵퐶 separă 퐴 și 퐷, iar △푀푁푃~ △ 퐷퐵퐶. a) Să se arate că 퐴퐷 ⊥ 푁푃. b) Demonstrați că [퐴푃퐷푁] = [퐴퐵퐶]. Soluție (Leonard Giugiuc): Fie 푏 = cot퐵 și 푐 = cot퐶. Alegem 퐴 = 푖,퐵 = −푏 și 퐶 = 푐; evident ∃푚 ∈ (−푏, 푐) și 푛,푝 ∈ (0,1) pentru care 푀 = 푚,푁 = 푛푐 + (1 − 푛)푖 și 푃 = −푝푏 + (1− 푝)푖. Deci ecuațiile celor trei perpendiculare în 푀,푁 și 푃 pe laturile 퐵퐶,퐶퐴 și 퐴퐵 respectiv sunt: 푥 = 푚,−푐푥 + 푦 = 1− 푛 − 푛푐 și 푏푥 + 푦 = 1 − 푝 − 푝푏 . Cum ele sunt concurente, atunci

1 0 푚푏 1 1 − 푝 − 푝푏−푐 1 1 − 푛 − 푛푐

= 0 ⟹푚 = . Triunghiurile 푀푁푃 și 퐷퐵퐶 sunt invers

asemenea ⟹ = ⟹ 퐷 = ( )( ) . De aici,

퐷 − 푖 = ( )( ) = ( ) ⟹ = ( )

| | ∈ 푖ℝ ⟹ 퐴퐷 ⊥ 푁푃. Acum aflăm cele două arii.

2[퐴퐵퐶] = 푏 + 푐푠푖2[퐴푃퐷푁] = 퐴퐷 ∙ 푃푁 = ( ) ∙ 푃푁 = 푏 + 푐. Deci [퐴퐵퐶] = [퐴푃퐷푁]. În încheiere vom propune cititorilor spre rezolvare câteva aplicații. Problema 1 (Culeasă de Miguel Ochoa Sanchez, Peru): Fie ∆퐴퐵퐶 în care ∡(퐴퐵퐶) = 2∡(퐴퐶퐵). Notăm cu 푀 mijlocul lui 퐵퐶. Bisectoarea 퐴퐶퐵 intersectează dreapta 퐴푀 în punctul 퐷. Arătați că ∡(퐶퐷푀) ≤ 45°. Problema 2 (Autor Leonard Giugiuc, BMO 2014 Shortlist): Fie ∆퐴퐵퐶 cu 퐴퐵 = 퐴퐶. Considerăm punctele 퐷 și 퐸 pe dreapta 퐵퐶 astfel încât 퐷 ∈ (퐵퐸) 퐸 ∈ (퐷퐶) și ∡(퐷퐴퐸) = ∡( ). Demonstrați că ∃∆푀푁푃 cu 푀푁 = 퐵퐷,푀푃 = 퐶퐸 și 푁푃 = 퐷퐸. Să se determine ∡(푁푀푃) + ∡(퐵퐴퐶). Notă: Cele două soluții sintetice ale acestei probleme i se datorează lui Cătălin Gherghe, Universitatea București. Problema 3 (Dao Thanh Oai, Vietnam și Leonard Giugiuc): Fie ∆퐴퐵퐶 având măsurile unghiurilor mai mici decât 120° și 퐼 centrul cercului înscris triunghiului. Considerăm punctele 퐷 ∈ (퐴퐼),퐸 ∈ (퐵퐼) și 퐹 ∈ (퐶퐼) pentru care 퐴퐷 = 푝 − 푎 −

√cos ,퐵퐸 = 푝 − 푏 −

√cos și 퐶퐹 = 푝 − 푐 −

√cos , unde notațiile sunt

cele uzuale. Demonstrați că ∆퐷퐸퐹 este echilateral. Bibliografie:

Page 21: SOCIETATEA DE ȘTIINȚE - ssmrmh.ro · S.S.M.ROMÂNIA - Filiala Mehedinți 2016 6 REVISTA DE MATEMATICĂ MEHEDINȚEANĂ NR. 17 PRINCIPIUL INCLUDERII ȘI EXCLUDERII Daniel Stretcu

S.S.M.ROMÂNIA - Filiala Mehedinți 2016

21 REVISTA DE MATEMATICĂ MEHEDINȚEANĂ NR. 17

http://www.cut-the-knot.org/ https://www.facebook.com/groups/ShMaId/ https://www.facebook.com/groups/perugeometrico1/

A SPECIAL INEQUALITY FOR TRIANGLE

Mihaly Bencze

Abstract: in this paper we present a special inequality for triangle Main results: Theorem: If 푢 , 푣 ∈ (0,∞); 푛 ∈ ℕ;푛 ≥ 2;푘 ∈ 1,푛 then:

푢푣 푣푣

+ 푢푣 푣푣

+ 푢푣 푣푣

≥ 3 푢 푢 푢

Proof:

푢푣 푣푣

≥ 3 푢푣 푣푣

=

= 3 푢 푢 푢 because if 푀 = ∏ then: lg푀 = ∑ lg lg =

= (lg 푣 − lg푣 )(lg 푣 + lg푣 − lg푣 ) = 0

therefore 푀 = 1 Corrolaries: In any triangle 퐴퐵퐶:

1. ∑푟 ≥ 3√푆 푟, 2. ∑(푆 − 푎) ≥ 3√푆푟 , 3. ∑ℎ ≥ 3

4. ∑ ctg ≥ 3 , 5. ∑ tg ≥ 3 ,

6. ∑ sin ≥ 3 , 7. ∑ cos ≥ 3

Reference: Octogon Mathematical Magazine (1993 – 2015)

SOLUTIONS AND GENERALIZATIONS OF SOME PROBLEMS FROM CRUX MATHEMATICORUM

D.M. Bătinețu – Giurgiu, Neculai Stanciu

3948. Proposed by George Apostolopoulos. Let 풂ퟏ,풂ퟐ, … , 풂풏 be real numbers such that 풂ퟏ > 풂ퟐ > ⋯ > 풂풏. Prove that

ퟏ풂ퟏ − 풂ퟐ

+ퟏ

풂ퟐ − 풂ퟑ+ ⋯+

ퟏ풂풏 ퟏ − 풂풏

+ 풂ퟏ −풂풏 ≥ ퟐ(풏− ퟏ)

When does the equality hold? Solution: By H. Bergström’s inequality we have

1푎 − 푎

(1 + 1 +⋯+ 1)( )

∑ (푎 − 푎 ) =(푛 − 1)푎 − 푎

,

so

Page 22: SOCIETATEA DE ȘTIINȚE - ssmrmh.ro · S.S.M.ROMÂNIA - Filiala Mehedinți 2016 6 REVISTA DE MATEMATICĂ MEHEDINȚEANĂ NR. 17 PRINCIPIUL INCLUDERII ȘI EXCLUDERII Daniel Stretcu

S.S.M.ROMÂNIA - Filiala Mehedinți 2016

22 REVISTA DE MATEMATICĂ MEHEDINȚEANĂ NR. 17

푎 − 푎 +1

푎 − 푎≥

(푛 − 1)푎 − 푎

+ (푎 − 푎 ) ≥⏞ 2(푛 − 1)푎 − 푎

(푎 − 푎 ) =

= 2(푛 − 1). The equality holds if and only if

(푛 − 1)푎 − 푎

= 푎 − 푎 ⇔ (푛 − 1) = (푎 − 푎 ) ⇔ 푎 = 푎 − 푛 + 1.

Remark. The problem from above was proposed at St. Petersburg Math Olympiad, 1999, i.e. Let 푥 > 푥 > 푥 > ⋯ > 푥 be real numbers. Prove that

푥 +1

푥 − 푥+

1푥 − 푥

+⋯+1

푥 − 푥≥ 푥 + 2푛.

This inequality was appeared also in 2001 at Romanian Mathematical Gazette, Vol. 106, no. 9 – 10 , p. 325. 3949. Proposed by Arkady Alt. For any positive real 풂 and 풃, find :

퐥퐢퐦풏→

⎜⎛

(풏+ ퟏ)풂

ퟏ풏 ퟏ + 풃

ퟏ풏 ퟏ

풏 ퟏ

− 풏풂ퟏ풏 + 풃

ퟏ풏

⎟⎞

.

Solution. If 푥 ∈ 푅∗ = (0,∞), then :

lim→푐 = 1and lim

푐 − 1푥

= lim→

푒 − 1푥 ln푐

ln푐 = 1 ⋅ ln 푐 = ln 푐.

Let 푎, 푏 ∈ 푅∗ ,퐴 (푎, 푏) = and 퐵 (푎, 푏) = (푛 + 1)퐴 (푎, 푏) − 푛퐴 (푎, 푏),∀푛 ∈ 푁∗.

Evidentely 퐴 (푎,푎) = 푎 and 퐵 (푎,푎) = 푎,∀푛 ∈ 푁∗. In the follow we consider

푛 ≥ 2,퐴(푎, 푏) = √ √ .We have:

lim→

퐴 (푎, 푏) = lim→

√푎 + √푏2

= lim→

1 +√푎 + √푏 − 2

2√ √

√ √

=

= 푒 → √ √ = 푒 ( ) = 푒 √ = √푎푏,(1) Let 푢 = ⋅ ( , )

( , ) ,∀푛 ≥ 2, then:

lim→

푢 = 1 ⋅√푎푏√푎푏

= 1 ⇒ lim→

푢 − 1ln푢

= 1;

and like in (1) we deduce that:

lim→

푢 = lim→

푛 + 1푛

lim→

퐴 (푎, 푏)퐴 (푎, 푏) =

= 푒 ⋅ √푎푏 ⋅ 푒 →( ) ( ),(2)

Where 푣 = √푎 − √푎,푤 = √푏 − √푏,∀푛 ≥ 2. We have

lim→

푛(푛 + 1)푣 = lim→

√푎 ⋅ 푛(푛 + 1) 1 − 푎 = 1 ⋅ lim→

푛(푛 + 1) 1 − 푎 =

= − lim→

푎 − 1

( )

= − ln푎

and similar we obtain:

Page 23: SOCIETATEA DE ȘTIINȚE - ssmrmh.ro · S.S.M.ROMÂNIA - Filiala Mehedinți 2016 6 REVISTA DE MATEMATICĂ MEHEDINȚEANĂ NR. 17 PRINCIPIUL INCLUDERII ȘI EXCLUDERII Daniel Stretcu

S.S.M.ROMÂNIA - Filiala Mehedinți 2016

23 REVISTA DE MATEMATICĂ MEHEDINȚEANĂ NR. 17

lim→

푛(푛 + 1)푤 = − ln푏 ,(3) By (2) and (3) we obtain that

lim→

푢 = 푒 ⋅ √푎푏 ⋅ 푒 ( ) = 푒 ⋅ √푎푏 ⋅ 푒 = 푒 ⋅ √푎푏 ⋅ 푒 √ = 푒 ⋅ √푎푏 ⋅1√푎푏

= 푒.

Therefore, 퐵 (푎, 푏) = 푛퐴 (푎, 푏)(푢 − 1) = 푛퐴 (푎, 푏) ⋅ ⋅ ln푢 = 퐴 (푎, 푏) ⋅ ,∀푛 ≥ 2

Hence, lim→

퐵 (푎, 푏) = √푎푏 ⋅ 1 ⋅ ln푒 = √푎푏, and we are done! 3974. Proposed by George Apostolopoulos. Let 풂,풃 and 풄 be positive real numbers such that 풂 + 풃 + 풄 = ퟑ. Prove that

풂풃

+ퟏ풂

+풃풄

+ퟏ풃

+풄풂

+ퟏ풄≥ ퟑ√ퟐ.

Generalization: We shall prove more general inequality, i.e.If 풎 ∈ (ퟏ,∞),풏 ∈ 푵∗ − {ퟏ}, 풂풌 ∈ 푹∗ ,풌 = ퟏ,풏 and

풂풌

풌 ퟏ

= 풂 ∈ 푹∗ ,

then

풂풌풂풌 ퟏ

+ퟏ풂풌

ퟏ풎

풌 ퟏ

≥ퟐ풂

ퟏퟐ풎풏ퟏ

ퟏퟐ풎,

where 풂풏 ퟏ = 풂ퟏ. Proof. By AM-GM inequality we have

푈 =푎푎

+1푎

≥ 2푎푎

⋅1푎

= 21

= 21

, (1)

Since 푎 ∈ 푅∗ , there exists 푥 ∈ 푅∗ such that 푎 = 푥 , 푘 = 1,푛; so

푎 = 푎 = 푥 ,(2)

Since the function 푓:푅∗ → 푅∗ , 푓(푥) = 푥 is convex on 푅∗ by Jensen’s inequality yields that

푎 = 푥 = 푓(푥 ) ≥ 푛푓1푛

푥 = 푛1푛

푥 =1

푛푥

⇔ 푎푛 ≥ 푥 ⇔ 푥 ≤ 푎 푛 ,(3)

By (1) and Bergström’s inequality we have

푈 ≥ 21

≥⏞ö

2 ⋅푛

∑ 푥,(4)

By (3) and (4) we obtain that

푈 ≥ 2 ⋅푛

푎 푛=

2푎

푛 , 푞. 푒.푑.

Observations. If 푎 = 푛 the inequality becomes

Page 24: SOCIETATEA DE ȘTIINȚE - ssmrmh.ro · S.S.M.ROMÂNIA - Filiala Mehedinți 2016 6 REVISTA DE MATEMATICĂ MEHEDINȚEANĂ NR. 17 PRINCIPIUL INCLUDERII ȘI EXCLUDERII Daniel Stretcu

S.S.M.ROMÂNIA - Filiala Mehedinți 2016

24 REVISTA DE MATEMATICĂ MEHEDINȚEANĂ NR. 17

푎푎

+1푎

≥2푚

푛 = 2 푛,(5)

If in (5) we take 푛 = 3,푚 = 2 we deduce

푎푎

+1푎

≥ 2 ⋅ 3,

where if we put 푎 = 푎, 푎 = 푏,푎 = 푐 we obtain

+ + + + + ≥ √2 ⋅ 3, i.e. the problem 3974 from CRUX.

3990. Proposed by Ángel Plaza: Let 풂ퟏ,풂ퟐ, … , 풂풏 be positive real numbers such that 풂ퟏ > 풂ퟐ > ⋯ > 풂풏. Prove that

(풂ퟏ − 풂풏)ퟏ

풂ퟏ − 풂ퟐ+

ퟏ풂ퟐ − 풂ퟑ

+ ⋯+ퟏ

풂풏 ퟏ − 풂풏≥ (풏 − ퟏ)ퟐ.

When does equlity hold? Generalization. If 풎 ∈ 푵∗ − {ퟏ},풂, 풄, 풔 ∈ 푹 ,풃,풅,풙풌,풚풌 ∈ 푹∗ ,풌 = ퟏ,풎, 풓 ∈ [ퟏ,∞) and

푿풎 = 풙풌

풌 ퟏ

,풀풎 = 풚풌

풌 ퟏ

,

then

(풂푿풎 + 풃풙풌)풓풎

풌 ퟏ

⋅ퟏ

(풄풀풎 + 풅풚풌)풔

풌 ퟏ

≥(풂풎+ 풃)풓(풄풎 + 풅)풔 ⋅

푿풎풓

풀풎풔⋅ 풎풔 풓 ퟐ,(ퟏ)

Proof: We have:

푊 = (푎푋 + 푏푥 ) ⋅1

(푐푌 + 푑푦 ) ≥⏞.

≥(∑ (푎푋 + 푏푥 ))

(1 + 1 + 1 + ⋯+ 1)

1(푐푌 + 푑푦 ) =

=(푎푚푋 + 푏푋 )

푚1

(푐푌 + 푑푦 ) =(푎푚 + 푏) 푋

푚⋅

1(푐푌 + 푑푌 ) ≥⏞

≥⏞(푎푚 + 푏) 푋

푚⋅

(1 + 1 +⋯+ 1)

(∑ (푐푌 + 푑푦 )) =(푎푚 + 푏) 푋

푚⋅

푚(푐푚푌 + 푑푌 ) =

= ( )( ) ⋅ ⋅ 푚 , q.e.d.

1. If we take 푟 = 푠, then (1) becomes:

(푎푋 + 푏푥 ) ⋅1

(푐푌 + 푑푦 ) ≥(푎푚 + 푏)(푐푚 + 푑) ⋅

푋푌

⋅ 푚 ,(ퟐ)

2. If 푎 = 푐 = 0, 푏 = 푑, then (2) becomes:

푥 ⋅1푦

≥푋푌

⋅ 푚 ,(ퟑ)

3. If 푥 = 푦 ,푘 = 1,푚, then (3) becomes:

푥 ⋅1푥

≥ 푚 ,(ퟒ)

Page 25: SOCIETATEA DE ȘTIINȚE - ssmrmh.ro · S.S.M.ROMÂNIA - Filiala Mehedinți 2016 6 REVISTA DE MATEMATICĂ MEHEDINȚEANĂ NR. 17 PRINCIPIUL INCLUDERII ȘI EXCLUDERII Daniel Stretcu

S.S.M.ROMÂNIA - Filiala Mehedinți 2016

25 REVISTA DE MATEMATICĂ MEHEDINȚEANĂ NR. 17

4. If 푟 = 1, then (4) becomes:

푥 ⋅1푥

≥ 푚 ,(ퟓ)

Let 푛 ∈ 푁∗ − {1},푎 ∈ 푅∗ , 푘 = 1,푛, such that 푎 > 푎 > ⋯ > 푎 it is obvious that 푎 > 푎 ⇔ 푎 − 푎 > 0,∀푘 = 1,푛 − 1.

Therefore, if we take 푚 = 푛 − 1, 푥 = 푎 − 푎 , 푘 = 1,푚, we obtain

(푎 − 푎 ) ⋅1

푎 − 푎≥ (푛 − 1) ,

i.e. the problem 3990 for CRUX Mathematicorum. The equality occurs if and only if 푥 = 푥 = ⋯ = 푥 = 푡 ∈ 푅∗ ⇔ 푎 − 푎 = 푎 − 푎 = ⋯ = 푎 − 푎 = 푡 ⇔

⇔ 푎 = 푎 − 푡,푎 = 푎 − 2푡… ,푎 = 푎 − (푛 − 2)푡, i.e. 푎 ,푎 , … ,푎 are the terms of increasing arithmetic progression.

AN APPLICATION OF SCHUR’S INEQUALITY

Daniel Sitaru,Leonard Giugiuc

Dan Sitaru has kindly communicated a problem he invented and solved in collaboration with Leo Giugiuc. Theirs is Proof. 1. Proof 2 is by Imad Zak. Prove that for 풙,풚, 풛 > 0 such that 풙풚풛 = ퟏ, the following inequality holds:

풙ퟒ + 풚ퟑ + 풛 ≥풙ퟐ + 풚ퟐ

풛+ ퟑ.

Proof 1: We use Schur's inequality twice: With 푟 = 1 in the form ∑푥 + 3푥푦푧 ≥ ∑푥푦(푥 + 푦) and, with 푟 = 2 in the form ∑푥 + 푥푦푧∑ 푥 ≥ ∑푥푦(푥 + 푦 ). Since 푥푦푧 = 1 the two can be rewritten as

푥 + 3 ≥푥 + 푦푧

,

푥 + 푥 ≥푥 + 푦

푧.

Adding up,

(푥 + 푥 + 푥) + 3 ≥푥 + 푦

푧+푥푧

+푧푥

+푦푥

+푥푦

+푦푧

+푧푦

.

But ∑ + ≥ 2 + 2 + 2 = 6. It follows that

(푥 + 푥 + 푥) ≥푥 + 푦

푧+ 6− 3

or

(푥 + 푦 + 푧) ≥푥 + 푦

푧+ 3

Proof 2: We start with Schur's inequality:

푥 + 푥푦푧 푥 = 푥 + 푥 = (푥 + 푥) ≥

≥ 푥푦(푥 + 푦 ) =푥 + 푦

푧.

Further, by the AM-GM inequality,∑푥 ≥ 3푥푦푧 = 3.

Page 26: SOCIETATEA DE ȘTIINȚE - ssmrmh.ro · S.S.M.ROMÂNIA - Filiala Mehedinți 2016 6 REVISTA DE MATEMATICĂ MEHEDINȚEANĂ NR. 17 PRINCIPIUL INCLUDERII ȘI EXCLUDERII Daniel Stretcu

S.S.M.ROMÂNIA - Filiala Mehedinți 2016

26 REVISTA DE MATEMATICĂ MEHEDINȚEANĂ NR. 17

Adding this to the preavious inequality yields the required result. Equlity holds for 푥 = 푦 = 푧 = 1. Bibliography: http://www.cut-the-knot.org/

SOME NEW INEQUALITIES

Mihaly Bencze

ABSTRACT: in this paper we present some new geometrical inequalities. Keywords: geometrical inequalities Main result: If 풙풌 > 0;푘 ∈ ퟏ,풏;풏 ∈ ℕ∗ then:

풆 풕ퟐ풙풌

풌 ퟏ

풅풕 ≤ 풏퐚퐫퐜퐭퐠ퟏ풏

풙풌풏

풌 ퟏ

Proof:

Let be 푓(푥) = arctg푥 − ∫ 푒 푑푡, 푓 (푥) = ( )( )

≥ 0; (∀)푥 > 0 therefore 푓(푥) ≥ 푓(0) or

∫ 푒 푑푡 ≤ arctg푥 . Function 푔(푥) = arctg푥 its concave because 푔 (푥) = ( ) < 0; (∀)푥 > 0 and from Jensen’s inequality:

푒 푑푡 ≤ arctg 푥 ≤ 푛 arctg1푛

Corollary 1: 1.

풆 풕ퟐ풌

풌 ퟏ

풅풕 ≤ 풏퐚퐫퐜퐭퐠풏+ ퟏퟐ

2.

풆 풕ퟐ풌ퟐ

풌 ퟏ

풅풕 ≤ 풏퐚퐫퐜퐭퐠(풏+ ퟏ)(ퟐ풏+ ퟏ)

3.

풆 풕ퟐ풌ퟑ

풌 ퟏ

풅풕 ≤ 풏퐚퐫퐜퐭퐠풏(풏+ ퟏ)ퟐ

4.

풆 풕ퟐ

ퟏ풌(풌 ퟏ)

풌 ퟏ

풅풕 ≤ 풏퐚퐫퐜퐭퐠ퟏ

풏+ ퟏ

Corollary 2: 1. If 풂풌 > 0; 푘 ∈ ퟏ,풏;풏 ∈ ℕ∗ is an arithmetical progression then:

풆 풕ퟐ풂풌

풌 ퟏ

풅풕 ≤ 풏퐚퐫퐜퐭퐠풂ퟏ + 풂풏

2. If 풃풌 > 0;푘 ∈ ퟏ,풏;풏 ∈ ℕ∗ is a geometrical progression with ration 풒 > 0 then:

Page 27: SOCIETATEA DE ȘTIINȚE - ssmrmh.ro · S.S.M.ROMÂNIA - Filiala Mehedinți 2016 6 REVISTA DE MATEMATICĂ MEHEDINȚEANĂ NR. 17 PRINCIPIUL INCLUDERII ȘI EXCLUDERII Daniel Stretcu

S.S.M.ROMÂNIA - Filiala Mehedinți 2016

27 REVISTA DE MATEMATICĂ MEHEDINȚEANĂ NR. 17

풆 풕ퟐ풃풌

풌 ퟏ

풅풕 ≤ 풏 퐚퐫퐜퐭퐠풃ퟏ(풒풏 − ퟏ)풏(풒− ퟏ)

Corollary 3: Let 푨ퟏ,푨ퟐ, … ,푨풏 be a convex polygone. Then its valid the relationship:

풆 풕ퟐ푨풌

풌 ퟏ

풅풕 ≤ 풏퐚퐫퐜퐭퐠(풏 − ퟐ)흅

Corollary 4: In all triangles 푨푩푪 next relationships holds: 1.

풆 풕ퟐ풂

풅풕 ≤ ퟑ퐚퐫퐜퐭퐠ퟐ푺ퟑ

2.

풆 풕ퟐ푺 풂

풅풕 ≤ ퟑ퐚퐫퐜퐭퐠푺ퟑ

3.

풆 풕ퟐ풂풃

풅풕 ≤ ퟑ퐚퐫퐜퐭퐠푺ퟐ + 풓ퟐ + ퟒ푹풓

4.

풆 풕ퟐ(푺 풂)(푺 풃)

풅풕 ≤ ퟑ퐚퐫퐜퐭퐠풓(ퟒ푹+ 풓)

5.

풆 풕ퟐ

ퟏ풂 풃

풅풕 ≤ ퟑ퐚퐫퐜퐭퐠ퟓ푺ퟐ + 풓ퟐ + ퟒ푹풓

ퟔ푺(푺ퟐ + 풓ퟐ + ퟐ푹풓)

6.

풆 풕ퟐ

ퟏ(풂 풃)(풃 풄)

풅풕 ≤ ퟑ퐚퐫퐜퐭퐠ퟐ

ퟑ(푺ퟐ + 풓ퟐ + ퟐ푹풓)

7.

풆 풕ퟐ

풂풃 풄

풅풕 ≤ ퟑ퐚퐫퐜퐭퐠ퟐ 푺ퟐ − 풓ퟐ − 푹풓ퟑ(푺ퟐ + 풓ퟐ + ퟐ푹풓)

8.

풆 풕ퟐ

풂(푺 풃)(푺 풄)

풅풕 ≤ ퟑ퐚퐫퐜퐭퐠ퟐ(ퟒ푹 + 풓)

ퟑ푺풓

9.

Page 28: SOCIETATEA DE ȘTIINȚE - ssmrmh.ro · S.S.M.ROMÂNIA - Filiala Mehedinți 2016 6 REVISTA DE MATEMATICĂ MEHEDINȚEANĂ NR. 17 PRINCIPIUL INCLUDERII ȘI EXCLUDERII Daniel Stretcu

S.S.M.ROMÂNIA - Filiala Mehedinți 2016

28 REVISTA DE MATEMATICĂ MEHEDINȚEANĂ NR. 17

풆 풕ퟐ풓풂

풅풕 ≤ ퟑ퐚퐫퐜퐭퐠ퟒ푹+ 풓ퟑ

10.

풆 풕ퟐ풓풂풓풃

풅풕 ≤ ퟑ퐚퐫퐜퐭퐠푺ퟐ

11.

풆 풕ퟐ

ퟏ풓풂풓풃

풅풕 ≤ ퟑ퐚퐫퐜퐭퐠ퟒ푹 + 풓ퟑ푺ퟐ풓

12.

풆 풕ퟐ

ퟏ풓풂 풓풃

풅풕 ≤ ퟑ퐚퐫퐜퐭퐠(ퟒ푹 + 풓)ퟐ + 푺ퟐ

ퟏퟐ푺ퟐ푹

13.

풆 풕ퟐ

ퟏ풓풂 풓풃 풓풃 풓풄

풅풕 ≤ ퟑ퐚퐫퐜퐭퐠ퟒ푹+ 풓ퟔ푺ퟐ풓

14.

풆 풕ퟐ푨

풅풕 ≤ ퟑ퐚퐫퐜퐭퐠흅ퟑ

15.

풆 풕ퟐ

퐬퐢퐧ퟐ푨ퟐ

풅풕 ≤ ퟑ퐚퐫퐜퐭퐠ퟐ푹− 풓ퟔ푹

16.

풆 풕ퟐ

퐬퐢퐧ퟐ푨ퟐ 퐬퐢퐧ퟐ푩ퟐ

풅풕 ≤ ퟑ퐚퐫퐜퐭퐠푺ퟐ + 풓ퟐ − ퟖ푹풓

ퟒퟖ푹ퟐ

17.

풆 풕ퟐ

퐬퐢퐧ퟐ푨ퟐ

풅풕 ≤ ퟑ퐚퐫퐜퐭퐠ퟖ푹ퟐ + 풓ퟐ − 푺ퟐ

ퟐퟒ푹ퟐ

18.

풆 풕ퟐ

퐜퐨퐬ퟐ푨ퟐ

풅풕 ≤ ퟑ 퐚퐫퐜퐭퐠ퟒ푹+ 풓ퟔ푹

19.

풆 풕ퟐ

퐜퐨퐬ퟐ푨ퟐ 퐜퐨퐬ퟐ푩ퟐ

풅풕 ≤ ퟑ퐚퐫퐜퐭퐠푺ퟐ + (ퟒ푹 + 풓)ퟐ

ퟒퟖ푹ퟐ

20.

Page 29: SOCIETATEA DE ȘTIINȚE - ssmrmh.ro · S.S.M.ROMÂNIA - Filiala Mehedinți 2016 6 REVISTA DE MATEMATICĂ MEHEDINȚEANĂ NR. 17 PRINCIPIUL INCLUDERII ȘI EXCLUDERII Daniel Stretcu

S.S.M.ROMÂNIA - Filiala Mehedinți 2016

29 REVISTA DE MATEMATICĂ MEHEDINȚEANĂ NR. 17

풆 풕ퟐ

퐜퐨퐬ퟒ푨ퟐ

풅풕 ≤ ퟑ퐚퐫퐜퐭퐠(ퟒ푹 + 풓)ퟐ − 푺ퟐ

ퟐퟒ푹ퟐ

Reference: Octogon Mathematical Magazine (1993 – 2015)

AN ANGLE INEQUALITY IN SIMPLE POLYGONS

Daniel Sitaru

Let 푨ퟏ,푨ퟐ …푨풏,풏 ≥ ퟑ, be a simple (without self intersections) polygon,

with the inner angles 휶ퟏ,휶ퟐ, … Then :

(ퟏ + 풂풌)풏

풌 ퟏ

≤흅풌(풏 − ퟐ)풌

풌!

풌 ퟏ

.

Examples: In a triangle with angles 휶,휷,휸, (ퟏ + 휶)(ퟏ + 휷)(ퟏ + 휸) ≤ ퟏ + 흅+ 흅ퟐ

ퟐ+ 흅ퟑ

ퟔ.

In a simple quadrilateral with angles 휶,휷,휸,휹:

(ퟏ+ 휶)(ퟏ+ 휷)(ퟏ+ 휸)(ퟏ+ 휹) ≤ ퟏ + ퟐ흅+ ퟐ흅ퟐ +ퟒ흅ퟑ

ퟑ+ퟐ흅ퟒ

ퟑ.

Proof: Define:

푺 = 푨풌풏

풌 ퟏ

(which is known to be (풏 − ퟐ)흅). By the Geometric Mean - Arithmetic Mean inequality,

(ퟏ + 풂풌)풏

풌 ퟏ

≤ ퟏ +푺풏

.

Implying the Binomial expansion:

ퟏ +푺풏

=푺풌

풏풌

풌 ퟏ

풏풌 =

푺풌

풌!

풌 ퟏ

=[흅(풏 − ퟐ)]풌

풌!

풌 ퟏ

.

Bibliography: http://www.cut-the-knot.org/

AN INEQUALITY VIA YOUNG’S THEOREM

Daniel Sitaru

For 퐚,퐛, 퐜 > 0 the following inequality holds:

퐚퐛 + 퐛퐜 + 퐜퐚 ≤ 퐥퐧 퐚퐚 ∙ 퐛퐛 ∙ 퐜퐜 +퐞퐚 + 퐞퐛 + 퐞퐜

퐞.

Page 30: SOCIETATEA DE ȘTIINȚE - ssmrmh.ro · S.S.M.ROMÂNIA - Filiala Mehedinți 2016 6 REVISTA DE MATEMATICĂ MEHEDINȚEANĂ NR. 17 PRINCIPIUL INCLUDERII ȘI EXCLUDERII Daniel Stretcu

S.S.M.ROMÂNIA - Filiala Mehedinți 2016

30 REVISTA DE MATEMATICĂ MEHEDINȚEANĂ NR. 17

The equality is reached for 퐚 = 퐛 = 퐜 = ퟏ. The above inequality follows from Young’s theorem:

Let 풇 be a continuous strictly increasing function on an interval [ퟎ,풄] and 풇 ퟏ it inverse.

For 풂 ∈ [풂,풄], and 풃 ∈ [ퟎ,풇(풄)],

풂풃 ≤ 풇(풙)풂

풅풙+ 풇 ퟏ(풙)풃

풅풙,

with the equality is only achieved if 풃 = 풇(풂).Proof of Sitaru’s inequality (Alexander Bogomolny):

Choose 풇(풙) = 퐥퐧(풙 + ퟏ) so that 풇 ퟏ(풙) = 풆풙 − ퟏ. Obviously such 풇 satisfies the conditions of Young’s inequality for any 풄 > 0.Now by elementary calculus:

풇 ퟏ(풙)풃

풅풙 = 퐥퐧(풙 + ퟏ)풅풙 = (풂 + ퟏ)풂

퐥퐧(풂 + ퟏ)−풂,

and also:

풇 ퟏ(풙)풃

= (풆풙 − ퟏ)풃

풅풙 = 풆풃 − ퟏ − 풃.

By Young’s theorem it follows that: 풂풃 ≤ (풂 + ퟏ) 퐥퐧(풂 + ퟏ) −풂 + 풆풃 − ퟏ − 풃.In other words,

(풂 + ퟏ)(풃 + ퟏ) ≤ (풂 + ퟏ) 퐥퐧(풂 + ퟏ) + 풆풃.

By replacing 풂 + ퟏ with 풂 and 풃 + ퟏ with 풃 we obtain:

풂풃 ≤ 풂 퐥퐧풂+ 풆풃 ퟏ = 퐥퐧 풂풂 + 풆풃

풆 .

Similarly, 풃풄 ≤ 퐥퐧풃풃 + 풆풄

풆 and 풄풂 ≤ 퐥퐧 풄풄 + 풆풂

풆.Adding the three proves the desired inequality.

Clearly, the statement and the proof extend to any number of parameters:

For positive 풙ퟏ,풙ퟐ, … ,풙풏,풏 ≥ ퟑ,풙풏 ퟏ = 풙ퟏ,

풙풌풙풌 ퟏ

풌 ퟏ

≤ 퐥퐧 풙풌풙풌

풌 ퟏ

+∑ 풆풙풌풏풌 ퟏ

풆.

Bibliography: http://www.cut-the-knot.org/

SYSTEM IN PERMUTATIONS

Daniel Sitaru

Solve in permutations:

풙풚풛ퟐ = ퟏ ퟐ ퟑ ퟒ ퟓퟏ ퟐ ퟒ ퟑ ퟓ 풛ퟒ ,

Page 31: SOCIETATEA DE ȘTIINȚE - ssmrmh.ro · S.S.M.ROMÂNIA - Filiala Mehedinți 2016 6 REVISTA DE MATEMATICĂ MEHEDINȚEANĂ NR. 17 PRINCIPIUL INCLUDERII ȘI EXCLUDERII Daniel Stretcu

S.S.M.ROMÂNIA - Filiala Mehedinți 2016

31 REVISTA DE MATEMATICĂ MEHEDINȚEANĂ NR. 17

풙풚ퟐ풛 = ퟏ ퟐ ퟑ ퟒ ퟓퟏ ퟑ ퟐ ퟒ ퟓ

풛ퟐ ,

풙ퟐ풚풛 = ퟏ ퟐ ퟑ ퟒ ퟓퟓ ퟑ ퟐ ퟒ ퟏ

풛ퟐ .

Solution: Let’s denote the signature of permutation 풑 as 흌(풑). Denote also the three given permutation on the right side of the equations 풖,풗, and 풘. Using the multiplicative property of the signature, Having the three identities would imply:

흌(풙)흌(풚)흌(풛)ퟐ = 흌(풖)흌(풛)ퟒ,

흌(풙)흌(풚)ퟐ흌(풛) = 흌(풗)흌(풛)ퟐ,

흌(풙)ퟐ흌(풚)흌(풛) = 흌(풘)흌(풛)ퟐ.

The product of the three gives: 흌(풙)ퟒ흌(풚)ퟒ흌(풛)ퟒ = 흌(풖)흌(풗)흌(풘)흌(풛)ퟖ.

This would imply that 흌(풖)흌(풗)흌(풘) = ퟏ. However, note that each of the three given permutations 풖,풗, or 풘 may be split into ퟑ transpozitions, sucht that, for each, the signature is −ퟏ, which leads to a contradiction. Thus the system has no solution.

Acknowledgment: The problem is from Dan Sitaru’s book Math Phenomenon. The solution is by Leo Giugiuc.

Bibliography: http://www.cut-the-knot.org/

SOLVED PROBLEMS-I Nguyen Viet Hung, Titu Zvonaru, Neculai Stanciu

S351. Let 풂,풃, 풄 be positive real numbers such that 풂풃풄 = ퟏ. Prove that

풂 + 풃 + 풄 ≥ퟏ

풂(풃+ ퟏ) +ퟏ

풃(풄 + ퟏ) +ퟏ

풄(풂+ ퟏ) +ퟑퟐ

Proposed by Nguyen Viet Hung, Hanoi, Vietnam

Solution by Titu Zvonaru and Neculai Stanciu, Romania:

Fie 푥,푦, 푧 astfel încât 푎 = , 푏 = , 푐 = . Inegalitatea de demonstrat devine

푥푦

+푦푧

+푧푥≥

푥푦푧(푥 + 푦) +

푦푧푥(푦 + 푧) +

푧푥푦(푧 + 푥) +

12

După eliminarea numitorilor și reducerea termenilor asemenea, rămâne de arătat că:

2 푥 푦푧 + 2 푥 푦 ≥ 푥 푦 푧 + 푥 푦푧 + 6푥 푦 푧 ,

inegalitatea adevărată deoarece:

Page 32: SOCIETATEA DE ȘTIINȚE - ssmrmh.ro · S.S.M.ROMÂNIA - Filiala Mehedinți 2016 6 REVISTA DE MATEMATICĂ MEHEDINȚEANĂ NR. 17 PRINCIPIUL INCLUDERII ȘI EXCLUDERII Daniel Stretcu

S.S.M.ROMÂNIA - Filiala Mehedinți 2016

32 REVISTA DE MATEMATICĂ MEHEDINȚEANĂ NR. 17

2 푥 푦푧 ≥ 푥 푦 푧 + 푥 푦푧 (푀푢푖푟ℎ푒푎푑)

2 푥 푦 ≥ 6푥 푦 푧 (퐴푀 −퐺푀)

Avem egalitate dacă și numai dacă 푥 = 푦 = 푧, adică 푎 = 푏 = 푐 = 1.

S352. In the triangle 푨푩푪, let 흎 denote its Brocard angle, and let 흋 satisfy the identity:

퐭퐚퐧흋 = 퐭퐚퐧푨 + 퐭퐚퐧푩+ 퐭퐚퐧푪

Prove that : 퐜퐨퐬 ퟐ푨 퐜퐨퐬 ퟐ푩 퐜퐨퐬ퟐ푪퐬퐢퐧ퟐ푨 퐬퐢퐧ퟐ푩 퐬퐢퐧ퟐ푪

= − ퟏퟒ

(퐜퐨퐭흎 + ퟑ 퐜퐨퐭흋)

Proposed by Oleg Faynshteyn, Leipzig, Germany

Solution by Titu Zvonaru, Comănești, România and Neculai Stanciu, Buzău, România:

Folosind următoarele relații din Geometria triunghiului, celebra carte a lui Traian Lalescu:

tan퐴 + tan퐵 + tan퐶 = tan퐴 tan퐵 tan퐶(13.4)

sin 2퐴 + sin 2퐵 + sin 2퐶 = 4 sin퐴 sin퐵 sin 퐶(13.7)

cos 2퐴 + cos 2퐵 + cos 2퐶 = −1− 4 cos퐴 cos퐵 cos 퐶 (13.8)

cot휔 =1 + cos 퐴 cos퐵 cos퐶

sin퐴 sin퐵 sin 퐶(17.2)

obținem imediat: − (cot휔 + 3 cot ∅) = − + =

O355. Let 푨푩푪 be a triangle with incenter 푰. Prove that

(푰푩+ 푰푪)ퟐ

풂(풃 + 풄) +(푰푪+ 푰푨)ퟐ

풃(풄 + 풂) +(푰푨 + 푰푩)ퟐ

풄(풂 + 풃) ≤ ퟐ.

Proposed by Nguyen Viet Hung, Hanoi University of Science, Vietnam

Solution by Neculai Stanciu and Titu Zvonaru, Romania:

Deoarece 퐼퐴 = , aplicând formule cunoscute și inegalitatea 퐶퐵푆, obținem

(퐼퐵 + 퐼퐶) =푎푐푠

cos퐵2

+푎푏푠

cos퐶2

=푎푠

푐푠(푠 − 푏)

푎푐+ 푏

푠(푠 − 푐)푎푏

=

=푎푠⋅푠푎

푐(푠 − 푏) + 푏(푠 − 푐) ≤푎푠

(푏 + 푐)(푠 − 푏 + 푠 − 푐) =푎 (푏 + 푐)

Rezultă că: ( )( ) + ( )

( ) + ( )( ) ≤ + + = 2.Avem egalitate dacă și numai dacă

triunghiul este echilateral.

Page 33: SOCIETATEA DE ȘTIINȚE - ssmrmh.ro · S.S.M.ROMÂNIA - Filiala Mehedinți 2016 6 REVISTA DE MATEMATICĂ MEHEDINȚEANĂ NR. 17 PRINCIPIUL INCLUDERII ȘI EXCLUDERII Daniel Stretcu

S.S.M.ROMÂNIA - Filiala Mehedinți 2016

33 REVISTA DE MATEMATICĂ MEHEDINȚEANĂ NR. 17

SOLVED PROBLEMS-II Nguyen Viet Hung, HSGS, Hanoi University of Science, Vietnam

Email address: [email protected]

Problem 1. Let 푨푩푪 be a triangle and let 푷 be an arbitrary point in its plane. Prove that

(풃 + 풄)푷푨 + (풄 + 풂)푷푩+ (풂 + 풃)푷푪 ≥ ퟐ 풂풃풄(풂 + 풃 + 풄)

where 풂 = 푩푪;풃 = 푪푨, and 풄 = 푨푩. Nguyen Viet Hung

The first, we need to prove the following lemmas:

Lemma 1. For all non-negative real numbers 푥,푦, 푧,푢, 푣,푤 then

(푦 + 푧)푢 + (푧 + 푥)푣 + (푥 + 푦)푤 ≥ 2 (푥푦 + 푦푧 + 푧푥)(푢푣 + 푣푤 + 푤푢)

Proof. By the Cauchy – Schwarz inequality, we have

(푦 + 푧)푢 + (푧 + 푥)푣 + (푥 + 푦)푤 = (푥 + 푦 + 푧)(푢 + 푣 + 푤) − (푥푢 + 푦푣 + 푧푤)

= [(푥 + 푦 + 푧 ) + 2(푥푦 + 푦푧 + 푧푥)][(푢 + 푣 + 푤 ) + 2(푢푣 + 푣푤 + 푤푢)]− (푥푢 + 푦푣 + 푧푤)

≥ (푥 + 푦 + 푧 ) + (푢 + 푣 +푤 ) + 2 (푥푦 + 푦푧 + 푧푥)(푢푣 + 푣푢 + 푤푢)− (푥푦 + 푦푣 + 푧푤)

≥ 2 (푥푦 + 푦푧 + 푧푥)(푢푣 + 푣푤 +푤푢).

Lemma 2. Given a triangle 퐴퐵퐶 and let 푃 be any point in its plane, then

푃퐵.푃퐶푏푐

+푃퐶.푃퐴푐푎

+푃퐴.푃퐵푎푏

≥ 1.

Proof. Obviously we have: 푥푃퐴⃗ + 푦푃퐵⃗ + 푧푃퐶⃗ ≥ 0.By expanding this, gives:

(푥 + 푦 + 푧)(푥푀퐴 + 푦푀퐵 + 푧푀퐶 ) ≥ 푎 푦푧 + 푏 푧푥 + 푐 푥푦. Choosing 푥 = ,푦 = , 푧 = ,

then we get + + (푎푃퐴+ 푏푃퐵 + 푐푃퐶) ≥ 푎푏푐.

+.

+.

.

Multiple both sides by 푃퐴 ⋅ 푃퐵 ⋅ 푃퐶, and then simplify, we get the desired result.Come back our problem:

Solution 1. (Nguyen Viet Hung) Applying the lemma 1 for (푥,푦, 푧) = (푏푐, 푐푎,푎푏) and

(푢, 푏,푤) = , , , then using the lemma 2 we obtain immediately the desired result.

Solution 2:(Pham Huang Nguyen)

Lemma 1: Give 푥,푦, 푧,푎, 푏, 푐 > 0 prove:

(푏 + 푐)푥 + (푐 + 푎)푦 + (푎 + 푏)푧 ≥ 2 (푥푦 + 푦푧 + 푧푥)(푎푏 + 푏푐 + 푐푎)

Proof: (푏 + 푐)푥 + (푐 + 푎)푦 + (푎 + 푏)푧 = (푎 + 푏 + 푐)(푥 + 푦 + 푧) − (푎푥 + 푏푦 + 푐푧) =

Page 34: SOCIETATEA DE ȘTIINȚE - ssmrmh.ro · S.S.M.ROMÂNIA - Filiala Mehedinți 2016 6 REVISTA DE MATEMATICĂ MEHEDINȚEANĂ NR. 17 PRINCIPIUL INCLUDERII ȘI EXCLUDERII Daniel Stretcu

S.S.M.ROMÂNIA - Filiala Mehedinți 2016

34 REVISTA DE MATEMATICĂ MEHEDINȚEANĂ NR. 17

= +2(푎푏 + 푏푐 + 푐푎) (푥 + 푦 + 푧 + 2(푥푦 + 푦푧 + 푧푥) − (푎푥 + 푏푦 + 푐푧)

≥ 2 (푎푏 + 푏푐 + 푐푎)(푥푦 + 푦푧 + 푧푥) + (푎 + 푏 + 푐 )(푥 + 푦 + 푧 ) − (푎푥 + 푏푦 + 푐푧)

≥ 2 (푎푏 + 푏푐 + 푐푎)(푥푦 + 푦푧 + 푧푥) + (푎 + 푏 + 푐 )(푥 + 푦 + 푧 ) − (푎푥 + 푏푦 + 푐푧)

≥ 2 (푎푏 + 푏푐 + 푐푎)(푥푦 + 푦푧 + 푧푥)

Lemma 2: Let 푃 be an arbitrary point in of triangle 퐴퐵퐶 prove:

푃퐴.푃퐵.퐴퐵 + 푃퐵.푃퐶.퐵퐶 + 푃퐶.푃퐴.퐶퐴 ≥ 퐴퐵.퐵퐶. 퐶퐴 (Hayashi Inequality.)

Lemma 3: (푎푏 + 푏푐 + 푐푎)(푥푦 + 푦푧 + 푥푧) ≥ (푎+ 푏 + 푐)(푎푦푧 + 푏푧푥 + 푐푥푦)

with 푥,푦, 푧 > 0 and 퐴퐵 = 푐;퐵퐶 = 푎;퐶퐴 = 푏

Solution: We have:

(푥푎푐 + 푦푎푏 + 푧푏푐)(푥푎푏 + 푦푏푐 + 푧푐푎) ≥ (푎푏 + 푏푐 + 푐푎)(푦푧푏푐 + 푥푦푎푐 + 푥푦푎푏) true (**)

Because: (푥푎푐 + 푦푎푏 + 푧푏푐)(푥푎푏 + 푦푏푐 + 푧푐푎)− (푎푏 + 푏푐 + 푐푎)(푦푧푏푐 + 푥푦푎푐 + 푥푦푎푏) =

= 푎푏푐 푥(푎 − 푏)(푎 − 푐) + 푦(푏 − 푐)(푏 − 푎) + 푧(푐 − 푎)(푐 − 푏) we have

푥(푎 − 푏)(푎 − 푐) + 푦(푏 − 푐)(푏 − 푎) + 푧(푐 − 푎)(푐 − 푏) ≥ 0 (vornicu schur inequality)

apply (**) with (푎, 푏, 푐) instead of (푏푐, 푐푎,푎푏) we have

(푎푏 + 푏푐 + 푐푎)(푥푦 + 푦푧 + 푥푧) ≥ (푎 + 푏 + 푐)(푎푦푧 + 푏푧푥 + 푐푥푦)

come back the main problem, we get: 푃퐴 = 푥;푃퐵 = 푦;푃퐶 = 푧. Apply lemma 1:

we have 퐿퐻푆(∗) ≥ 2 (푥푦 + 푦푧 + 푧푥)(푎푏 + 푏푐 + 푐푎)other we have: apply lemma 2

we have 푥푦푐 + 푦푧푎 + 푧푥푏 ≥ 푎푏푐so we need prove (푎푏 + 푏푐 + 푐푎)(푥푦 + 푦푧 + 푥푧) ≥ (푎 + 푏 +푐푎푦푧+푏푧푥+푐푥푦 true → inequality (*) true of course

(푎푏 + 푏푐 + 푐푎)(푥푦 + 푦푧 + 푥푧) ≥ (푎 + 푏 + 푐)(푎푦푧 + 푏푧푥 + 푐푥푦) is lemma 3.

→ (푏 + 푐)푃퐴+ (푐 + 푎)푃퐵 + (푎 + 푏)푃퐶 ≥ 2 푎푏푐(푎 + 푏 + 푐)

Problem 2. Prove the following inequality holds for all positive real numbers 푎, 푏 and 푐,

14푎

+1

4푏+

14푐

+1

2푎 + 푏 + 푐+

12푏 + 푐 + 푎

+1

2푐 + 푎 + 푏≥

1푎 + 푏

+1

푏 + 푐+

1푐 + 푎

Nguyen Viet Hung

Solution: (Nguyen Viet Hung). From 4th dgree Shur’s inequality we have

푎 + 푏 + 푐 + 푎푏푐(푎 + 푏 + 푐) ≥ 푎푏(푎 + 푏 ) + 푏푐(푏 + 푐 ) + 푐푎(푐 + 푎 )

This combines with 푎 + 푏 ≥ 2푎푏, 푏 + 푐 ≥ 2푏푐, 푐 + 푎 ≥ 2푐푎 to give

Page 35: SOCIETATEA DE ȘTIINȚE - ssmrmh.ro · S.S.M.ROMÂNIA - Filiala Mehedinți 2016 6 REVISTA DE MATEMATICĂ MEHEDINȚEANĂ NR. 17 PRINCIPIUL INCLUDERII ȘI EXCLUDERII Daniel Stretcu

S.S.M.ROMÂNIA - Filiala Mehedinți 2016

35 REVISTA DE MATEMATICĂ MEHEDINȚEANĂ NR. 17

푎 + 푏 + 푐 + 푎푏푐(푎 + 푏 + 푐) ≥ 2푎 푏 + 2푏 푐 + 2푐 푎

In this inequality, we replace (푎, 푏, 푐) by 푥 , 푥 , 푥 , with 푥 > 0 then

푥 + 푥 + 푥 + 푥 + 푥 + 푥 ≥ 푥 + 푥 + 푥

It follows that : ∫ 푥 + 푥 + 푥 + 푥 + 푥 + 푥 푑푥 ≥

≥1푥푥 + 푥 + 푥 푑푥

From here, we infer that

14푎

+1

4푏+

14푐

+1

2푎 + 푏 + 푐+

12푏 + 푐 + 푎

+1

2푐 + 푎 + 푏≥

1푎 + 푏

+1

푏 + 푐+

1푐 + 푎

Problem 3. Let 푥,푦, 푧 be real numbers greater than 1 and satisfying

2 + 푥푦 + 푦푧 + 푧푥 ≤ 푥 + 푦+ 푧 + 푥푦푧.

Prove that : 푥푦 + 푦푧 + 푧푥 ≤ 푥푦푧(푥 + 푦 + 푧 − 3)

Nguyen Viet Hung

Solution (Nguyen Viet Hung): The given condition can be rewritten as(푥 − 1)(푦 − 1)(푧 − 1) ≥ 1.

The desired inequality is equivalent to: + + + 3 ≤ 푥 + 푦 + 푧, or ( ) + ( ) + ( ) ≥ 3

Applying the AM-GM inequality we obtain: ( ) + ( ) + ( ) ≥ 3 (푥 − 1)(푦 − 1)(푧 − 1) ≥ 3

and we are done!

Problem 4. Let 푎, 푏, 푐 be the lengths of three sides of a triangle and let 푥,푦, 푧 be arbitraty real numbers. Prove that

푎 (푥 + 2푦푧) + 푏 (푦 + 2푧푥) + 푐 (푧 + 2푥푦) ≥ (푎 + 푏 + 푐 )(푥푦 + 푦푧 + 푧푥)

Nguyen Viet Hung

Solution:( Nguyen Viet Hung) From the well-known inequality

푥 + 푦 + 푧 ≥ 2푦푧 cos퐴 + 2푧푥 cos퐵 + 2푥푦 cos퐶

we replace (푥,푦, 푧) by (푥푎,푦푏, 푧푐) to get

푥 푎 + 푦 푏 + 푧 푐 ≥ 2푦푧푏푐 cos퐴 + 2푧푥푐푎 cos퐵 + 2푥푦푎푏 cos퐶.

Note that: cos퐴 = , cos퐵 = , cos퐶 = .Hence

푥 푎 + 푦 푏 + 푧 푐 ≥ 푦푧(푏 + 푐 − 푎 ) + 푧푥(푐 + 푎 − 푏 ) + 푥푦(푎 + 푏 − 푐 )

This is equivalent :푎 (푥 + 2푦푧) + 푏 (푦 + 2푧푥) + 푐 (푧 + 2푥푦) ≥ (푎 + 푏 + 푐 )(푥푦 + 푦푧 + 푧푥)

Page 36: SOCIETATEA DE ȘTIINȚE - ssmrmh.ro · S.S.M.ROMÂNIA - Filiala Mehedinți 2016 6 REVISTA DE MATEMATICĂ MEHEDINȚEANĂ NR. 17 PRINCIPIUL INCLUDERII ȘI EXCLUDERII Daniel Stretcu

S.S.M.ROMÂNIA - Filiala Mehedinți 2016

36 REVISTA DE MATEMATICĂ MEHEDINȚEANĂ NR. 17

Problem 5. Let 푎, 푏, 푐 ∈ (0,1] such that 푎 + 푏 + 푐 = 2. Prove that

(1 − 푎)(1 − 푏)푎푏

+(1− 푏)(1 − 푐)

푏푐+

(1− 푐)(1 − 푎)푐푎

≤32

.

Nguyen Viet Hung

Solution: (Nguyen Viet Hung )We put: = 푎, = 푏, = 푐,then

푥 = ,푦 = , 푧 = . The given condition becomes: + + = 2 or

푎푎 + 1

+푏

푏 + 1+

푐푐 + 1

= 1

Now we applying Cauchy – Schwarz inequality to get:1 = + + ≥ ( )

which follows that: 푎 + 푏 + 푐 + 3 ≥ (푎 + 푏 + 푐) . This is equivalent to:푎푏 + 푏푐 + 푐푎 ≤

Problem 6. Let 푎, 푏, be non-negative real numbers and 푚,푛 be positive integers. Prove that

1(푚 + 푛)푎 + 1

+1

(푚 + 푛)푏 + 1≥

1푚푎 + 푛푏 + 1

+1

푛푎 +푚푏 + 1

Nguyen Viet Hung

Solution 1: (Nguyen Viet Hung): Applying Cauchy – Schwarz inequality we obtain

1(푚 + 푛)푎 + 1

+ ⋯+1

(푚 + 푛)푎+ 1+

1(푚 + 푛)푏 + 1

+ ⋯+1

(푚 + 푛)푏 + 1

≥(푚 + 푛)

푚(푚 + 푛)푎+ 푛(푚 + 푛)푏+ 푚 + 푛=

푚 + 푛푚푎 + 푛푏 + 1

and ( ) +⋯+ ( ) + ( ) + ⋯+ ( ) ≥ ( )( ) ( ) =

Adding up two above inequalities yields: ( ) + ( ) ≥ +

Divide the both of this inequality by 푚 + 푛 we get the desired result.

Solution 2. (Daniel Sitaru): WLOG suppose that 푎 ≥ 푏. We denote:

푥 = 푡 ; 푧 = 푡 ; 푦 = 푡 ; 푧 = 푡 . By 푎 ≥ 푏 ⇒ 푚푎 ≥ 푛푏푛푎 ≥ 푛푏 ⇒ 푡 ≤ 푡

푡 ≤ 푡⇒⇒ 푥 ≤ 푧;푦 ≤ 푡 ⇒

(푦 − 푡)(푥 − 푧) ≥ 0 ⇒ 푥푦 + 푧푡 ≥ 푥푡 + 푦푧

Page 37: SOCIETATEA DE ȘTIINȚE - ssmrmh.ro · S.S.M.ROMÂNIA - Filiala Mehedinți 2016 6 REVISTA DE MATEMATICĂ MEHEDINȚEANĂ NR. 17 PRINCIPIUL INCLUDERII ȘI EXCLUDERII Daniel Stretcu

S.S.M.ROMÂNIA - Filiala Mehedinți 2016

37 REVISTA DE MATEMATICĂ MEHEDINȚEANĂ NR. 17

푡( ) + 푡( ) ≥ 푡 + 푡

푡( ) + 푡( ) 푑푡 ≥ 푡 푑푡 + 푡 푑푡

푡( )

(푚 + 푛)푎 + 110 +

푡( )

(푚 + 푛)푏 + 110 ≥

푡푚푎 + 푛푏 + 1

10 +

푡푛푎 + 푚푏 + 1

10

1(푚 + 푛)푎 + 1

+1

(푚 + 푛)푏 + 1≥

1푚푎 + 푛푏 + 1

+1

푛푎 +푚푏 + 1

Problem 7. Let 푃 be an interior point of the triangle 퐴퐵퐶. The lines 퐴푃,퐵푃,퐶푃 intersect the sides 퐵퐶,퐶퐴,퐴퐵 at 퐴 ,퐵 ,퐶 , respectively. Determine position of the point 푃 such that

푃퐴푃퐴

+푃퐵푃퐵

+푃퐶푃퐶

= 4푃퐴푃퐴

+푃퐵푃퐵

+푃퐶푃퐶

.

Nguyen Viet Hung

Solution: Nguyen Viet Hung We observe that: = [ ][ ] = [ ]

[ ] = [ ] [ ][ ] ,

where [푋푌푍] denotes area of triangle 푋푌푍. We put [푃퐵퐶] = 푥, [푃퐶퐴] = 푦, [푃퐴퐵] = 푧 then

= .Similarly = , = .

Thus, the given condition is equivalent to: + + = 4 + + ,

푥1푦

+1푧−

4푦+ 푧

+ 푦1푧

+1푥−

4푧 + 푥

+ 푧1푥

+1푦−

4푥 + 푦

= 0,

푥(푦 − 푧)푦푧(푦 + 푧) +

푦(푧 − 푥)푧푥(푧 + 푥) +

푧(푥 − 푦)푥푦(푥 + 푦) = 0.

But the last equality only occurs when 푥 = 푦 = 푧. It follows that 푃 is the centroid of the triangle 퐴퐵퐶.

Problem 8. Let 푎, 푏, 푐 be non-negative real numbers and let

퐴 =1

5푎 + 1+

15푏 + 1

+1

5푐 + 1,

퐵 =1

4푎 + 푏 + 1+

14푏 + 푐 + 1

+1

4푐 + 푎 + 1,

퐶 =1

3푎 + 푏 + 푐 + 1+

13푏 + 푐 + 푎 + 1

+1

3푐 + 푎 + 푏 + 1,

퐷 =1

2푎 + 2푏 + 푐 + 1+

12푏 + 2푐 + 푎 + 1

+1

2푐 + 2푎 + 푏 + 1.

Page 38: SOCIETATEA DE ȘTIINȚE - ssmrmh.ro · S.S.M.ROMÂNIA - Filiala Mehedinți 2016 6 REVISTA DE MATEMATICĂ MEHEDINȚEANĂ NR. 17 PRINCIPIUL INCLUDERII ȘI EXCLUDERII Daniel Stretcu

S.S.M.ROMÂNIA - Filiala Mehedinți 2016

38 REVISTA DE MATEMATICĂ MEHEDINȚEANĂ NR. 17

Prove that 퐴 ≥ 퐵 ≥ 퐶 ≥ 퐷. Nguyen Viet Hung

Solution: (Nguyen Viet Hung) Applying the Cauchy – Schwarz inequality in form

+ + + + ≥ we obtain + + + + ≥

or + ≥ . Similarly + ≥ , + ≥

Adding up these three inequality, we get 퐴 ≥ 퐵 (1)

We also have: + ≥ +

13푏 + 푐 + 푎 + 1

+1

3푐 + 푎 + 푏 + 1≥

44푏 + 4푐 + 2푎 + 2

=2

2푏 + 2푐 + 푎 + 1

13푐 + 푎 + 푏 + 1

+1

3푎 + 푏 + 푐 + 1≥

44푐 + 4푎+ 2푏 + 2

=2

2푐 + 2푎+ 푏 + 1

By adding up these three inequality yields: 퐶 ≥ 퐷 (2)

To be continue, we have again: + ≥ =

14푏 + 푐 + 1

+1

2푎 + 2푏 + 푐 + 1≥

46푏 + 2푐 + 2푎 + 2

=2

3푏 + 푐 + 푎 + 1

14푐 + 푎 + 1

+1

2푏 + 2푐 + 푎 + 1≥

46푐 + 2푎 + 2푏 + 1

=2

3푐 + 푎 + 푏 + 1

Adding up these three inequality gives: 퐵 + 퐷 ≥ 2퐶

This combines with 퐶 ≥ 퐷, we infer that 퐵 ≥ 퐶 (3)

The desired inequalities follow from (1), (2), and (3).

Solution 2: (Leonard Giugiuc)

Lemma: For any 푥,푦, 푧 nonnegative numbers hold the relationships:

푥 + 푦 + 푧 ≥ 푥 푦 + 푦 푧 + 푧 푥 ≥ 푥푦푧(푥 + 푦 + 푧 ) ≥ 푥푦푧(푥푦 + 푦푧 + 푧푥)

Proof. The sequences (푥 ,푦 , 푧 ) and (푥,푦, 푧) are same oriented. By reareangements inequality:

푥 + 푦 + 푧 ≥ 푥 푦 + 푦 푧 + 푧 푥.

Next we will prove that 푥 푦 + 푦 푧 + 푧 푥 ≥ 푥푦푧(푥 + 푦 + 푧 ).If 푥푦푧 = 0, then the problem its solved. Suppose that 푥,푦, 푧 > 0. By CBS we have:

(푥 푦 + 푦 푧 + 푧 푥)(푥 푦푧 + 푥 푦 푧 + 푥푦 푧 ) ≥ (푥 푦푧 + 푥푦 푧 + 푥푦푧 )

Page 39: SOCIETATEA DE ȘTIINȚE - ssmrmh.ro · S.S.M.ROMÂNIA - Filiala Mehedinți 2016 6 REVISTA DE MATEMATICĂ MEHEDINȚEANĂ NR. 17 PRINCIPIUL INCLUDERII ȘI EXCLUDERII Daniel Stretcu

S.S.M.ROMÂNIA - Filiala Mehedinți 2016

39 REVISTA DE MATEMATICĂ MEHEDINȚEANĂ NR. 17

⇒푥 푦 + 푦 푧 + 푧 푥

푥 푦푧 + 푥푦 푧 + 푥푦푧≥

푥 푦푧 + 푥푦 푧 + 푥푦푧푥 푦푧 + 푥 푦 푧 + 푥푦 푧

=푥 + 푦 + 푧푥푦 + 푦푧 + 푧푥

≥ 1 ⇒ 푥 푦 + 푦 푧 + 푧 푥 ≥

≥ 푥푦푧(푥 + 푦 + 푧 )

The inequality 푥푦푧(푥 + 푦 + 푧 ) ≥ 푥푦푧(푥푦 + 푦푧 + 푧푥) its obviously.

Proof of the question: Let be 푡 ∈ [0,1] a fixed number. We apply lemma for 푥 = 푡 ; 푦 = 푡 ; 푧 = 푡^푐.

푡 ≥ 푡 ≥ 푡 ≥ 푡

푡 푑푡 ≥ 푡 푑푡 ≥ 푡 푑푡 ≥ 푡 푑푡

푡5푎 + 1

10 ≥

푡4푎 + 푏 + 1

10 ≥

푡3푎 + 푏 + 푐 + 1

10 ≥

푡2푎 + 2푏 + 푐 + 1

10

15푎 + 1

≥1

4푎 + 푏 + 1≥

13푎+ 푏 + 푐

≥1

2푎 + 2푏 + 푐 + 1

퐴 ≥ 퐵 ≥ 퐶 ≥ 퐷

Problem 9. Find: ∫ . Abnushav Mishra

Proof: (Daniel Sitaru) − = cos 휋 − = cos = cos + 푥 + − 푥 =

= cos휋3

+ 푥 cos휋3− 푥 − sin

휋3

+ 푥 sin휋3− 푥 ,

sin휋3

+ 푥 sin휋3− 푥 =

12

+ cos휋3

+ 푥 cos휋3− 푥

tan 3푥tan 푥

푑푥 = tan휋3

+ 푥 tan휋3− 푥 푑푥 =

+ cos + 푥 cos − 푥

cos + 푥 cos − 푥푑푥 =

=휋

12+

12⋅

2√3

sin + 푥 + − 푥

cos + 푥 cos − 푥푑푥 =

=휋

12+

1√3

ln tan휋3

+ 푥휋

120−

1√3

ln tan휋3− 푥

휋120

Problem 10. Let 푎, 푏, 푐,푑, 푒 and 푓 be positive real numbers. Prove that for each positive integer 푛,

1 ++

1 ++

1 ++

1 ++

1 ++

1 +≥푎 + 푏 + 푐 + 푑 + 푒 + 푓

2

Page 40: SOCIETATEA DE ȘTIINȚE - ssmrmh.ro · S.S.M.ROMÂNIA - Filiala Mehedinți 2016 6 REVISTA DE MATEMATICĂ MEHEDINȚEANĂ NR. 17 PRINCIPIUL INCLUDERII ȘI EXCLUDERII Daniel Stretcu

S.S.M.ROMÂNIA - Filiala Mehedinți 2016

40 REVISTA DE MATEMATICĂ MEHEDINȚEANĂ NR. 17

Kunihiko Chikaya

Proof: (Daniel Sitaru)

1 +=

푎(푎 + 푏) ≥⏞

(∑푎)(∑2푎) =

(∑푎)2 (∑푎) =

∑푎2

=푎 + 푏 + 푐 + 푑 + 푒 + 푓

2

SOLVED PROBLEMS-III

Asupra problemei 4075 din revista CRUX MATHEMATICORUM

D. M. Bătinețu Giurgiu, Neculai Stanciu

În revista canadiană de matematică CRUX MATHEMATICORUM VOL. 41 NO 8, octombrie 2015 a fost propusă de către Leonard Giugiuc și Daniel Sitaru problema 4075 cu următorul enunț:

Dacă în triunghiul 푨푩푪,푩푪 = 풂,푪푨 = 풃,푨푩 = 풄 iar [푨푩푪] este aria triunghiului, atunci

√풂풃풄ퟑ ⋅ 풂ퟐ + 풃ퟐ + 풄ퟐ ≥ ퟒ[푨푩푪] (G-S)

În continuare, ne propunem să prezentăm mai multe soluții ale acestei probleme interesante.

Notăm cu 푠 − semiperimetrul triunghiului, 푟 − raza cercului înscris; 푅 − raza cercului circumscris.

Mai întâi o nouă soluție a inegalității Ionescu – Weitzenbock:

풂ퟐ + 풃ퟐ + 풄ퟐ ≥ ퟒ√ퟑ[푨푩푪] (I-W)

și o soluție a inegalității Polya – Szego: 풂ퟐ ⋅ 풃ퟐ ⋅ 풄ퟐ ≥ ퟒ[푨푩푪]√ퟑ

ퟑ (P-S)

într-adevăr: 푎푏푐 = 4푅[퐴퐵퐶] = 4푅푠푟 ⇒ 푎 푏 푐 = 16푅 푠 푟 =

= 16푅 [퐴퐵퐶] ≥⏞ 16 ⋅ 푅 ⋅ 2푟[퐴퐵퐶] = 32푅푟[퐴퐵퐶] ≥⏞ 32 ⋅2푠

3√3⋅ 푟[퐴퐵퐶] =

=√

[퐴퐵퐶] = [ ]

√= [ ]

√ adică (P-S)

Soluții ale problemei 4075:

Soluția 1: Din inegalitățile (I-W) și (P-S) obținem, respectiv:

√푎 + 푏 + 푐 ≥ 2√3 ⋅ [퐴퐵퐶] și √푎푏푐 ≤[ ]

√. Prin înmulțire:

√푎푏푐 ⋅ 푎 + 푏 + 푐 ≥ 2√3 [퐴퐵퐶] ⋅2 [퐴퐵퐶]

√3= 4[퐴퐵퐶]

Soluția 2: Este cunoscută inegalitatea:

Page 41: SOCIETATEA DE ȘTIINȚE - ssmrmh.ro · S.S.M.ROMÂNIA - Filiala Mehedinți 2016 6 REVISTA DE MATEMATICĂ MEHEDINȚEANĂ NR. 17 PRINCIPIUL INCLUDERII ȘI EXCLUDERII Daniel Stretcu

S.S.M.ROMÂNIA - Filiala Mehedinți 2016

41 REVISTA DE MATEMATICĂ MEHEDINȚEANĂ NR. 17

푎 + 푏 + 푐 ≥ 푎푏 + 푏푐 + 푐푎 = 2[퐴퐵퐶]1

sin퐴+

1sin퐵

+1

sin퐶

Funcția 푓: (0,휋) → (0,∞); 푓(푥) = ;푓 (푥) = − ; 푓 (푥) = > 0;

(∀)푥 ∈ (0,휋) este convexă pe (0,휋). Din inegalitatea Jensen:

1sin퐴

+1

sin퐵+

1sin퐶

≥ 3 ⋅1

sin= 3 ⋅

1sin

= 2√3

În aceste condiții: ∑푎 ≥ ∑푎푏 = 2[퐴퐵퐶]∑ ≥ 2[퐴퐵퐶] ⋅ 2√3 =

= 4[퐴퐵퐶] ⋅ 4√3 ⇒ 푎 + 푏 + 푐 ≥ 2√3[퐴퐵퐶]

√푎푏푐 ≥2 [퐴퐵퐶]

√3푎 + 푏 + 푐 ≥ 2√3[퐴퐵퐶]

⇒ √푎푏푐 푎 + 푏 + 푐 ≥ 4[퐴퐵퐶]

Soluția 3: Din inegalitatea mediilor avem:

푎 + 푏 + 푐 ≥ 3 푎 푏 푐 ⇒ 푎 + 푏 + 푐 ≥ √3 ⋅ √푎푏푐

√푎푏푐 ⋅ 푎 + 푏 + 푐 ≥ 2[퐴퐵퐶]√3

⋅ √3 ⋅ √푎푏푐 ≥2 [퐴퐵퐶]

√3⋅ √3 ⋅

2 [퐴퐵퐶]√3

= 4[퐴퐵퐶]

Soluția 4: Din inegalitatea Bergstrom:

푎 + 푏 + 푐 ≥(푎 + 푏 + 푐)

3⇒

푎 + 푏 + 푐3

≥푎 + 푏 + 푐

3=

2푠3

⇒ 푎 + 푏 + 푐 ≥2푠√3

√푎푏푐 ⋅ 푎 + 푏 + 푐 ≥2 [퐴퐵퐶]

√3⋅

2푠√3

=4푠√푠푟√3 ⋅ √3

≥⏞4푠 3√3푟 ⋅ 푟

√3√3=

=4푠√3 ⋅ √3 ⋅ 푟√3 ⋅ √3

= 4푠푟 = 4[퐴퐵퐶]

Egalitatea are loc numai în cazul triunghiului echilateral.

SOME APPLICATIONS OF THE CONVEXITY AND CONCAVITY

Marian Dincă

Theorem 1: The function 풇(풙) = 풙퐬퐢퐧풙

,풙 ∈ (ퟎ,흅) is convex.

Page 42: SOCIETATEA DE ȘTIINȚE - ssmrmh.ro · S.S.M.ROMÂNIA - Filiala Mehedinți 2016 6 REVISTA DE MATEMATICĂ MEHEDINȚEANĂ NR. 17 PRINCIPIUL INCLUDERII ȘI EXCLUDERII Daniel Stretcu

S.S.M.ROMÂNIA - Filiala Mehedinți 2016

42 REVISTA DE MATEMATICĂ MEHEDINȚEANĂ NR. 17

Proof: 푓 (푥) = ( )

푓 (푥) =(cos 푥 − cos 푥 + sin 푥 ⋅ 푥)(sin 푥) − 2 sin 푥 cos 푥 (sin 푥 − 푥 cos 푥)

(sin 푥)

=(sin푥) ⋅ 푥 − 2 sin 푥 cos 푥 (sin 푥 − 푥 cos 푥)

(sin푥) =(sin 푥) ⋅ 푥 − 2 cos 푥 (sin 푥 − 푥 cos 푥)

(sin 푥) =

=푥((sin 푥) + 2(cos 푥) ) − 2 cos 푥 sin 푥

(sin푥) =푥(1 + (cos 푥) )− 2 cos 푥 sin 푥

(sin푥)

푥 ≥ sin 푥 ⇒ 푥(1 + (cos푥) ) ≥ sin 푥 (1 + (cos 푥) )

sin푥 (1 + (cos 푥) )− 2 cos 푥 sin푥 = sin푥 [1 + (cos 푥) − 2 cos 푥] = sin 푥 (1 − cos 푥) ≥ 0

result: 푓 (푥) ≥ 0,푓(푥) = is convex.

Theorem 2: The function 푔(푥) = , 푥 ∈ 0, , is concave.

Proof: 푔 (푥) = ⋅

푔 (푥) =(− sin푥 ⋅ 푥 + cos 푥 − cos 푥) ⋅ 푥 − 2푥(cos 푥 ⋅ 푥 − sin 푥)

푥=

=− sin 푥 ⋅ 푥 − 2푥 cos 푥 + 2푥 sin 푥

푥=− sin 푥 ⋅ 푥 − 2푥 cos 푥 + 2 sin푥

Prove: − sin푥 ⋅ 푥 − 2푥 cos 푥 + 2 sin푥 ≤ 0 ⇔

2 sin 푥 ≤ sin 푥 ⋅ 푥 + 2푥 cos 푥 ⇔ 2 ≤sin 푥 ⋅ 푥 + 2푥 cos 푥

sin 푥

sin 푥 ⋅ 푥 + 2푥 cos 푥sin 푥

= 푥 +2푥 cos 푥

sin푥= 푥 +

푥sin 푥

⋅ 2 cos 푥 ≥ 푥 + 2 cos 푥

because ≥ 1 and 푥 + 2 cos 푥 ≥ 2 ⇔ 푥 ≥ 2− 2 cos 푥 = 2(1− cos 푥) = 4 sin

and 푥 ≥ 4 sin ⇔ 푥 ≥ 2 sin ⇔ ≥ sin ; result: 푔 (푥) ≤ 0,푔(푥) = concave

sin 푥 sin 푦푥 sin 푦+ 푦 sin 푥

=1+

(푢푠푒푇ℎ푒표푟푒푚1) ≤1

2=

12

sin

≤32

sin∑

∑=

32⋅

sin=

32⋅√

=32⋅√32⋅

3휋

=9√34휋

9√34휋

<54⇔

9√35

< 휋 ⇔ 3.11769145362 < 3.14159265359

Page 43: SOCIETATEA DE ȘTIINȚE - ssmrmh.ro · S.S.M.ROMÂNIA - Filiala Mehedinți 2016 6 REVISTA DE MATEMATICĂ MEHEDINȚEANĂ NR. 17 PRINCIPIUL INCLUDERII ȘI EXCLUDERII Daniel Stretcu

S.S.M.ROMÂNIA - Filiala Mehedinți 2016

43 REVISTA DE MATEMATICĂ MEHEDINȚEANĂ NR. 17

+ + ≤ √ , 푓(푥) = , 푥 ∈ 0, , if 퐴,퐵,퐶 ∈ 0, use Jensen inequality

sin퐴퐴

+sin퐵퐵

+sin 퐶퐶

≤ 3sin

=3 ⋅ √

=9√32휋

if 퐴 ≥ , result 퐵 + 퐶 ≤ ⇒ 퐵 ∈ 0, ,퐶 ∈ 0,

+ ≤ 2 = 2 ; + + ≤ + 2

let 푔(퐴) = + 2

푔 (퐴) =cos퐴 ⋅ 퐴 − sin퐴

퐴−

cos − ⋅ − + sin −

−< 0 ⇒ 푔(퐴) ≤ 푔

휋2

≤ 푔휋3

=9√32휋

Generalisation: Let 푥 ∈ (0,휋) and ∑ 푥 = 휋. Prove that: ∑ ≤ .

TRIANGLE SOLUTION AND INEQUALITIES OF ALEXANDRU SZOROS

By Le Van-June MMXVI

Si rursus incipiens consilium sequerer Platonis incipere studiis mathematics.

Galilæus Galilæi

ABSTRACT

Trigonometric inequalities in triangles may not be a new topic in the world of mathematics, but its problems have never been too old. Otherwise, they are created and challenged it day by day all around the world, and so are the inequalities proposed by teacher Alexandru Szoros. And it is truly my pleasure to represent in this article those problems, which I solved by using the triangle solution method. 1.Introduction:

In triangle 퐴퐵퐶:

sin퐴2≤

푎푏푐(푝 − 푎)(푎 + 푏)(푎 + 푐)

푅2

≥푟 푟 + 푟 푟 + 푟 푟

27≥ 푟

4푅푎≥푏푙

+푐푙≥

8푟푎

Alexandru Szoros

Page 44: SOCIETATEA DE ȘTIINȚE - ssmrmh.ro · S.S.M.ROMÂNIA - Filiala Mehedinți 2016 6 REVISTA DE MATEMATICĂ MEHEDINȚEANĂ NR. 17 PRINCIPIUL INCLUDERII ȘI EXCLUDERII Daniel Stretcu

S.S.M.ROMÂNIA - Filiala Mehedinți 2016

44 REVISTA DE MATEMATICĂ MEHEDINȚEANĂ NR. 17

In this article, I focus on solving these problems by using acknowledgements of triangle solution. In lieu of representing theories and solutions separately, I would discuss on them in parallel sections. The three problems proposed by Alexandru Szoros are represented in Section 2, Section 3, and Section 4, respectively. In Section 5, I represent more discussions of solving techniques and tricks. And Section 6 contains the conclusion of this article.

1. A. Szoros’ Inequality I: Problem: Given triangle 퐴퐵퐶, prove that

sin퐴2≤

푎푏푐(푝 − 푎)(푎 + 푏)(푎 + 푐) (퐼)

Solution: Note that, if we put: 푓(푎;푏; 푐) = 퐿퐻푆(퐼); 푔(푎; 푏;푐) = 푅퐻푆(퐼) Then 푑푒푔푓(푎; 푏; 푐) = 푑푒푔푔(푎; 푏; 푐) = 0. This notation reminds me to rewrite 푅퐻푆(퐼) under trigonometric form. And fortunately, the law of sine indicates that, in any triangle 퐴퐵퐶:

푎sin퐴

=푏

sin퐵=

푐sin 퐶

= 2푅

And this law results in a consequence, if: ℎ(푎;푏; 푐) = ( ; ; )( ; ; ) where 푑푒푔푃 = 푑푒푔푄 = 푛,

then: ℎ(푎; 푏; 푐) = ℎ(sin퐴 ; sin퐵 ; sin퐶). Indeed:

ℎ(푎; 푏; 푐) =(2푅) 푃(sin퐴 ; sin퐵 ; sin 퐶)(2푅) 푄(sin퐴 ; sin퐵 ; sin 퐶) = ℎ(sin퐴 ; sin퐵 ; sin 퐶)

Hence, we are able to rewrite 푅퐻푆(퐼): 푎푏푐

(푝 − 푎)(푎 + 푏)(푎 + 푐) =2푎푏푐

(푏 + 푐 − 푎)(푎 + 푏)(푎 + 푐)

=2 sin퐴 sin퐵 sin 퐶

(sin퐵 + sin 퐶 − sin퐴)(sin퐴 + sin퐵)(sin퐴 + sin 퐶)

=4 sin cos

2 sin cos − 2 sin cos

2 sin cos

2 sin cos

2 sin cos

2 sin cos

=2 sin cos

cos cos − cos cos

sin cos

cos cos

sin cos

cos cos

=2 sin sin sin

cos cos cos − cos=

2 sin sin sin

2 sin sin cos cos=

sin

cos cos

And obviously, since

cos퐴 − 퐵

2cos

퐴 − 퐶2

≤ 1

We get: 푅퐻푆(퐼) ≥ sin ≥ 퐿퐻푆(퐼)

QED. Equality holds when cos cos = 1, in other words, 퐴 = 퐵 = 퐶 = . ■

2. A. Szoros’ Inequality II: Problem: Given triangle 퐴퐵퐶, prove that

푅2

≥푟 푟 + 푟 푟 + 푟 푟

27≥ 푟 (퐼퐼)

Page 45: SOCIETATEA DE ȘTIINȚE - ssmrmh.ro · S.S.M.ROMÂNIA - Filiala Mehedinți 2016 6 REVISTA DE MATEMATICĂ MEHEDINȚEANĂ NR. 17 PRINCIPIUL INCLUDERII ȘI EXCLUDERII Daniel Stretcu

S.S.M.ROMÂNIA - Filiala Mehedinți 2016

45 REVISTA DE MATEMATICĂ MEHEDINȚEANĂ NR. 17

2.1. About inradius and exradii of a triangle: In this subsection, I revise the calculation of inradius and exradii of a triangle.

Figure 1. Excircle (IA; ra) of triangle ABC

From Figure 1, we get:

푆 = 푆 + 푆 − 푆 =12퐴퐵. 퐼 푍 +

12퐴퐶. 퐼 푌 −

12퐵퐶. 퐼 푋

=12

(푏 + 푐 − 푎)푟 = (푝 − 푎)푟

And therefore: 푟 = = . Totally similarly

푟 =푆

푝 − 푏=

2푆푐 + 푎 − 푏

;푟 =푆

푝 − 푐=

2푆푎 + 푏 − 푐

And additionally: 푟 = = .

2.2. Solution for A. Szoros’ Inequality II Rewrite (퐼퐼):

푅2

≥4푆27

1(푏 + 푐 − 푎)(푐 + 푎 − 푏) ≥

4푆(푎 + 푏 + 푐)

⟺푅2

≥4푆 (푎 + 푏 + 푏)

27(푏 + 푐 − 푎)(푐 + 푎 − 푏)(푎 + 푏 − 푐) ≥4푆

(푎 + 푏 + 푐)

Firstly, I show the back part of this double-inequality, which could be rewritten as (푎 + 푏 + 푐) ≥ 27(푏 + 푐 − 푎)(푐 + 푎 − 푏)(푎 + 푏 − 푐)

Since 푎, 푏, and 푐 are three sides of a triangle, there exist three positive numbers 푥, 푦, and 푧 such that: 푎 = 푥 + 푦; 푏 = 푦+ 푧; 푐 = 푧 + 푥.

Hence, the above inequality should be transformed to 8(푥 + 푦 + 푧) ≥ 216푥푦푧 ⟺ (푥 + 푦 + 푧) ≥ 27푥푦푧 ⟺ 푥 + 푦 + 푥 ≥ 3 푥푦푧

This is true due to the AM-GM inequality of three non-negative numbers. Return to the front part, where we need to prove 푅2

≥푝푆

27(푝 − 푎)(푝 − 푏)(푝 − 푐) =푝27

⟺푅2≥

푝3√3

=푎 + 푏 + 푐

6√3=푅(sin퐴 + sin퐵 + sin 퐶)

3√3⟺sin퐴 + sin퐵 + sin 퐶 ≤

3√32

This is also true in any triangle 퐴퐵퐶 thanks to the proof of using Jensen’s inequality to concave functions. However, here I represent an amazing way to prove this, which was recommended by teacher Tran Phuong. Recall:

푇 = sin퐴 + sin퐵 + sin 퐶 = sin퐴 + sin퐵 + sin(퐴 + 퐵) =

Page 46: SOCIETATEA DE ȘTIINȚE - ssmrmh.ro · S.S.M.ROMÂNIA - Filiala Mehedinți 2016 6 REVISTA DE MATEMATICĂ MEHEDINȚEANĂ NR. 17 PRINCIPIUL INCLUDERII ȘI EXCLUDERII Daniel Stretcu

S.S.M.ROMÂNIA - Filiala Mehedinți 2016

46 REVISTA DE MATEMATICĂ MEHEDINȚEANĂ NR. 17

= sin퐴 + sin퐵 + sin퐴 cos퐵 + sin퐵 cos퐴

=2√3

√32

sin퐴 +√32

sin퐵 +1√3

√3 cos퐴 sin퐵 + √3 cos퐵 sin퐴

Using AM-GM inequality from the geometric mean side, we get

푇 ≤1√3

34 + (sin퐴) +

34 + (sin퐵) +

12√3

[3(cos퐴) + (sin퐵) + 3(cos퐵) + (sin퐴) ] =3√3

2

Both parts of inequality (퐼퐼) are proven, QED. Equalities hold when 푎 = 푏 = 푐. ■ In Section 5, I would demonstrate more about Tran Phuong’s recommendation.

3. A. Szoros’ Inequality III: Problem: Given triangle 퐴퐵퐶, prove that: ≥ + ≥ (퐼퐼퐼)

3.1. About angle bisectors of a triangle

Figure 2. Angle bisector AD and incenter I of triangle ABC

Lemma I. (where 푙 is the length of 퐴퐷): 푙 = Proof. According to the angle bisector’s property

퐵퐷퐴퐵

=퐶퐷퐴퐶

=퐵퐷 + 퐶퐷퐴퐵 + 퐴퐶

=퐵퐶

퐴퐵 + 퐴퐶=

푎푏 + 푐

⟹ 퐵퐷 =푎푐푏 + 푐

; 퐶퐷 =푎푏푏 + 푐

Using the law of cosine in triangles 퐴퐵퐷 and 퐴퐶퐷:

퐵퐷 = 퐴퐵 + 퐴퐷 − 2퐴퐵.퐴퐷 cos퐵퐴퐷퐶퐷 = 퐴퐶 + 퐴퐷 − 2퐴퐶.퐴퐷 cos 퐶퐴퐷

⎩⎨

⎧푎푐푏 + 푐

= 푐 + 푙 − 2푐푙 cos퐴2

푎푏푏 + 푐

= 푏+ 푙 − 2푏푙 cos퐴2

⟹푎 (푐 − 푏 )

(푏+ 푐) = 푐 − 푏 − 2(푐 − 푏)푙 cos퐴2⟹

푎푏 + 푐

= 푏 + 푐 − 2푙 cos퐴2

(we might assume that 푏 ≠ 푐)

⟹ 2푙 cos퐴2

= 푏 + 푐 −푎푏 + 푐

=푏 + 푐 − 푎 + 2푏푐

푏 + 푐=

2푏푐 cos퐴 + 2푏푐푏 + 푐

=2푏푐(1 + cos퐴)

푏 + 푐

=4푏푐 cos

푏 + 푐⟹ 푙 =

2푏푐 cos푏 + 푐

If 푏 = 푐, triangle 퐴퐵퐶 becomes isosceles at vertex 퐴, where 푙 = ℎ , and the Lemma is clearly true since

2푏푐 cos푏 + 푐

=2푏 cos

2푏= 푏 cos

퐴2

= ℎ

Page 47: SOCIETATEA DE ȘTIINȚE - ssmrmh.ro · S.S.M.ROMÂNIA - Filiala Mehedinți 2016 6 REVISTA DE MATEMATICĂ MEHEDINȚEANĂ NR. 17 PRINCIPIUL INCLUDERII ȘI EXCLUDERII Daniel Stretcu

S.S.M.ROMÂNIA - Filiala Mehedinți 2016

47 REVISTA DE MATEMATICĂ MEHEDINȚEANĂ NR. 17

Lemma II. (where 푙 is the length of 퐴퐷): 푙 = 푏푐푝(푝 − 푎) Proof. In Figure 2, I solve for 퐴퐷 in triangle 퐴퐵퐷, of which 퐴퐵, 퐵퐷, and cos퐵 are given.

퐴퐷 = 퐴퐵 + 퐵퐷 − 2퐴퐵.퐵퐷 cos퐴퐵퐷

⟹푙 = 푐 +푎푐푏 + 푐

− 2푐푎푐푏 + 푐

푐 + 푎 − 푏2푐푎

푙 = 푐 +푎푐푏 + 푐

−푐(푐 + 푎 − 푏 )

푏 + 푐

=1

(푏 + 푐)[푎 푐 + 푐 (푏 + 푐) − 푐(푏 + 푐)(푐 + 푎 − 푏 )]

=1

(푏 + 푐)[푎 푐 + 푐(푏 + 푐)(푏푐 + 푐 − 푐 − 푎 + 푏 )]

=1

(푏 + 푐) [푎 푐 + (푏푐 + 푐 )(푏푐 − 푎 + 푏 )]

=1

(푏 + 푐)(푎 푐 + 푏 푐 − 푎 푏푐 + 푏 푐 + 푏푐 − 푎 푐 + 푏 푐 )

=1

(푏 + 푐)(푏 푐 + 2푏 푐 + 푏푐 − 푎 푏푐) =

푏푐(푏 + 2푏푐 + 푐 − 푎 )(푏 + 푐)

=푏푐[(푏 + 푐) − 푎 ]

(푏 + 푐) =푏푐(푏 + 푐 + 푎)(푏 + 푐 − 푎)

(푏 + 푐) =4푏푐푝(푝 − 푎)

(푏 + 푐)

⟹푙 =2

푏 + 푐푏푐푝(푝 − 푎)

QED. ● From the above lemmas, we could obtain: cos = ( )

And in Figure 2, using the law of sine in triangle 퐴퐵퐷, we get: =

And therefore

sin퐴2

=퐵퐷 sin퐵퐴퐷

=푏푐푝(푝 − 푎)

=푎푏푐

4푅 푏푐푝(푝 − 푎)=

푆푏푐푝(푝 − 푎)

=(푝 − 푏)(푝 − 푐)

푏푐

These formulas allow us to find trigonometric functions of with three sides given. Furthermore, I would represent one direct way to find any triangle’s incenter in plane 푂푥푦. Property: Given triangle 퐴퐵퐶 with incenter I, then: 푎퐼퐴⃗ + 푏퐼퐵⃗ + 푐퐼퐶⃗ = 0⃗ Proof: In Figure 2, 퐵퐼 is the angle bisector of 퐴퐵퐶. Consider triangle 퐴퐵퐷:

퐼퐷퐵퐷

=퐼퐴퐴퐵

⟹ 퐼퐴 =퐴퐵퐵퐷

퐼퐷 =푐퐼퐷 =

푏 + 푐푎

퐼퐷 ⟹ 푎퐼퐴⃗ = −(푏 + 푐)퐼퐷⃗

And note that: = ⟹푏퐷퐵⃗ = −푐퐷퐶⃗. Hence: 푎퐼퐴⃗ + 푏퐼퐵⃗+ 푐퐼퐶⃗ = −(푏 + 푐)퐼퐷⃗ + 푏퐼퐵⃗ + 푐퐼퐶⃗ = 푏 퐼퐵⃗ − 퐼퐷⃗ + 푐 퐼퐶⃗ − 퐼퐷⃗ = 푏퐷퐵⃗ + 푐퐷퐶⃗ = 0⃗

3.2. Solution for A. Szoros’ Inequality III Rewrite (퐼퐼퐼): 4푅푎≥

푏+

푐≥

8푟푎⟺ 4푅 ≥

푎 + 푏

2 cos+푎 + 푐

2 cos≥ 8푟

⟺ 4푅 ≥2푅(sin퐴 + sin퐵)

2 cos+

2푅(sin퐴 + sin 퐶)

2 cos≥ 8푟

Page 48: SOCIETATEA DE ȘTIINȚE - ssmrmh.ro · S.S.M.ROMÂNIA - Filiala Mehedinți 2016 6 REVISTA DE MATEMATICĂ MEHEDINȚEANĂ NR. 17 PRINCIPIUL INCLUDERII ȘI EXCLUDERII Daniel Stretcu

S.S.M.ROMÂNIA - Filiala Mehedinți 2016

48 REVISTA DE MATEMATICĂ MEHEDINȚEANĂ NR. 17

⟺ 4 ≥2 sin cos

sin+

2 sin cos

sin≥

8푟푅⟺ 2 ≥ cos

퐴 − 퐵2

+ cos퐴− 퐶

2≥

4푟푅

The front part is obviously true since cos 휃 ≤ 1, ∀휃 ∈ ℝ, so equality holds only when 퐴 = 퐵 = 퐶 = . However, to prove the back part of this double-inequality, I am supposed to return to initial steps, with help from Lemma II. Rewrite the back part of (퐼퐼퐼):

푏(푎 + 푏)

2 푎푏푝(푝 − 푐)+

푐(푎 + 푐)

2 푎푐푝(푝 − 푏)≥

8푆푝푎

⟺(푎 + 푏)√푎푏

2 푝 − 푐+

(푎 + 푐)√푎푐2 푝 − 푏

≥ 8 (푝 − 푎)(푝 − 푏)(푝 − 푐)(∗)

Using AM-GM inequality, of which: 푎 + 푏 ≥ 2√푎푏; 푎 + 푐 ≥ 2√푎푐

퐿퐻푆(∗) ≥푎푏푝 − 푐

+푎푐푝 − 푏

≥2푎√푏푐

(푝 − 푏)(푝 − 푐)

So it is enough to show that 푎√푏푐 ≥ 4 (푝 − 푎)(푝 − 푏)(푝 − 푐) (푝 − 푏)(푝 − 푐) ⟺ 푎 푏푐 ≥ 16(푝 − 푎)(푝 − 푏)(푝 − 푐) (푝 − 푏)(푝 − 푐)(∗∗)

Indeed, from geometric mean side to arithmetic mean side: 2 (푝 − 푏)(푝 − 푐) ≤ 푝− 푏 + 푝 − 푐 = 푎,2 (푝 − 푏)(푝 − 푐) ≤ 푎

2 (푝 − 푐)(푝 − 푎) ≤ 푏, 2 (푝 − 푎)(푝 − 푏) ≤ 푐 ⟹ 푅퐻푆(∗∗) ≤ 퐿퐻푆(∗∗) Equality holds when 푎 = 푏 = 푐. QED. ■

4. More discussions 4.1. The relation between circumscribed radius and inradius of a triangle

From A. Szoros’ Inequalities II and III, we obtain 푅 ≥ 2푟. This is also consequently resulted in from many triangle-trigonometric problems, and I would represent one direct way of proving this:

푅 ≥ 2푟 ⟺푎푏푐4푆

≥2푆푝⟺ 푎푏푐 ≥

8푆푝

= 8(푝 − 푎)(푝 − 푏)(푝 − 푐)

This is due to a result in last subsection, where:2 (푝 − 푏)(푝 − 푐) ≤ 푎 2 (푝 − 푐)(푝 − 푎) ≤ 푏, 2 (푝 − 푎)(푝 − 푏) ≤ 푐 And the back part of (퐼퐼퐼) leads to a nice problem:

cos퐴− 퐵

2+ cos

퐴 − 퐶2

≥4푟푅

Or generally: cos + cos + cos ≥

4.2. Proving basic triangle-trigonometric inequalities: In triangle 푨푩푪 푇 = sin퐴+ sin퐵 + sin 퐶 ≤ √ (푖푣), 푈 = cos퐴 + cos퐵 + cos 퐶 ≤ (푣)

푉 = sin + sin + sin ≤ (푣푖), 푊 = cos + cos + cos ≤ √ (푣푖푖) Except (푣), all (푖푣), (푣푖), and (푣푖푖) could be proven by using Jensen’s inequality to concave functions. But I would represent a method proposed by Tran Phuong to prove these inequalities, just like in subsection 4.2. Moreover, problem (푣) could be proven by using the SOS method, a highly convenient way.

Page 49: SOCIETATEA DE ȘTIINȚE - ssmrmh.ro · S.S.M.ROMÂNIA - Filiala Mehedinți 2016 6 REVISTA DE MATEMATICĂ MEHEDINȚEANĂ NR. 17 PRINCIPIUL INCLUDERII ȘI EXCLUDERII Daniel Stretcu

S.S.M.ROMÂNIA - Filiala Mehedinți 2016

49 REVISTA DE MATEMATICĂ MEHEDINȚEANĂ NR. 17

Indeed:푅퐻푆(푣) − 퐿퐻푆(푣) = 1− cos퐴 − (cos퐵 + cos퐶) +

= 2 sin퐴2

− 2 cos퐵 + 퐶

2cos

퐵 − 퐶2

+12

cos퐵 − 퐶

2+

12

sin퐵 − 퐶

2

= 2 sin퐴2

− 2 sin퐴2

cos퐵 − 퐶

2+

12

cos퐵 − 퐶

2+

12

sin퐵 − 퐶

2

= 2 sin − cos + sin ≥ 0. QED. Equality holds when 퐴 = 퐵 = 퐶 = . Tran Phuong’s recommendation: Proof of (푣), assuming 퐶 = max( ; ){퐴;퐵;퐶}:

푈 = cos퐴 + cos퐵 + cos퐶 = cos 퐴 + cos퐵 − cos(퐴 + 퐵) = 1(cos퐴 + cos퐵)− cos퐴 cos퐵 + sin퐴 sin퐵

Using AM-GM inequality:

푈 ≤12

[1 + (cos퐴 + cos퐵) ] − cos퐴 cos퐵 +12

[(sin퐴) + (sin퐵) ] =32

Proof of (푣푖):

푉 = sin퐴2

+ sin퐵2

+ sin퐶2

= sin퐴2

+ sin퐵2

+ cos퐴 + 퐵

2

= 1 sin퐴2

+ sin퐵2

+ cos퐴2

cos퐵2− sin

퐴2

sin퐵2

Using AM-GM inequality:

푉 ≤12

1 + sin퐴2

+ sin퐵2

− sin퐴2

sin퐵2

+12

cos퐴2

+ cos퐵2

=32

Proof of (푣푖푖):

푊 = cos퐴2

+ cos퐵2

+ cos퐶2

= cos퐴2

+ cos퐵2

+ sin퐴 + 퐵

2

= cos퐴2

+ cos퐵2

+ sin퐴2

cos퐵2

+ sin퐵2

cos퐴2

=2√3

√32

cos퐴2

+√32

cos퐵2

+1√3

√3 sin퐴2

cos퐵2

+ √3 sin퐵2

cos퐴2

Using AM-GM inequality:

푊 ≤1√3

34

+ cos퐴2

+34

+ cos퐵2

+1

2√33 sin

퐴2

+ cos퐴2

+ 3 sin퐵2

+ cos퐵2

=3√3

2

QED. To all of these problems, equalities hold when triangle 퐴퐵퐶 is equilateral. ●

4.3. About A. Szoros’ Inequality I:

sin퐴2

=(푝 − 푏)(푝 − 푐)

푏푐

Using this formula, then (퐼) becomes:

푎푏푐(푝 − 푎)(푎 + 푏)(푎 + 푐) ≥

(푝 − 푏)(푝 − 푐)푏푐

⟺ 푎푏푐√푏푐 ≥ (푝 − 푎)(푎 + 푏)(푎 + 푐) (푝 − 푏)(푝 − 푐)(푣푖푖푖) Like I mentioned above:2 (푝 − 푐)(푝 − 푎) ≤ 푏

2 (푝 − 푎)(푝 − 푏) ≤ 푐 ⟹ 푏푐 ≥ 4(푝 − 푎) (푝 − 푏)(푝 − 푐) So it would be enough were it possible to show that:

Page 50: SOCIETATEA DE ȘTIINȚE - ssmrmh.ro · S.S.M.ROMÂNIA - Filiala Mehedinți 2016 6 REVISTA DE MATEMATICĂ MEHEDINȚEANĂ NR. 17 PRINCIPIUL INCLUDERII ȘI EXCLUDERII Daniel Stretcu

S.S.M.ROMÂNIA - Filiala Mehedinți 2016

50 REVISTA DE MATEMATICĂ MEHEDINȚEANĂ NR. 17

푎√푏푐 ≥(푎 + 푏)(푎 + 푐)

4

However, this is totally not true, since: 푎 + 푏 ≥ 2√푎푏; 푎 + 푐 ≥ 2√푎푐 Therefore, problem (푣푖푖푖) could be an unpleasant challenge in any contest.

5. Conclusion:Solving Alexandru Szoros’ Inequalities about triangle and trigonometric factors brought me three lessons. First, transform expressions from complicated to simple forms. Second, try to find common factors of both sides. And final, basic methods with classical inequalities should be approached initially, there must be a key to clinch the problems.

References [1] https://www.facebook.com/groups/Imad.Zak/ [2] http://www.geogebra.org/

[3] Tran Phuong, Selected Topics for Vietnam College Entrance Exam Preparation-Trigonometric Relations, Ha Noi Publishing Co., 2008, p.

PROBLEME PROPUSE

Clasa I I.1. Câte pătrate albe are o tablă de șah? Maria Ungureanu

I.2. Câte zile de luni pot fi într-o lună? Mariana Cornea

I.3. Construiți o propoziție care să aibă trei cuvinte având 4,3 respectiv 4 litere. Gheorghița Tufiș

I.4. De câte ori poate să apară un an bisect în trei ani consecutivi? Maria Ungureanu

I.5. Alina are cu 4 ani mai mult decât Maria. Suma vârstelor lor este 6. Câți ani are Alina? Dar Maria?

Mariana Cornea

I.6. Ana, Maria și Alina au schimbat fotografii între ele. Câte fotografii au fost necesare?

Gheorghița Tufiș

I.7. Câte litere are cuvântul „Geometrie”? Maria Ungureanu

I.8. Câte picioare are familia „Caprei cu trei iezi”? Mariana Cornea

I.9. „Scufița Roșie” avea în coșul pentru bunica 2 mere și cu 6 mai multe nuci. Câte nuci avea în coș?

Gheorghița Tufiș

I.10. Cosmina a rezolvat 2 probleme și Andrei cu 3 mai multe. Câte probleme am rezolvat împreună?

Maria Ungureanu

Clasa a II-a II.1. Adaugă suma numerelor 415 și 245 la diferența lor. Maria Ungureanu

Page 51: SOCIETATEA DE ȘTIINȚE - ssmrmh.ro · S.S.M.ROMÂNIA - Filiala Mehedinți 2016 6 REVISTA DE MATEMATICĂ MEHEDINȚEANĂ NR. 17 PRINCIPIUL INCLUDERII ȘI EXCLUDERII Daniel Stretcu

S.S.M.ROMÂNIA - Filiala Mehedinți 2016

51 REVISTA DE MATEMATICĂ MEHEDINȚEANĂ NR. 17

II.2. Suma a trei numere este 989. Primul număr este 591 iar al doilea este cu 32 mai mare decât al treilea. Aflați al treilea număr. Mariana Cornea

II.3. Trei saci de făină cântăresc 90 kg. Primii doi saci cântăresc 60 kg iar al doilea cântărește cu 4 kg mai puțin decât al treilea. Câte kg cântărește fiecare sac? Gheorghița Tufiș

II.4. Din cei 400 l de apă minerală primită, un magazin a vândut în prima zi 143 l iar a doua zi cu 28 l mai mult. Câți litri de apă minerală i-au rămas de vândut? Maria Ungureanu

II.5. Câte zile au (împreună) lunile de primăvară și lunile de toamnă? Mariana Cornea

II.6. Câte zile sunt în 10 săptămâni și 4 zile? Puteți preciza câte zile de joi pot fi în 4 săptămâni și 5 zile? Gheorghița Tufiș

II.7. Din banii pe care îi are Cosmina ar putea cumpăra un ghiozdan de 42 lei, un penar de 11 lei și i-ar mai rămâne 8 lei. Câți lei are Cosmina? Maria Ungureanu

II.8. Trei prieteni au vârstele de 7 ani, 8 ani și 10 ani. Câți ani vor avea împreună după 5 ani?

Mariana Cornea

II.9. În două butoaie sunt 540 l de ulei. Dacă din primul s-ar vinde 50 l iar din al doilea 90 l, în fiecare butoi ar rămâne 200 l. Câți litri de ulei conține fiecare butoi? Gheorghița Tufiș

II.10. Curtea bunicii lui Adrian are formă de pătrat cu latura de 70 m. Câți metri aleargă Adrian dacă înconjoară curtea de 2 ori și jumătate? Maria Ungureanu

Clasa a III-a III.1. Scrieți 5 numere consecutive, în ordine crescătoare, unul din ele fiind 198. Câte posibilități există? Maria Ungureanu

III.2. Scrie 7 numere impare consecutive, în ordine descrescătoare, unul dintre ele fiind cel mai mare număr format din trei cifre identice. Mariana Cornea

III.3. Scrieți numărul 푎푏푐 știind că 푎 este succesorul numărului 7,푏 este predecesorul numărului 4 iar 푐 este triplul numărului 푏. Gheorghița Tufiș

III.4. De câte ori se folosește cifra 5 în scrierea numerelor cuprinse între 200 și 300?

Maria Ungureanu

III.5. O carte are 98 pagini. Andrei citește timp de o săptămână astfel: luni și miercuri câte 5 pagini; marți, joi și sâmbătă câte 10 pagini, vineri și duminică același număr de pagini. Câte pagini a citit duminică? Mariana Cornea

III.6. Produsul a trei numere este 64. Produsul primelor două este 8 iar produsul ultimelor două este 16. Să se afle numerele. Gheorghița Tufiș

III.7. Un bidon plin cu apă cântărește 22 kg iar bidonul plin pe jumătate cântărește 12 kg. Cât cântărește bidonul gol? Maria Ungureanu

Page 52: SOCIETATEA DE ȘTIINȚE - ssmrmh.ro · S.S.M.ROMÂNIA - Filiala Mehedinți 2016 6 REVISTA DE MATEMATICĂ MEHEDINȚEANĂ NR. 17 PRINCIPIUL INCLUDERII ȘI EXCLUDERII Daniel Stretcu

S.S.M.ROMÂNIA - Filiala Mehedinți 2016

52 REVISTA DE MATEMATICĂ MEHEDINȚEANĂ NR. 17

III.8. Cu cât este mai mare jumătatea numărului 98 față de jumătatea sumei vecinilor numărului 21? Mariana Cornea

III.9. În 6 ore un călător parcurge 24 km iar un biciclist 60 km. Care este diferența vitezelor?

Gheorghița Tufiș

III.10. Pe un raft sunt 142 cărți, iar pe altul de două ori mai puțin. Câte cărți sunt pe cele două rafturi? Maria Ungureanu

Clasa a IV-a IV.1. La o bibliotecă sunt 40000 volume. S-au mai adus 543 volume și s-au împrumutat cititorilor 297 volume. Câte volume au rămas în bibliotecă? Maria Ungureanu

IV.2. La ora de educație fizică elevii s-au aranjat în 4 rânduri a câte 5 elevi și 3 rânduri a câte 3 elevi. Câți elevi participă la oră? Mariana Cornea

IV.3. La o librărie s-au adus 245 cutii a câte 4 stilouri. Se vând 105 cutii. Câte stilouri au rămas în magazin? Gheorghița Tufiș

IV.4. Într-o cutie sunt 15 rânduri a câte 4 pungi cu orez pe fiecare rând. Știind că fiecare pungă cântărește 2 kg să se afle câte kilograme de orez sunt în cutie. Maria Ungureanu

IV.5. Diferența a două numere naturale este 80. La împărțirea lor se obține câtul 3 și restul 10. Să se afle numerele. Mariana Cornea

IV.6. Să se calculeze: 1440: 6: 5: 4: 3: 2 și 720: 2: 3: 4. Gheorghița Tufiș

IV.7. Să se determine numărul necunoscut 푥: a. [(푥 − 1): 1000 + 900]: 100 − 100 = 0,

b. (푥: 10 + 35: 7): 3 = 3. Maria Ungureanu

IV.8. Într-o livadă sunt 7 rânduri a câte 112 cireși și 9 rânduri a câte 143 vișini. Câți pomi fructiferi sunt în livadă? Mariana Cornea

IV.9. Suma a două numere este 275 și un număr este cu 5 mai mare decât dublul celuilalt. Să se afle numerele. Gheorghița Tufiș

IV.10. Mama avea 32 de ani când s-a născut Andreea și 35 de ani când s-a născut Cosmin. Câți ani are acum fiecare, dacă impreună au 41 ani? Maria Ungureanu

Clasa a V-a V.1. Să se afle trei numere naturale știind că produsul primelor două este 96, produsul ultimelor două este 336 iar suma dintre primul și ultimul număr este 36. Gheorghe Calafeteanu V.2. Un număr este cu 47 mai mare decât alt număr natural. Dacă împărțim suma la diferența lor obținem 1 și restul 28. Să se afle numerele. Gheorghe Calafeteanu V.3. Să se determine 푥 din următoarea egalitate: 푥(푥 + 2)(푥 + 4)( ) = (푥 + 2)(푥 − 4)(푥 − 2)( )

Page 53: SOCIETATEA DE ȘTIINȚE - ssmrmh.ro · S.S.M.ROMÂNIA - Filiala Mehedinți 2016 6 REVISTA DE MATEMATICĂ MEHEDINȚEANĂ NR. 17 PRINCIPIUL INCLUDERII ȘI EXCLUDERII Daniel Stretcu

S.S.M.ROMÂNIA - Filiala Mehedinți 2016

53 REVISTA DE MATEMATICĂ MEHEDINȚEANĂ NR. 17

Daniel Stretcu V.4. Aflați numărul 푎푏, 푏 ≠ 0, știind că îndeplinește simultan condițiile:

1. 푎 + 푏 este număr prim; 2. 9푎 + 푏 este număr divizibil cu 3; 3. 푎 + 푏 este pătrat perfect. Daniel Stretcu

V.5. Doi prieteni locuiesc în același bloc, unul la etajul 2, apartamentul 105, celălalt la etajul 3, apartamentul 139. Știind că pe fiecare scară, atât la parter cât și la etaj sunt câte 6 apartamente, aflați câte nivele are blocul. Daniel Stretcu V.6. Să se determine un număr de șase cifre scris în baza zece, căruia punându-i la stânga cifra 3 se obține un număr de două ori mai mare decât dacă am pune cifra 7 în dreapta numărului.

Daniel Stretcu V.7. Reconstituiți: 퐴퐴퐴퐴− 퐴퐴퐴− 퐴 − 퐴 = 퐵퐶퐵퐶 Daniel Stretcu V.8. Se consideră mulțimile 퐴 = {푥 ∈ 푁/2 < 푥 < 2 } și 퐵 = {푥 ∈ 푁∗/푥 ≤ 10 }. Comparați card퐴 cu card퐵, unde card퐴 reprezintă numărul elementelor mulțimii 퐴.

Daniel Stretcu V.9. Să se determine mulțimile 푋 și 푌 care satisfac simultan condițiile:

i. 푐 ∈ 푋 ∩ 푌; ii. 푋 ∖ {푎, 푓} = 푌 ∪ {푏,푑, 푒,푔}; iii. 푌 ∖ {푎, 푐, 푓} = 푋 ∩ {푏,푑}. Daniel Stretcu

V.10. Determinați 푛 numărul natural nenul pentru care numărul 푎 = 2 ⋅ 4 ⋅ 6 ⋅ … ⋅ (2푛) + 617 este pătrat perfect. Daniel Stretcu V.11. Numărul din mijloc al unui șir constituit din 2015 numere naturale consecutive este 2016. Determinați primul și ultimul termen al șirului. Daniel Stretcu V.12. Comparați numerele푎 și푏 unde: 푎 = 2 , 푏 = (5 − 4 ⋅ 5 + 9 : 3 − 5 )

Doru Preșneanu

V.13. Fie 푎, 푏, 푐,푑, 푒 ∈ ℕ∗ astfel încât 푎- + 푏 (푎 + 푐)(푎 + 푑)(푎 + 푒) = 5005.Arătați că 푎, 푏, 푐,푑, 푒 nu pot fi toate numere prime. Daniel Sitaru

V.14. Să se determine 푎, 푏, 푐 ∈ ℕ încât: 0,푎푎(푏) + 0,푏푏(푐) + 0, 푐푐(푎) = 0, (6) Daniel Sitaru

V.15. Să se determine a, b, c ∈ ℕ încât: 0, a(bb) + 0, b(cc) + 0, c(aa) = 0, (6) Daniel Sitaru

V.16. Să se scrie numărul 5005 ca o sumă de numere naturale al căror produs să fie 5005.

Daniel Sitaru

V.17. Fie 푝, 푞, 푟 numere prime distincte mai mari decât 3. Se cere restul împărțirii sumei

푝 + 푞 + 푟 la 12. Daniel Sitaru

V.18. Să se rezolve ecuația: 푎푏푐7 + 푏푐7푎 + 푐7푎푏 + 7푎푏푐 = 푎푏푐 + 푏푐푎 + 푐푎푏 + 푥(푎 + 푏 + 푐)

Daniel Sitaru

V.19. Arătați că un număr natural cu 2020 cifre din care jumătate sunt cifra 2 și jumătate sunt cifra4 nu poate fi pătrat perfect. Daniel Sitaru

Page 54: SOCIETATEA DE ȘTIINȚE - ssmrmh.ro · S.S.M.ROMÂNIA - Filiala Mehedinți 2016 6 REVISTA DE MATEMATICĂ MEHEDINȚEANĂ NR. 17 PRINCIPIUL INCLUDERII ȘI EXCLUDERII Daniel Stretcu

S.S.M.ROMÂNIA - Filiala Mehedinți 2016

54 REVISTA DE MATEMATICĂ MEHEDINȚEANĂ NR. 17

V.20. Să se determine cifrele푎, 푏, 푐astfel încât: = = = 111 Daniel Sitaru

V.21. Fie 푥 ∈ ℕ∗,푛 ≥ ℕ.Să se determine 푎, 푏, 푐astfel încât:

푎푏푐( ) + 푏푐푎( ) + 푐푎푏( ) = 27(푥 + 푥 + 1) Daniel Sitaru

V.22. Câte numere de forma 푎푏푎푏3푐 sunt divizibile cu 4 și au proprietatea că 푎푏 este pătrat perfect? Daniel Sitaru

V.23. Aflați 푥, 푎, 푏, 푐 ∈ ℕ încât: 푥 ∙ 푎푏푐 = 1푎푏푐 Daniel Sitaru

V.24. Să se determine numerele naturale m și n știind că 푚 (푛 + 3) + (1 + 2 + 2 + ⋯+ 2 ): [2 − (2 ) ] = 31214 Iuliana Trașcă

V.25. Să se determine câtul și restul împărțirii numărului 10 la 41. Dana Paponiu

V.26. Determinați numerele naturale 푥,푦, 푧 care verifică simultan relațiile:

4푥 = 5푦 + 6푧 + 10 și 17푥 = 5푦 + 6푧 + 35 Dan Nedeianu

V.27. Să se rezolve în mulțimea numerelor naturale ecuația: 푥 − 2 = 1 Manuela Prajea

V.28. Să se afle toate mulțimile de forma {푎, 푏, 푐} ⊂ ℕ∗ cu 푎 + 푏 + 푐 = 276 și 푎 < 푏 < 푐.

Manuela Prajea

V.29. În produsul 1 ⋅ 2 ⋅ 3 ⋅ … ⋅ 2007 ⋅ 2008 se elimină toate numerele pare și toate numerele divizibile cu 5. Să se determine ultima cifră a produsului numerelor rămase.

Gheorghe Căiniceanu

V.30. Să se afle toate numerele naturale 푎, 푏 pentru care 푎 = 푎 . Manuela Prajea

V.31. Dacă 푎, 푏 sunt numere naturale nenule cu 푎 > 푏 să se arate că:

(푎 + 푏) − (푎 − 푏) este divizibil cu 4. Manuela Prajea

V.32. Să se afle câtul împărțirii numărului: 퐴 = 10 + 4 ⋅ 5 − 25 ⋅ 128 la numărul 100 .

Manuela Prajea

V.33. Să se compare numerele 5 și 2 . Manuela Prajea

V.34. Un număr natural n se numește “cub prietenos” dacă 푛 se poate scrie ca sumă de n numere naturale consecutive.Arătați că : a) 7 este cub prietenos, b)orice număr impar poate fi cub prietenos.

Victor Săceanu

V.35. Aflați numărul natural 푎푏푐 pătrat perfect pentru care avem 5a=4b-9c. Victor Săceanu

V.36. Arătați că: 10 + 24 < 26 . Victor Săceanu

Page 55: SOCIETATEA DE ȘTIINȚE - ssmrmh.ro · S.S.M.ROMÂNIA - Filiala Mehedinți 2016 6 REVISTA DE MATEMATICĂ MEHEDINȚEANĂ NR. 17 PRINCIPIUL INCLUDERII ȘI EXCLUDERII Daniel Stretcu

S.S.M.ROMÂNIA - Filiala Mehedinți 2016

55 REVISTA DE MATEMATICĂ MEHEDINȚEANĂ NR. 17

Clasa a VI-a VI.1. Știind că c.m.m.m.c a două numere naturale 푎 și 푏 este 2016, iar c.m.m.d.c este 32, unde 푎 < 푏, să se afle numerele. Elena Rîmnicianu VI.2. a. Stabiliți valoarea de adevăr a propoziției:

12016

−1

2017=

12016 ⋅ 2017

b. Demonstrați că:

⋅+

⋅+

⋅+⋯+

⋅< Elena Rîmnicianu

VI.3. Să se arate că ⋅ ⋅… ⋅ este număr natural. Elena Rîmnicianu

VI.4. Se consideră numerele raționale pozitive: 푋 = + + +

și 푦 = 1 + + + + ⋯+ + + + +⋯+ Să se calculeze 푥 + 푦,푦 − 푥 și (푦 − 2015) Elena Rîmnicianu VI.5. Fie 푎, 푏, 푐 numere naturale astfel încât 2푎 = 5푏 + 7푐. Să se demonstreze că 2 divide pe 푏 + 푐 și 7 divide pe 푎 + 푏. Gheorghe Calafeteanu VI.6. Determinați toate numerele de forma 푎푏푐 pentru care numărul 푛 = 푎푏푐 + 푏푐푎 + 푐푎푏 este număr de 4 cifre divizibil cu 75. Gheorghe Calafeteanu VI.7. Să se arate că numărul 푛 = 120020150002017 nu este pătrat perfect.

Gheorghe Calafeteanu

VI.8. Fie 푎,푛 numere naturale astfel încât 2푛 + 3 divide 푎 și 3푛 + 4 divide 푎. Arătați că 6푛 + 17푛 + 12 divide 푎. Gheorghe Calafeteanu VI.9. Să se arate că într-o clasă de 29 de elevi se află 5 elevi care nu sunt prieteni sau există un elev cu cel puțin 7 prieteni. Daniel Stretcu VI.10. Determinați numerele naturale mai mari decât 2000 și mai mici decât 3000, care împărțite la 6,7 și 11 dau resturile 3,1 și respectiv 4. Daniel Stretcu VI.11. Să se calculeze măsurile unghiurilor triunghiului isoscel 퐴퐵퐶, cu [퐴퐵] ≡ [퐴퐶], pentru care bisectoarea unghiului exterior al unghiului 퐶 intersectează latura [퐴퐵] într-un punct 퐷, astfel încât 퐴 ∈ [퐵퐷] și 푚(푆퐵퐷퐶) = 33°. Daniel Stretcu VI.12. Să se determine restul împărțirii pătratului unui număr prim 푝 la 12, unde 푝 > 2016.

Daniel Stretcu

VI.13. Arătați că orice număr natural 푛 mai mare decât 2016 este suma a două numere naturale compuse. Daniel Stretcu VI.14. Să se găsească cel mai mic număr cu 30 de cifre, care are suma cifrelor egală cu 30 și se divide la 30. Daniel Stretcu

Page 56: SOCIETATEA DE ȘTIINȚE - ssmrmh.ro · S.S.M.ROMÂNIA - Filiala Mehedinți 2016 6 REVISTA DE MATEMATICĂ MEHEDINȚEANĂ NR. 17 PRINCIPIUL INCLUDERII ȘI EXCLUDERII Daniel Stretcu

S.S.M.ROMÂNIA - Filiala Mehedinți 2016

56 REVISTA DE MATEMATICĂ MEHEDINȚEANĂ NR. 17

VI.15. În câte zerouri se termină numărul 2004!, unde 2004! = 1 ∗ 2 ∗ 3 ∗ … ∗ 2004. Daniel Stretcu

VI.16. Doi elevi cu vârstele de 14 și respectiv 13 ani, au depus la o bancă, în total suma de 424.000 lei. Dobânda băncii este de 12% pe an și se adaugă sumei depuse la sfârșitul fiecărui an. Să se afle ce sumă a depus fiecare știind că la împlinirea vârstei de 18 ani fiecare va avea în bancă aceeași sumă.

Daniel Stretcu VI.17. Rezolvați în numere întregi următoarea ecuație: 푥 (푦 − 2) + 푦 (푥 − 2) = 4

Daniel Stretcu VI.18. Fie 푥 = 푎푎… 푎

푏푏…푏

.Arătați că: dacă 푎 + 푏 = 11 atunci numărul 푥 este divizibil cu 121.

Daniel Stretcu VI.19. Din numărul , după eliminarea primelor două cifre zecimale nenule, se obține numărul

⋅ , cu 푎, 푏 ∈ 푁. Să se determine 푎 și 푏, știind că 푎 și 푏 sunt prime între ele. Daniel Stretcu VI.20. Într-o clasă sunt 30 de elevi. Să se arate că în clasă există cinci elevi care s-au născut în aceeași zi a săptămânii. Daniel Stretcu VI.21. Să se arate că oricum am alege 1009 numere dintre 1,2,3, … ,2015, 2016 există două dintre acestea care sunt prime între ele. Daniel Stretcu VI.22. Care este numărul maxim de numere dintre 1,2,3, … ,2015,2016 care se pot alege astfel încât suma oricăror două numere să nu se dividă cu diferența lor? Daniel Stretcu VI.23. În triunghiul isoscel 퐴퐵퐶, cu [퐴퐵] ≡ [퐴퐶] și 푚(∢퐵퐴퐶) = 100°, pe laturile (퐵퐶) și respectiv (퐴퐵) se consideră punctele 퐷 și respectiv 퐸 astfel încât [퐶퐷] ≡ [퐶퐴] și 퐷퐸 ∥ 퐴퐶. Aflați măsura unghiului ∢퐷퐸퐶. Daniel Stretcu VI.24. În triunghiul 퐴퐵퐶 luăm punctele 퐷 și 퐸 pe latura (퐵퐶) astfel încât [퐵퐷] ≡ [퐷퐸] ≡ [퐷퐶]. Știind că dreapta 퐴퐷 trece prin mijlocul înălțimii [퐵퐵 ], 퐵 ∈ 퐴퐶 și dreapta 퐴퐸 trece prin mijlocul bisectoarei (퐶퐶 ,퐶 ∈ 퐴퐵. Să se arate că [퐵퐵 ], [퐶퐶 ] și mediana [퐴푀] sunt concurente, cu 푀 ∈ (퐵퐶). Daniel Stretcu VI.25. Fie 퐴퐵퐶퐷 un pătrat și în interiorul triunghiului 퐴퐵퐶 considerăm un punct 푀 astfel încât 푚(∢푀퐴퐵) = 푚(∢푀퐶퐴) = 훼. Să se calculeze 푚(∢푀퐷퐴). Daniel Stretcu VI.26. Arătați că 3 + 10 este divizibil cu 11. Daniel Stretcu VI.27. Măsurile 푎 și 푏 a două unghiuri complementare sunt direct proporționale cu numerele prime 푝 și 푞. Determinați 푎 și 푏, știind că este un număr prim. Daniel Stretcu VI.28. Fie 푥 , 푥 , 푥 , … , 푥 numere naturale astfel încât: 3 < 푥 < 푥 < 푥 < ⋯ < 푥 și

푥 + 푥 + 푥 +⋯+ 푥 = 174. Demonstrați că cel puțin unul dintre cele 13 numere este prim. Daniel Stretcu

Page 57: SOCIETATEA DE ȘTIINȚE - ssmrmh.ro · S.S.M.ROMÂNIA - Filiala Mehedinți 2016 6 REVISTA DE MATEMATICĂ MEHEDINȚEANĂ NR. 17 PRINCIPIUL INCLUDERII ȘI EXCLUDERII Daniel Stretcu

S.S.M.ROMÂNIA - Filiala Mehedinți 2016

57 REVISTA DE MATEMATICĂ MEHEDINȚEANĂ NR. 17

VI.29. Să se arate că 7 + 3 + 1 ⋮ 7 Doru Preșneanu

VI.30. Să se arate că fracţia se simplifică prin 7, ∀푛휖ℕ* Doru Preșneanu

VI.31. Să se arate că∀푛 ,푛 , … ,푛 ∈ ℕ, 푛푢푚ă푟푢푙푎 − 1 ⋮ 2016푢푛푑푒

푎 = 2017 + 2017 +⋯+ 2017 Doru Preșneanu

VI.32. Să se arate că푎 = 7 ⋅ 3 − 49 ⋮ 490∀푛 ∈ ℕ* Doru Preșneanu

VI.33. Să se afle푎, 푏 ∈ ℕ, 푏푝푟푖푚ș푖푎 + 푏 + 푎 = 508. Doru Preșneanu

VI.34. a. Dacă 푁 este un număr natural de cel puțin două cifre, iar 푃 este produsul cifrelor numărului 푁, să se arate că 푁 ≠ 푃. b. Dacă 퐴 = 푎푏 − 푎푏|푎, 푏, 푐푖푓푟푒,푎 ≠ 0 , determinați cel mai mare, respectiv cel mai mic element al lui 퐴. Dan Nedeianu

VI.35. Se amestecă o cantitate de saramură cu concentrația 푝% sare și o cantitate de saramură cu concentrația (푝 + 10)% sare și se obține o saramură cu concentrația (푝 + 3)% sare. Care este raportul cantităților amestecate? (1 ≤ 푝 ≤ 30;푝 ∈ ℕ). Gheorghe Căiniceanu

VI.36. Fiind date 21 de puncte distincte în plan, există o așezare a acestora astfel încât numărul maxim de drepte distincte care se pot duce prin oricare două dintre ele să fie 209?

Manuela Prajea

VI.37. Să se afle numărul meselor și persoanelor dintr-o încăpere știind că dacă așezam câte două persoane la masă rămân în picioare un procent de 20% iar dacă așezăm câte trei persoane la masă va rămâne o masă cu doar două persoane și încă un procent de 14, (285714)% mese libere.

Manuela Prajea

VI.38. Fie 푚,푛 ∈ ℕ∗ cu proprietatea + ∈ ℕ. Să se arate că 푑 ≤ 푚 + 푛, unde 푑 = (푚,푛).

Gheorghe Căiniceanu

VI.39. Să se afle numărul elementelor mulțimii: 퐴 = 푥 ∈ ℕ ∈ ℕ . Manuela Prajea

VI.40. Să se afle perechile de numere naturale (푥,푦) care verifică relația:

(3 + 2 ) − (3 − 2 ) = 1 Manuela Prajea

VI.41. Calculați pătratul produsul divizorilor numărului 2016. Victor Săceanu

VI.42. Aflați 푚,푛,푝 ∈ ℕ∗ ,푝-număr prim încât: 푝 = 푛(푛 + 3푛 + 3).

Dan Nedeianu(„CătălinȚigaeru”- Contest-2016)

VI.43. Să se arate că: + + ⋯+ < 0, (001)

Daniel Sitaru,Dorel Căpraru

Page 58: SOCIETATEA DE ȘTIINȚE - ssmrmh.ro · S.S.M.ROMÂNIA - Filiala Mehedinți 2016 6 REVISTA DE MATEMATICĂ MEHEDINȚEANĂ NR. 17 PRINCIPIUL INCLUDERII ȘI EXCLUDERII Daniel Stretcu

S.S.M.ROMÂNIA - Filiala Mehedinți 2016

58 REVISTA DE MATEMATICĂ MEHEDINȚEANĂ NR. 17

VI.44. Demonstrați că nu există 푥,푦, 푧 ∈ ℕ;푥 ≠ 푦 ≠ 푧 ≠ 푥 încât: 1024 + 1024 = 1024 .

Daniel Sitaru,Dorel Căpraru Clasa a VII-a

VII.1. Fie ∆퐴퐵퐶 ascuțitunghic, cu 퐴퐵 ≠ 퐴퐶 și punctul 푇 din interiorul său cu 푚 퐴푇퐵 = 푚 퐵푇퐶 = 푚 퐶푇퐴 = 120°. Notăm {퐵′} = 퐵푇 ∩ 퐴퐶 și {퐶′} = 퐶푇 ∩ 퐴퐵. a. Demonstrați că dreptele 퐵′퐶′ și 퐵퐶 sunt concurente. b. Dacă {푀} = 퐵′퐶′ ∩ 퐵퐶, atunci = . Leonard Giugiuc VII.2. Să se arate că dacă 푎, 푏, 푐 ∈ [0,∞) atunci: 푎 (푏 + 푐) + 푏 (푎 + 푐) + 푐 (푎 + 푏) ≥ 8푎푏푐 Claudia Nănuți

VII.3. Să se arate că dacă 푎, 푏, 푐 ∈ (0,∞) atunci:

푎 + 푏 + 푐 +푏 푐푎

+푎 푐푏

+푎 푏푐

≥ 6푎푏푐

Claudia Nănuți VII.4. Să se arate că dacă 푥,푦, 푧 ∈ (0,∞) atunci:

∑ ≥ 2∑ ≥ 3 Mihaly Bencze VII.5. Să se determine raza cercului înscris în triunghiul de laturi 푎, 푏, 푐 știind că √푎 − 18 + 81 + |12− 푐| = 225− 30푏 + 푏 Elena Rîmnicianu VII.6. Dacă √ √ =

√ √ , atunci media geometrică a numerelor 푥 și 푦 este un număr

natural. Elena Rîmnicianu VII.7. Să se afle numărul întreg 푛 astfel încât √푛 + 2푛 + 8 să fie număr natural.

Elena Rîmnicianu VII.8. Să se determine numerele reale 푥 și 푦 astfel încât √25푥 + 20푥+ 8 + 9푦 − 30푦+ 61 = 8. Elena Rîmnicianu VII.9. Să se arate că 푎 = (푛 + 2016푛)(푛 + 2016푛 + 10) + 25 este pătrat perfect oricare ar fi 푛 număr întreg. Elena Rîmnicianu VII.10. Să se demonstreze că: a. √6 + √14 + √21 < 12 b.

√+

√+

√< Gheorghe Calafeteanu

VII.11. Fie numerele naturale 푎 și 푏 pentru care = 1− + − + ⋯− + . Să se arate că 3023 divide numărul 푎. Daniel Stretcu

VII.12. Fie mulțimile 퐴 = 푥 ∈ 푁 ∈ 푍 și 퐵 = 푦 ∈ 푍 ∈ 푁 .

Calculați 퐶 = (퐴\퐵) ∪ (퐵\퐴). Daniel Stretcu VII.13. Într-un sistem solar sunt 2015 planete, pe fiecare din ele aflându-se un astronaut care se uită prin telescop la planeta cea mai apropiată. Să se demonstreze că dacă distanțele dintre planete sunt diferite două câte două, atunci există o planetă la care nu se uită nimeni.

Daniel Stretcu

Page 59: SOCIETATEA DE ȘTIINȚE - ssmrmh.ro · S.S.M.ROMÂNIA - Filiala Mehedinți 2016 6 REVISTA DE MATEMATICĂ MEHEDINȚEANĂ NR. 17 PRINCIPIUL INCLUDERII ȘI EXCLUDERII Daniel Stretcu

S.S.M.ROMÂNIA - Filiala Mehedinți 2016

59 REVISTA DE MATEMATICĂ MEHEDINȚEANĂ NR. 17

VII.14. Să se arate că numărul 푥 = 1,2332333233332 … este irațional. Daniel Stretcu VII.15. Numărul 2 are 90 cifre. Să se arate că una din cifre se repetă de cel puțin 10 ori.

Daniel Stretcu VII.16. Să se afle primele 2005 zecimale ale numărului 0, 11 … 1

Daniel Stretcu

VII.17. Să se determine numărul de soluții numere întregi pozitive ale ecuației: + = Daniel Stretcu VII.18. Determinați toate soluțiile întregi pentru ecuația: (푥 + 1)(푦 + 4) = 2(2푥 − 푦)(2 − 푥푦) = 8푥푦 + 9 Daniel Stretcu VII.19. Numerele 2 și 5 au fost scrise unul alipit de celălalt, în scrierea zecimală. Câte cifre au fost scrise în total? Daniel Stretcu VII.20. Determinați numărul întreg 푛 astfel încât numărul 푝 = 푛 + 16푛− 17 să fie un număr prim.

Daniel Stretcu VII.21. Să se descompună în sumă de șapte pătrate perfecte numărul: 7푛 + 28

Doru Preșneanu

VII.22. Să se afle푛 ∈ ℕ푑푎푐ă ∈ ℕ Doru Preșneanu

VII.23. Să se determine numerele 푎, 푏 ∈ ℕ știind că

(1 + 2 + 2 +⋯+ 2 ): 2 + 1 + 2016 = 2 Iuliana Trașcă

VII.24. Să se rezolve în mulțimea numerelor întregi ecuația:

7푥 + 7푥 + ⋯+ 7푥 = 2푥 + 2푥 +⋯+ 2푥 . Dana Paponiu

VII.25. Să se determine numerele reale 푥, 푦, 푧 care satisfac relația:

√3 + 2푥 − 푥 + 5 + 4푦 − 푦 = 푧 − 6푧 + 14. Dana Paponiu

VII.26. Se consideră numerele reale nenule distincte care verifică relația:

푎 + = 푏 + = 푐 + . Să se determine valorile posibile ale expresiei √

.

Dana Paponiu

VII.27. Se consideră patrulaterul convex 퐴퐵퐶퐷, cu 푀 mijlocul segmentului 퐵퐶.

Determinați măsura unghiului 퐵퐴퐷, știind că 푀퐴 ⊥ 푀퐷,푚 퐴퐷푀 = 15° și 퐴퐵 + 퐶퐷 = 퐴퐷.

Dan Nedeianu

VII.28. a. Dacă 퐴퐵퐶퐷 este un pătrat de latură 1 cm și 푀,푁,푃,푄 sunt puncte situate pe laturile (퐴퐵), (퐵퐶), (퐶퐷), (퐷퐴) astfel ca lungimile segmentelor (퐴푀), (퐴푄), (퐶푁) și (퐶푃) să fie mai mici ca 0,5 cm să se arate că 퐴[푀푁푃푄] < 0,5 cm2 b. Dacă 푥, 푦, 푧, 푡 sunt strict pozitive și mai mici ca atunci:

Page 60: SOCIETATEA DE ȘTIINȚE - ssmrmh.ro · S.S.M.ROMÂNIA - Filiala Mehedinți 2016 6 REVISTA DE MATEMATICĂ MEHEDINȚEANĂ NR. 17 PRINCIPIUL INCLUDERII ȘI EXCLUDERII Daniel Stretcu

S.S.M.ROMÂNIA - Filiala Mehedinți 2016

60 REVISTA DE MATEMATICĂ MEHEDINȚEANĂ NR. 17

1 < 푥푦 + (1− 푦)(1− 푧) + (1− 푥)(1 − 푡) + 푧푡 < 2 Manuela Prajea

VII.29. Fie 퐴퐵퐶퐷 un patrulater convex și 푀,푁,푃,푄 situate pe (퐴퐵), (퐵퐶), (퐶퐷), (퐷퐴) cu = = = . Să se arate că: a. 4푀푄 ⋅ 푁푃 ≤ 퐵퐷 ; b. dacă 푀푁 ∩푃푄 = {푇} atunci 퐴,퐶 ,푇 sunt

puncte coliniare. Manuela Prajea

VII.30. Fie mulțimea 퐴 = 푥 ∈ ℝ ∈ ℕ . Să se afle 퐴 ∩ℕ;퐴 ∩ 푄 și 퐴 ∩ (ℝ ∖ 푄).

Manuela Prajea

VII.31. Fie rombul 퐴퐵퐶퐷 și punctele 퐸,퐹,퐺 ,퐻 situate pe laturile (퐴퐵), (퐵퐶), (퐶퐷), (퐷퐴) astfel ca (퐴퐸) = (퐶퐹) = (퐶퐺) = (퐴퐻). Fie {푀} = 퐷퐸 ∩ 퐵퐻, {푁} = 퐴퐹 ∩ 퐶퐸; {푝} = 퐵퐺 ∩ 퐷퐹,

{푄} = 퐶퐻 ∩ 퐴퐺. Să se arate că 푀푁푃푄 este un romb. Manuela Prajea

VII.32. Se dă triunghiul ABC în care BC=2AB, m(<B)=60° și AC=24 cm.Aflați perimetrul și aria triunghiului ABC. Victor Săceanu VII.33. În triunghiul cu 푚(∢퐴) = 90° și 푚(∢퐶) = 30° considerăm bisectoarea 퐵퐷, 퐷 ∈ 퐴퐶, mediana 퐶퐸,퐸 ∈ 퐴퐵 concurente în punctul 푃 și 퐴푃 ∩ 퐵퐶 = {푀}. Aflați valoarea raportului .

Victor Săceanu

VII.34. Fie 퐴퐵퐶퐷 un patrulater convex, ortodiagonal (퐴퐶 ⊥ 퐵퐷) și 퐸,퐹 mijloacele lui [퐴퐶], respectiv [퐵퐷]. Arătați că avem relația: 퐴퐵 + 퐶퐷 = 퐵퐶 + 퐴퐷 = 2(퐴퐸 + 퐵퐹 + 퐸퐹 )

Victor Săceanu

VII.35. Comparați suma divizorilor numărului 2015 cu suma divizorilor numărului 2015 cu suma divizorilor numărului 2016. Victor Săceanu

Clasa a VIII-a VIII.1. Să se afle 푎, 푏 ∈ ℝ încât: 2푎 + 5푏 − 2푎푏 − 12푎+ 18푏 + 26 = 0 Dan Nănuți VIII.2. Să se arate că dacă 푎, 푏, 푐 ∈ [0,∞) atunci:

푎 (푐 + 4푎푏) + 푏 (푎 + 4푐) + 푐 (푏 + 4푎) ≥ 12푎푏푐 Claudia Nănuți

VIII.3. Să se arate că dacă 푎, 푏, 푐 ∈ (0,∞) atunci:

푎√푏푐 + 푏√푐푎 + 푐√푎푏 ≤√푎푏 + √푏푐 + √푐푎

3≤ 푎푏 + 푏푐 + 푐푎

Claudia Nănuți

VIII.4. Să se arate că ecuația: (푥 − 1)(푦 − 1)(푧 − 1) = 푢 − 푣 are o infinitate de soluții în ℤ. Mihaly Bencze

VIII.5. Rezolvați ecuația: + − = + 2016− 푥 Elena Rîmnicianu

Page 61: SOCIETATEA DE ȘTIINȚE - ssmrmh.ro · S.S.M.ROMÂNIA - Filiala Mehedinți 2016 6 REVISTA DE MATEMATICĂ MEHEDINȚEANĂ NR. 17 PRINCIPIUL INCLUDERII ȘI EXCLUDERII Daniel Stretcu

S.S.M.ROMÂNIA - Filiala Mehedinți 2016

61 REVISTA DE MATEMATICĂ MEHEDINȚEANĂ NR. 17

VIII.6. Fie 푎 ∈ [0,∞); 푖 ∈ 1,2016;푎 + 푎 +⋯+ 푎 = 1. Să se arate că: 2푎 + 1+ 2푎 + 1 +⋯+ 2푎 + 1 ≤ 2017 Gheorghe Calafeteanu VIII.7. Să se rezolve ecuația: (푥 − 6푥 + 2015)(푥 − 6푥 + 2017) + 2016 = 0 Gheorghe Calafeteanu VIII.8. Dacă 푥,푦, 푧 ∈ (0,∞) să se determine o relație între 푥,푦, 푧 astfel încât:

= D.M. Bătinețu – Giurgiu VIII.9. Fie 푆 , 푆 , 푆 , 푆 ariile fețelor opuse vârfurilor 퐴,퐵,퐶 respectiv 퐷 în tetraedrul [퐴퐵퐶퐷]. Să se arate că:

∑ ≥ D.M. Bătinețu – Giurgiu; Neculai Stanciu

VIII.10. Se dau numerele:

푥 =1

1 ⋅ 2−

12 ⋅ 3

+1

3 ⋅ 4−

14 ⋅ 5

+⋯+1

2007 ⋅ 2008−

12008 ⋅ 2009

și 푦 =⋅−

⋅+

⋅−

⋅+⋯+

⋅−

Să se calculeze suma 푆 = 푥 + 2푦. Daniel Stretcu VIII.11. Un număr natural de 2004 cifre are 2003 cifre egale cu 9. Demonstrați că el nu poate fi pătrat perfect. Daniel Stretcu VIII.12. Să se rezolve în mulțimea numerelor întregi ecuația diofantică: 푥 + 5푥 + 1 = 푦

Daniel Stretcu

VIII.13. Descompuneți numărul 5 − 1 ca produs de șase factori, toți mai mari ca 5 . Daniel Stretcu

VIII.14. Dacă푥 ∈ [−2, ∞)ș푖푦 ∈ [−2, ∞)푎푡푢푛푐푖푛푢푚ă푟푢푙푥푦 + 2푥 + 2푦 ∈ [−4, ∞)

Doru Preșneanu

VIII.15. Știind că intervalul(푎, 푏)are proprietatea că(푎, 푏) ∩ ℤ = {2016},arătați că

푆 = |푎 − 2016| + |푏 − 2016|− |푎 − 푏|este constant. Doru Preșneanu

VIII.16. Să se determine 푥 ∈ ℝ știind că: (푥 + 3푥 − 4) + (푥 − 3푥 − 10) = 8(푥 − 7)

Iuliana Trașcă

VIII.17. Să se arate că în orice triunghi 퐴퐵퐶 este valabilă relația:

푎푏(푎 + 푏) + 푏푐(푏 + 푐) + 푐푎(푐 + 푎) > 3푎푏푐 Ion Nănuți, Valeriu Drulă

VIII.18. Să se demonstreze că, pentru orice numere pozitive a, bși c, are loc inegalitatea:

√+

√+

ó

√. Dana Paponiu

VIII.19. Dacă 푥,푦, 푧 ∈ ℝ, astfel încât 12푥 푦 + 8푥 푧 > 9푧 + 36푥푦푧 + 48푦 , să se arate că 푥 > 3푦. Dan Nedeianu

Page 62: SOCIETATEA DE ȘTIINȚE - ssmrmh.ro · S.S.M.ROMÂNIA - Filiala Mehedinți 2016 6 REVISTA DE MATEMATICĂ MEHEDINȚEANĂ NR. 17 PRINCIPIUL INCLUDERII ȘI EXCLUDERII Daniel Stretcu

S.S.M.ROMÂNIA - Filiala Mehedinți 2016

62 REVISTA DE MATEMATICĂ MEHEDINȚEANĂ NR. 17

VIII.20. Să se arate că dacă 푎, 푏, 푐,푑 ∈ 푄∗ și 푎 + 푏 + 푐 + 푑 = 0 atunci

푎 + 푏 + 푐 + 푑 + 4푎푏푐푑 > 0 și √푎 + 푏 + 푐 + 푑 + 4푎푏푐푑 ∈ ℝ ∖ 푄. Manuela Prajea

VIII.21. Să se determine funcțiile 푓:ℝ → ℝ cu proprietatea că:

푎푓(푎) + 푏푓(푏) = √푎 + 푏 ⋅ 푓 (푎) + 푓 (푏), pentru orice 푎, 푏 ∈ ℝ. Manuela Prajea

VIII.22. Să se afle 푎, 푏 ∈ 푄 știind că: √ √√

∈ 푄 Manuela Prajea

VIII.23. Să se afle mulțimea: 퐹 = {푓:ℝ → ℝ|푓(푥) + 푎푓(1 − 푥) = 푎푥, (∀)푥 ∈ ℝ}

Manuela Prajea

VIII.24. Un punct mobil trebuie să ajungă din vârful 퐴 în vârful 퐶 al pătratului 퐴퐵퐶퐷 de latură 1 și are în față un baraj triunghiular 퐵퐷퐸, perpendicular pe planul pătratului, cu laturile 퐵퐸 = 1;퐷퐸 = 1. Știind că nu poate ocoli barajul, determinați drumul minim. Gheorghe Căiniceanu

VIII.25. Dacă 푥 ≥ 1;푦 ∈ ℝ și 푦√푥 − 1 = 5 să se găsească valoarea minimă a lui 푥 + 4푦 .

Manuela Prajea

VIII.26. Fie x,y numere reale astfel incât : 푥 + 푦 + 10푥 − 14푦+ 64 = 0. Arătați că: |푥 + 푦 − 2| ≤ 2√10 . Victor Săceanu VIII.27. Arătați că ecuația : (푥 − 3푥 + 푘 − 4)(푥 − 3푥 − 푘 − 4) + 푘 = 0 are rădăcini întregi independente de k∈R. Victor Săceanu

VIII.28. Se consideră șirul: 3;6;11;18…. Aflați: a) al 100-lea termen,b)daca 2016 este termen al sirului, c)suma primilor 10 termeni. Victor Săceanu VIII.29. Rezolvați în ℕ ecuația: 푥 + 2푥 + 푥 + 1 = 401 Victor Săceanu

VIII.30. Să se arate că dacă 푎, 푏, 푐 ∈ (0,∞) atunci:

푎 푏푎 + 푏

+푏 푐푏 + 푐

+푐 푎푐 + 푎

≤1

푎 + 푏+

1푏 + 푐

+1

푐 + 푎

Mongolian Olympiad-Daniel Sitaru,Diana Trăilescu

VIII.31. Dacă 푎, 푏, 푐,푑 ∈ ℝ atunci:

푏 + 푐 + 푑 + 푎 + 푐 + 푑 + 푎 + 푏 + 푑 + 푎 + 푏 + 푐 ≥ √3(푎 + 푏 + 푐 + 푑)

Manuela Prajea

VIII.32. Se consideră tetraedrul 퐴퐵퐶퐷 în care 퐴퐴ʹ ⊥ (퐵퐶퐷),퐵퐵ʹ ⊥ (퐴퐶퐷),퐶퐶 ʹ ⊥ (퐴퐵퐷),

퐷퐷ʹ ⊥ (퐴퐵퐶),퐴ʹ ∈ (퐵퐶퐷),퐵ʹ ∈ (퐴퐶퐷),퐶 ʹ ∈ (퐴퐵퐷),퐷ʹ ∈ (퐴퐵퐶).

Dacă 퐴퐴ʹ ∩ 퐵퐵ʹ ∩ 퐶퐶 ʹ ∩퐷퐷ʹ = {퐻} să se arate că: AH + BH + CH + DH ≥ 3min{AAʹ, BBʹ , CCʹ, DDʹ}

Page 63: SOCIETATEA DE ȘTIINȚE - ssmrmh.ro · S.S.M.ROMÂNIA - Filiala Mehedinți 2016 6 REVISTA DE MATEMATICĂ MEHEDINȚEANĂ NR. 17 PRINCIPIUL INCLUDERII ȘI EXCLUDERII Daniel Stretcu

S.S.M.ROMÂNIA - Filiala Mehedinți 2016

63 REVISTA DE MATEMATICĂ MEHEDINȚEANĂ NR. 17

Dan Nedeianu(„CătălinȚigaeru”- Contest-2016)

Clasa a IX-a IX.1. Hexagonul convex 퐴퐵퐶퐷퐸퐹 este înscris în cercul 휔 de centru 푂 și raza 푅. Notăm cu 푆 suma pătratelor lungimilor laturilor hexagonului și cu 퐼 centru cercului înscris în ∆퐵퐷퐹. a. Dacă unghiurile hexagonuui din vârfurile 퐴,퐶 și 퐸 sunt obtuze, demonstrați că 푆 ≥ 2(3푅 + 푂퐼 ). b. Poate avea loc inegalitatea 푆 < 2(3푅 + 푂퐼 )? Justificați răspunsul. Leonard Giugiuc IX.2. Fie ∆퐴퐵퐶 ascuțitunghic cu 퐴퐵 ≠ 퐴퐶 și 퐷,퐸,퐹 punctele de contact ale lui 푤, cercul înscris în triunghi, cu laturile 퐵퐶,퐶퐴 și respectiv 퐴퐵. Perpendiculara în 퐶 pe 퐵퐶 intersectează 퐸퐹 în 푀 și analog perpendiculara în 퐵 pe 퐵퐶 intersectează 퐸퐹 în 푁. Dreapta 퐷푀 intersectează a doua oară 푤 în 푃 și dreapta 퐷푁 intersectează a doua oară 푤 în 푄. Demonstrați că 퐷푃 = 퐷푄. Ruben Dario, Leonard Giugiuc IX.3. Fie numerele reale strict pozitive 푥 , 푥 , … , 푥 unde 푛 ∈ ℕ,푛 ≥ 2. Presupunem 푥 + 푥 +⋯+ 푥 = 푛. Demonstrați inegalitatea:

ퟏ풙ퟏ풏 + 풙ퟐ풏 ퟏ + ⋯+ 풙풏풏 ퟏ +

ퟏ풙ퟏ풏 ퟏ + 풙ퟐ풏 +⋯+ 풙풏풏 ퟏ + ⋯+

ퟏ풙ퟏ풏 ퟏ + 풙ퟐ풏 ퟏ + ⋯+ 풙풏풏

≤ퟏ

풙ퟏ풙ퟐ …풙풏

Leonard Giugiuc IX.4. Să se arate că: √푛 − 1 + √푛 − 2 + √푛 + 1 + √푛 + 2 < 4√푛; (∀)푛 ∈ ℕ; 푛 ≥ 2

Paris Manuela IX.5. Să se arate că dacă 푎, 푏, 푐 ∈ (0,∞)

푎 푐 + 푏 푎 + 푐 푏 +푐푏

+푎푐

+푏푎

≥ 2푎푐푏

+푎푏푐

+푏푐푎

Claudia Nănuți IX.6. Fie 푎, 푏, 푐 ∈ ℝ; 푎 + 푏 + 푐 = 0. Calculați produsul tuturor rădăcinilor ecuațiilor: 푎푥 + 푏푐 + 푐 = 0, 푐푥 + 푎푥 + 푏 = 0 și 푏푥 + 푐푥 + 푎 = 0 Paris Manuela IX.7. Fie 푎, 푏, 푐 ∈ ℝ încât 푎 + 푏 + 푐 = 0. Să se arate că ecuația: 3푎푥 + 2푏푥 + 푐 are toate rădăcinile reale. Emilia Ștefania – Răducan IX.8. Să se arate că dacă 푎, 푏, 푐 ∈ (0,∞) atunci:

∑ ≤ ∑ ∑

∑√ Mihaly Bencze

IX.9. Să se arate că dacă 푎, 푏 ∈ (0, ∞) și 푛 ∈ ℕ∗ atunci: ≥

D.M. Bătinețu – Giurgiu; Neculai Stanciu IX.10. Fie 푛 ∈ ℕ∗\{1} și 푎 ∈ ℝ; 푘 ∈ 1,푛. Să se arate că:

푎 − 푎 푎 + 푎 ≥ 푎 ;푎 = 푎

D. M. Bătinețu – Giurgiu; Neculai Stanciu IX.11. Fie ecuația 36푥 − 96푥 + 63푥 + 2푚푥 −푚 = 0, unde 푚 ∈ ℝ. Să se determine 푚 astfel încât toate rădăcinile ecuației să fie reale. Daniel Stretcu IX.12. Fie 푎, 푏, 푐 numere reale cu proprietățile: 푎 ≤ 푏 ≤ 푐,푎 + 푏 + 푐 = 0 și 푎 + 푏 + 푐 = 38. Să se arate că 푏 ≤ și (푎 − 푐) ≤ 76. Daniel Stretcu

Page 64: SOCIETATEA DE ȘTIINȚE - ssmrmh.ro · S.S.M.ROMÂNIA - Filiala Mehedinți 2016 6 REVISTA DE MATEMATICĂ MEHEDINȚEANĂ NR. 17 PRINCIPIUL INCLUDERII ȘI EXCLUDERII Daniel Stretcu

S.S.M.ROMÂNIA - Filiala Mehedinți 2016

64 REVISTA DE MATEMATICĂ MEHEDINȚEANĂ NR. 17

IX.13. Să se determine funcțiile 푓: [0,∞) → [0,∞) care satisfac simultan condițiile: 1. 푓(7) = 0, 2. 푓(푥) ≠ 0, pentru oricare 푥 ∈ [0,7), 3. 푓(푥 + 푦) = 푓(푦) ⋅ 푓 푥 ⋅ 푓(푦) , pentru oricare 푥,푦 ∈ [0,∞) Daniel Stretcu IX.14. Arătați că 푛√3 nu este întreg, pentru niciun număr natural nenul 푛. Daniel Stretcu IX.15. Scriem numerele naturale nenule, excluzând multiplii de 5, sub forma scării din figura alăturată, adică începând din stânga de jos și completând în sus pe coloane.

Precizați în care coloană și în ce poziție a coloanei se află numărul 2004? Daniel Stretcu IX.16. Determinați soluțiile întregi pozitive ale ecuației 푥 − 푦 = 푥푦 + 2007

Daniel Stretcu

IX.17. Se dă un cerc 퐶 de centru 푂 și rază 푅, având diametrul 퐴퐵. Construim cercurile 퐶 și 퐶 de diametre 퐴푂 și respectiv 퐵푂, iar apoi cercurile 퐶 și 퐶 tangente cercului 퐶 și respectiv cercurilor 퐶 și 퐶 . În final, construim cercurile 퐶 și 퐶 , tangente cercurilor 퐶 ,퐶 și 퐶 , respectiv 퐶 ,퐶 și 퐶 . Să se afle raza cercurilor 퐶 și 퐶 în funcție de 푅. Daniel Stretcu IX.18. Să se determine cea mai mare valoare pentru 푆 = 푥 푥 + 푥 푥 + ⋯+ 푥 푥 , știind că 푥 , 푥 …푥 sunt numere nenegative cu proprietatea: 푥 + 푥 + 푥 + ⋯+ 푥 = 1

Daniel Stretcu IX.19. Să se rezolve: = , unde {푥} notează partea fracționară a numărului real 푥.

Daniel Stretcu IX.20. Să se rezolve ecuația: (5[푥 ] − 3[푥]− 1) ⋅ (5[푥 ]− 2[푥]− 1) = 2[푥] unde [푥] notează partea întreagă a lui 푥. Daniel Stretcu IX.21. Fie mulțimile퐴,퐵ș푖푀.Să se demonstreze implicația 퐴 ∪ (푀\퐵) = 퐵 ∪ (푀\퐴) => 퐴 = 퐵

Doru Preșneanu

IX.22. Se consideră funcția 푓:ℝ → ℝ definită prin relația 푓(푥) = .

Să se calculeze: 푓 + 푓 + 푓 + ⋯+ 푓 . Dana Paponiu

IX.23. Determinați numerele reale 푥,푦, 푧, 푡 știind că acestea îndeplinesc condițiile

푥 ≥ 푦 ≥ 푧 ≥ 푡 ≥ 0,푥 + 푦 + 푧 + 푡 = 1,푥 + 푦 + 푧 + 푡 ≥ 푥 Dan Nedeianu

IX.24. Să se determine minimul funcției 푓: [0,푝] → ℝ, 푓(푥) = − 2푥 cos 푥 − 4 sin 푥.

Gheorghe Căiniceanu

Page 65: SOCIETATEA DE ȘTIINȚE - ssmrmh.ro · S.S.M.ROMÂNIA - Filiala Mehedinți 2016 6 REVISTA DE MATEMATICĂ MEHEDINȚEANĂ NR. 17 PRINCIPIUL INCLUDERII ȘI EXCLUDERII Daniel Stretcu

S.S.M.ROMÂNIA - Filiala Mehedinți 2016

65 REVISTA DE MATEMATICĂ MEHEDINȚEANĂ NR. 17

IX.25. Fie 푓:ℝ → ℝ încât: (푓 ∘ 푓 ∘ … ∘ 푓)„ ”

(푥) = 푎푥 + 푏푥 + 푐; (∀)푥 ∈ ℝ; 푎, 푏, 푐 ∈ ℝ;푎 ≠ 0. Să se

rezolve ecuația 푓(푥) = 푥. Manuela Prajea

IX.26. Să se determine toate funcțiile 푓:ℝ → ℝ care verifică:

푓(푥 ) + 2푥푦+ 푓(푦 ) = 푥푓(푥) + 2푓(푥푦) + 푦푓(푦),∀푥,푦 ∈ ℝ. Radu – Cătălin Șeitan

IX.27. Fie n un număr natural nenul fixat. Rezolvați în ℝ ecuația: + 푛 = 2푛푥, unde [y] reprezintă partea întreagă a numărului real y. Radu – Cătălin Șeitan

IX.28 Fie a,b și c numere reale pozitive cu 푎푏푐 = 8. Găsiți minimul expresiei:

퐴 = + + . Radu – Cătălin Șeitan

IX.29. If x, y, z ∈ ℝ we denote: A = |x + y + z| + |x− z| + |z− y| + |y − x|

B = |x + y − z| + |y + z − x| + |x + z − y|. Prove that: |x| + |y| + |z| ≤ min(A, B)

Daniel Sitaru

IX.30. Prove that if: 푥 ∈ [푎, 푏]; 푖 ∈ 1,푛;푛 ∈ ℕ∗; 푎, 푏 ∈ ℕ∗; 푎 < 푏, then:

푛 +푛 푎푏∑ 푥

≤ (푎 + 푏)1푥

Daniel Sitaru

IX.31. Prove that: 1 − 푐표푠 + 1 − 푐표푠 > 1 − 푐표푠 Daniel Sitaru

IX.32. Prove that if a, b, c, d ∈ (0,∞); a + b + c + d = 1 then: 3∑ ≥ 5∑ Daniel Sitaru

IX.33. Prove that if: 0 < 푥 < 푎 < 푏 + 푐; 0 < 푦 < 푏 < 푎 + 푐; 0 < 푧 < 푐 < 푎 + 푏 then:

푎 + 푏 + 푐 +(푏 − 푐 )푥

푎+

(푐 − 푎 )푦푐

+(푎 − 푏 )푧

푐> 푎푥 + 푏푦 + 푐푧 − (푥 + 푦 + 푧 )

Daniel Sitaru

Clasa a X-a X.1. Fie 푧 un număr complex astfel încât |푧 + 6 − 7푖| ≤ 2√5. Demonstrați că |(푧 + 4 + 2푖)(푧 − 6 − 3푖)| ≥ 50 și studiați cazurile de egalitate.

Leonard Giugiuc , Daniel Sitaru X.2. Pe laturile 퐴퐵,퐵퐶,퐶퐷 și respectiv 퐷퐴 ale patrulaterului convex 퐴퐵퐶퐷 considerăm punctele 푀,푁,푃 și 푄. Știm că 푀푁푃푄 este pătrat și = = = = 푡. a. Dacă 푡 ≠ 1, demonstrați că 퐴퐵퐶퐷 este pătrat. b. Fie 퐴ʹ,퐵ʹ ,퐶 ʹ și respectiv 퐷ʹ punctele de intersecție ale perechilor de drepte (푄푀,퐴퐶), (푀푁,퐵퐷), (푁푃,퐴퐶) și (푃푄,퐵퐷). Demonstrați că dacă 퐴ʹ퐵ʹ퐶 ʹ퐷ʹ este pătrat, atunci 퐴퐵퐶퐷 este pătrat.

Leonard Giugiuc

Page 66: SOCIETATEA DE ȘTIINȚE - ssmrmh.ro · S.S.M.ROMÂNIA - Filiala Mehedinți 2016 6 REVISTA DE MATEMATICĂ MEHEDINȚEANĂ NR. 17 PRINCIPIUL INCLUDERII ȘI EXCLUDERII Daniel Stretcu

S.S.M.ROMÂNIA - Filiala Mehedinți 2016

66 REVISTA DE MATEMATICĂ MEHEDINȚEANĂ NR. 17

X.3. Fie 푧 ∈ ℂ;푎 = −7 + 24푖. Să se calculeze (2 + 푖) . Să se rezolve în mulțimea ℂ ecuația 푧 = 푎. Mihai Octavian Ungureanu X.4. Fie 푧 ∈ ℂ; 푧 = √ √ și 푎 = |푧 |. Să se rezolve ecuația: 푋 − 2푎푋 − 3 = 0

Leonard Giugiuc X.5. Fie 푎, 푏, 푐 ∈ ℂ; |푎| = |푏| = |푐| = 1;푎 + 푏 + 푐 = 0. Să se arate că: 푎 + 푏 + 푐 = 푎푏 + 푏푐 + 푎푐 Mihai Octavian Ungureanu X.6. Fie ∆퐴퐵퐶;퐴(−6);퐵(3);퐶(3푖). Construim în exteriorul acestora triunghiurile echilaterale 퐴퐵퐶 ,퐵퐶퐴′ și 퐶퐴퐵′. Fie 푀,푁,푃 centrele de greutate ale triunghiurilor퐴퐵퐶 ,퐵퐶퐴 respectiv 퐶퐴퐵′. Să se arate că ∆푀푁푃 este echilateral.

Claudia Nănuți X.7. Să se arate că: arcsin(3푥 − 4푥 ) = 3 arccos 푥 ; (∀)푥 ∈ − ,

Claudia Nănuți X.8. Dacă 푥 , 푥 , … , 푥 ∈ (0,∞), 푆 = ∑ 푥 atunci:

푥 (푥 + 푥 )(푆 − 2푥 − 푥 ) ≤푛 ⋅ 푆

3

D. M. Bătinețu – Giurgiu; Neculai Stanciu X.9. Dacă 푚 ∈ [0,∞) și 퐴 ,퐴 , … ,퐴 sunt măsurile, în radiani, ale unghiurilor poligonului convex 퐴 ,퐴 , … ,퐴 iar

푆(푡) = 퐴 ; 푡 ∈ (0,∞)

atunci:

푆(푚 + 1)− 퐴 ≥ (푛 − 1) ⋅ (푛 − 2)휋

D. M. Bătinețu – Giurgiu; Neculai Stanciu

X.10. Dacă 푚 ∈ [0,∞); 푛 ∈ ℕ; 푛 ≥ 2;푥 ∈ (0,∞); 푘 ∈ 1,푛 și 푆 = ∑ 푥 atunci:

푆 ⋅1푥

≥ 푛

D. M. Bătinețu – Giurgiu; Neculai Stanciu

X.11. Fie 푎, 푏, 푐 > 0 cu √푎 + √푏 + √푐 = 3. Să se arate că: 2(푎 + 푏 + 푐) ≥ √푎푏 + √푏푐 + √푐푎 + 3푎푏푐

Emilia Ștefania – Răducan X.12. Să se arate că dacă 푥,푦, 푧 ∈ ℝ atunci: ∑푒 ≤ ∑푒 ≤ ∑푒

Claudia Nănuți

X.13. Fie 푎, 푏, 푐,푑 ∈ (0,∞). Să se rezolve sistemul:

푦푧푡 = 푎푥푧푡푥 = 푏푦푡푥푦 = 푐푧푥푦푧 = 푑푡

. Claudia Nănuți

X.14. Să se arate că dacă 푎, 푏, 푐 ∈ (0,∞) atunci:

∑ ( )( ) ≤∑ ∑ Mihaly Bencze

Page 67: SOCIETATEA DE ȘTIINȚE - ssmrmh.ro · S.S.M.ROMÂNIA - Filiala Mehedinți 2016 6 REVISTA DE MATEMATICĂ MEHEDINȚEANĂ NR. 17 PRINCIPIUL INCLUDERII ȘI EXCLUDERII Daniel Stretcu

S.S.M.ROMÂNIA - Filiala Mehedinți 2016

67 REVISTA DE MATEMATICĂ MEHEDINȚEANĂ NR. 17

X.15. Să se arate că dacă 푎 ∈ [0,∞); 푘 ∈ 1,푛;푛 ∈ ℕ; 푛 ≥ 2 atunci:

∑ ≥ 3푛 Mihaly Bencze X.16. Să se arate că dacă 푎, 푏, 푥,푦, 푧 ∈ (0,∞) atunci:

푦푧(푎 푦 + 푏 푧)푥

+푧푥(푎 푧 + 푏 푥)

푦+푥푦(푎 푥 + 푏 푦)

푧≥

23푎푏(푥 + 푦+ 푧)

D. M. Bătinețu – Giurgiu; Neculai Stanciu

X.17. Fie 푋,푌,푍 puncte oarecare pe dreptele 퐵퐶,퐶퐴 respectiv 퐴퐵 în ∆퐴퐵퐶 de laturi 푎, 푏, 푐 și centru de greutate 퐺. Să se arate că: ⋅ + ⋅ + ⋅ ≥ D. M. Bătinețu – Giurgiu; Neculai Stanciu X.18. Dacă 푎, 푏, 푐 sunt lungimile laturilor unui triunghi 퐴퐵퐶 atunci:

√+ +

√≥ 2 푝 D.M. Bătinețu – Giurgiu; Neculai Stanciu

X.19. Să se arate că în orice ∆퐴퐵퐶 este valabilă relația:

sin 퐴 + sin 퐵 sin 퐶1

sin 퐴+

1sin 퐵

+1

sin 퐶≥

94

D.M. Bătinețu – Giurgiu; Neculai Stanciu X.20. Fie 푡 ∈ [0,∞). Să se arate că în orice ∆퐴퐵퐶 este valabilă relația:

푚푏

+푚푐

+푚푎

≥3 √3푆

4

D.M. Bătinețu – Giurgiu; Neculai Stanciu X.21. Să se arate că în orice ∆퐴퐵퐶 este valabilă relația:

sin퐵 cos+

sin 퐶 cos+

sin퐴 cos≥ 9푆

D.M. Bătinețu – Giurgiu; Neculai Stanciu X.22. Un profesor de matematică scrie pe tablă un polinom 푓(푥) cu coeficienți întregi și zice: „Azi este aniversarea fiului meu Mihai care împlinește 푦 ani”. Apoi adaugă: „Dacă pentru polinomul nostru calculați 푓(0), atunci veți găsi primul număr prim mai mare decât 푦, iar 푓(푦) = 푦.” Câți ani împlinește Mihai? Daniel Stretcu X.23. Dacă 푧 este un număr complex cu Re푧 > , unde 푘 ∈ 푅∗ , arătați că − < .

Daniel Stretcu X.24. Să se rezolve mulțimea numerelor reale:

2004 + 2005 = 40092004 + 2005 = 40092004 + 2005 = 4009

Daniel Stretcu

X.25. Să se rezolve: 2004 + 2004 = 2 Daniel Stretcu X.26. Să se rezolve ecuația: √푥 − 3푥 + 2 = √−2− 푥 . Ovidiu Ticuși

X.27. Determinați 푧 ∈ ℂ, |푧| = 1, cu proprietatea că 8푧 (푧 + 1) = 푧 + 1. Dan Nedeianu

Page 68: SOCIETATEA DE ȘTIINȚE - ssmrmh.ro · S.S.M.ROMÂNIA - Filiala Mehedinți 2016 6 REVISTA DE MATEMATICĂ MEHEDINȚEANĂ NR. 17 PRINCIPIUL INCLUDERII ȘI EXCLUDERII Daniel Stretcu

S.S.M.ROMÂNIA - Filiala Mehedinți 2016

68 REVISTA DE MATEMATICĂ MEHEDINȚEANĂ NR. 17

X.28. Fie a, b și c numere reale pozitivecu 푎 + 푏 + 푐 = 9.Să se determine maximul expresiei 푆 = ln(푎푏푐) + √푎 + √푏 + √푐 −푎 −푏 −푐 . Radu – Cătălin Șeitan

X.29. Solve in complex numbers: 푥 (푥 − 푥 )(푥 − 푥 ) … (푥 − 푥 ) = 푛,

푥 (푥 − 푥 )(푥 − 푥 ) … (푥 − 푥 ) = 푛, − − −, 푥 (푥 − 푥 )(푥 − 푥 ) … (푥 − 푥 ) = 푛

Leonard Giugiuc X.30. Find all integers 푛 ≥ 2 for which there exist the real numbers 푎 , 1 ≤ 푘 ≤ 푛, which are satisfying the following conditions: ∑ 푎 = 0;∑ 푎 = 1and √푛 ⋅ ∑ 푎 = 2 푏√푛 − 1 ,

where 푏 = max {푎 }. (BMO – Shortlist 2016) – Leonard Giugiuc,Daniel Sitaru

X.31. Let 푎, 푏, 푐 and 푏 real numbers such that 푎 + 푏 + 푐 + 푑 = 2 and

푎푏 + 푏푐 + 푐푑 + 푑푎 + 푎푐 + 푏푑 = 0. Find the minimum value and the maximum value of the product 푎푏푐푑. (BMO – Shortlist 2016) – Leonard Giugiuc, Diana Trăilescu

X.32. Fie [퐴퐵퐶퐷]; [퐴 퐵 퐶 퐷 ] două tetraedre și 푀,푁,푃,푄 astfel încât:

퐴푀⃗ = 퐾푀퐴⃗; 퐵푁⃗ = 퐾푁퐵⃗;퐶푃⃗ = 퐾푃퐶⃗;퐷푄⃗ = 퐾푄퐷⃗; 푘 ∈ ℝ∗

Să se arate că tetraedrele [푀푁푃푄], [퐴퐵퐶퐷], [퐴 퐵 퐶 퐷 ] au centrele de greutate coliniare.

Manuela Prajea

X.33. Să se rezolve inecuația: 푥푒 − (푒 − 1) ln(푒 − 1) > 0,∀푥 ∈ (0,∞).

Dan Nedeianu(„CătălinȚigaeru”- Contest-2016) X.34. Find the real numbers k such that: (푎푏푐) + [(1 − 푎)(1− 푏)(1− 푐)] ≤ 1,∀푎, 푏, 푐 ∈ (0,1)

Leonard Giugiuc,Daniel Sitaru(“Sphera”-Contest-2016)

X.35. Fie 푎, 푏, 푐 ∈ (0,∞),푎푏 + 푏푐 + 푐푎 = 3. Să se arate că: + + ≤ 1

Leonard Giugiuc,Daniel Sitaru(“Danubius”-Contest-2016)

Clasa a XI-a XI.1. În planul complex considerăm patrulaterul convex 퐴퐵퐶퐷, cu 퐴(푎),퐵(푏),퐶(푐) și 퐷(푑). Dacă centrul de greutate 퐺 al lui 퐴퐵퐶퐷 nu se află în origine și ∆= 0, unde

∆=

푎 푏 푐 푑푑 푎 푏 푐푐 푑 푎 푏푏 푐 푑 푎

, atunci 퐴퐵퐶퐷 este paralelogram sau 퐴퐵퐶퐷 are diagonalele perpendiculare și de

lungimi egale. Leonard Giugiuc , Diana Trăilescu XI.2. Fie numerele naturale nenule 푚,푛 și 푝, unde 푛 − 1 ≥ 푚 ≥ 푝 + 2. Demonstrați că ecuația 푚 + 푛 = 푝 + (푛 + 1) are exact două soluții reale.

Lucian Georges Lăduncă, Leonard Giugiuc

Page 69: SOCIETATEA DE ȘTIINȚE - ssmrmh.ro · S.S.M.ROMÂNIA - Filiala Mehedinți 2016 6 REVISTA DE MATEMATICĂ MEHEDINȚEANĂ NR. 17 PRINCIPIUL INCLUDERII ȘI EXCLUDERII Daniel Stretcu

S.S.M.ROMÂNIA - Filiala Mehedinți 2016

69 REVISTA DE MATEMATICĂ MEHEDINȚEANĂ NR. 17

XI.3. Fie numerele reale 푎, 푏 și numărul natural cu 푏 > 푎 > 0 și 푛 ≥ 2. Considerăm numerele reale

strict pozitive 푥 , 푥 , … , 푥 astfel încât 푥 + 푥 + ⋯+ 푥 = 푛. Demonstrați inegalitatea: +

+ ⋯+ ≥ Leonard Giugiuc XI.4. Fie 퐴 ∈ 푀 (ℂ); 퐴 ≠ 푂 ; Tr퐴 = 0; det퐴 = 0. Arătați că ecuația: 푋 = 퐴 nu are soluții în 푀 (ℂ). Paris Manuela XI.5. Să se rezolve în 푀 (ℝ) ecuația 푋 = 3 5

3 5 Paris Manuela XI.6. Să se arate că (∀)푥 ∈ ℝ; 푥 ≥ 2 are loc: √푥 − 3 + √푥 − 1 + √푥 + 3 + √푥 + 1 < 4√푥 Leonard Giugiuc XI.7. Fie 푎, 푏, 푐 ∈ (0,∞);푎 + 푏 + 푐 = 1. Să se arate că:

+√

+√

≥ Mihai Octavian Ungureanu

XI.8. Fie 푓: [0,∞) → ℝ; 푓(푥) = arctg(푥 + 2)− arctg푥 + arctg ( ) . Calculați 푓(2016).

Emilia Ștefania Răducan

XI.9. Fie 퐴 ∈ 푀 (ℝ); 퐴 =1 0 00 2 00 0 3

. Fie (퐵 ) ,퐵 = 퐼 + 퐴 ; (∀)푛 ≥ 1.

a. Să se arate că (∀)푛 ≥ 1, (∃)푎 ,푏 , 푐 încât 퐵 =푎 0 00 푎 00 0 푐

b. Să se calculeze: lim→

푎 , lim→

푏 , lim→

Mihai Octavian – Ungureanu XI.10. Fie 푛 ∈ ℕ;푛 ≥ 2;푎, 푏, 푐,푑, 푥 ,푦 ∈ (0,∞); 푘 ∈ 1,푛;

푆 = 푥 ;푆 = 푦

astfel încât: 푎푆 > 푏max (푥 ) ; 푐푆 > 푑max (푦 ) atunci:

푥푎푆 − 푏푥

+푦

푐푆 − 푑푦≥

(푎 + 푐)푛 − (푏 + 푑)푛(푎푛 − 푏)(푐푛 − 푑)

D.M. Bătinețu – Giurgiu; Daniel Sitaru XI.11. Să se arate că dacă 푚 ∈ [1,∞); 푛 ∈ ℕ atunci:

12푛 + 1푘

≥(푛 + 1)

4

D.M. Bătinețu – Giurgiu: Daniel Sitaru XI.12. Fie 푚 ∈ [1,∞) și poligonul convex 퐴 ,퐴 , … ,퐴 ;푛 ≥ 3 astfel încât

푥 = 휇 퐴 퐴 퐴 ; 푝 ∈ 1,푛;퐴 = 퐴 ;퐴 = 퐴 Dacă

푆(푡) = 푥 ; 푡 ∈ (0,∞),

Page 70: SOCIETATEA DE ȘTIINȚE - ssmrmh.ro · S.S.M.ROMÂNIA - Filiala Mehedinți 2016 6 REVISTA DE MATEMATICĂ MEHEDINȚEANĂ NR. 17 PRINCIPIUL INCLUDERII ȘI EXCLUDERII Daniel Stretcu

S.S.M.ROMÂNIA - Filiala Mehedinți 2016

70 REVISTA DE MATEMATICĂ MEHEDINȚEANĂ NR. 17

atunci 푆(푚 + 1)− 퐴

푆(1)− 퐴≥

(푛 − 2) 휋푛

D.M. Bătinețu – Giurgiu; Daniel Sitaru

XI.13. Dacă 퐴,퐵,퐶 ∈ (0,휋); 퐴 + 퐵 + 퐶 = 휋;푚, 푥,푦, 푧 ∈ (0,∞) atunci: 퐴

(푥퐴 + 푦퐵 + 푧퐶) +퐵

(푥퐵 + 푦퐶 + 푧퐴) +푐

(푥퐶 + 푦퐴 + 푧퐵) ≥휋

(푥 + 푦 + 푧) (퐴 + 퐵 + 퐶 )

D.M. Bătinețu – Giurgiu; Daniel Sitaru

XI.14. Dacă 푚 ∈ [0,∞); 푛 ∈ ℕ; 푛 ≥ 3;푎, 푏, 푥 ∈ (0,∞); 푘 ∈ 1,푛 atunci:

푎 + 푏 ⋅푥푥

≥(푎 + 푏) ⋅ 푛

2

D.M. Bătinețu – Giurgiu; Daniel Sitaru

XI.15. Dacă 푚 ∈ [0,∞); 푛 ∈ ℕ; 푛 ≥ 3;푎, 푏, 푥 ∈ (0,∞); 푘 ∈ 1,푛 atunci:

푎 + 푏 ⋅푥푥

≥ 2 ⋅ 푛 ⋅ 푎 ⋅ 푏

unde 푥 = 푥 . D.M. Bătinețu – Giurgiu; Daniel Sitaru

XI.16. Dacă 푡 ∈ [0,∞); 푎, 푏,푚,푛 ∈ (0,∞) atunci:

푎 + 푏 ⋅푚푛

+ 푏 + 푎 ⋅푛푚

≥(푎 + 푏)

2

D.M. Bătinețu – Giurgiu: Daniel Sitaru

XI.17. Să se arate că 푓: (0,3) → ℝ; 푓(푥) = este funcție convexă pe (0,3). Diana Trăilescu

XI.18. Să se arate că dacă 푎, 푏, 푐,푑 ∈ ℝ atunci:

2 푎 푏 + 푏 푐 + 푐 푑 + 푑 푎 ≥ 푎푏 + 푏푐 + 푐푑 + 푑푎 Emilia Ștefania – Răducan

XI.19. Se consideră șirurile: (푥 ) ; (푦 ) ; (푧 ) Dacă

lim→

푒 = lim→

푒 = 퐿

să se calculeze:

lim→

푥 푦

Claudia Nănuți XI.20. Să se arate că dacă 푛 ∈ ℕ;푛 ≥ 3 atunci: ! ≤ 푒 Mihaly Bencze XI.21. Fie 푎, 푏,푚 ∈ [0,∞); 푛 ∈ ℕ∗\{1};푥 ∈ (0,∞); 푘 ∈ 1,푛. Să se arate că:

푥(푎푥 + 푏푥 ) ≥

푛(푎 + 푏) 푥

unde 푥 = 푥 ; 푥 = 푥 . D.M. Bătinețu – Giurgiu; Neculai Stanciu

Page 71: SOCIETATEA DE ȘTIINȚE - ssmrmh.ro · S.S.M.ROMÂNIA - Filiala Mehedinți 2016 6 REVISTA DE MATEMATICĂ MEHEDINȚEANĂ NR. 17 PRINCIPIUL INCLUDERII ȘI EXCLUDERII Daniel Stretcu

S.S.M.ROMÂNIA - Filiala Mehedinți 2016

71 REVISTA DE MATEMATICĂ MEHEDINȚEANĂ NR. 17

XI.22. Fie 푚 ∈ [0, ∞) și 퐴 퐴 ⋯퐴 ;푛 ≥ 3 având laturile de lungimi 푎 = 퐴 퐴 ; 퐴 = 퐴 ;퐴 = 퐴 și măsurile unghiurilor: 훼 = 휇 퐴 퐴 퐴 ;푘 ∈ 1,푛. Să se arate că:

(푛 − 2) 휋푎훼

≥ 푛 푎

D.M. Bătinețu – Giurgiu; Neculai Stanciu XI.23. Fie 훼,훽,푚 ∈ [0,∞); 푛 ∈ ℕ∗\{1};

푥 ∈ (0,∞); 푘 ∈ 1,푛;푆 = 푥 .

Să se arate că: 푥

(훼푥 + 훽푥 ) ≥푆

(훼 + 훽)

D.M. Bătinețu – Giurgiu; Neculai Stanciu XI.24. Fie 푎, 푟 ∈ (0, ∞); (푎 ) ; 푎 = 푎; 푎 = 푎 + 푟,푛 ∈ ℕ∗; 푏 = ∏ 푎 ; 푐 = ∏ 푏 . Să se calculeze:

lim→∞

푛푐

D.M. Bătinețu – Giurgiu; Neculai Stanciu XI.25. Fie 푚,푛 ∈ (0,∞) și 푋,푌,푍 puncte arbitrare situate pe dreptele 퐵퐶 ,퐶퐴,퐴퐵 care reprezintă

dreptele suport ale laturilor ∆퐴퐵퐶 având 퐺 - centrul de greutate. Să se arate că: +

+ ≥ D.M. Bătinețu – Giurgiu; Neculai Stanciu XI.26. Se consideră șirul (푥 ) de numere reale ce satisface relația:

+ 4푛 ≤ 9 + 16푥 ,∀푛 ≥ 1. Să se determine lim →∞ . Să se calculeze:

lim→∞

1푥

+1푥

+⋯+1푥

1푛

Dana Paponiu

XI.27. Se consideră șirul de numere reale definit astfel: x = x − nx , ∀n ∈ ℕ∗ , x = 3 și

x = 4. Să se calculeze: lim → si lim → √x . Dana Paponiu

XI.28. Să se rezolve inecuația cos ( ) ≥ 푥 sin ( ) , 푥 ∈ ℝ. Dan Nedeianu

XI.29. Să se arate că dacă 퐴,퐵 ∈ 푀 (ℝ) atunci: det(퐴 + 퐵)− det(퐴 − 퐵) = 2 det퐵 + Tr(퐴∗ ⋅ 퐵)

Manuela Prajea

XI.30. Fie 퐴 = 0 −11 0 ∈ 푀 (ℝ). Să se arate că există 훼 ∈ ℝ astfel încât: 훼(퐼 + 퐴)(퐼 − 퐴) = 퐼

Să se arate că: ∑ 퐴 = 훼(퐼 − 퐴 + 퐴− 퐴 ); 푛 ∈ ℕ. Gheorghe Căiniceanu

Page 72: SOCIETATEA DE ȘTIINȚE - ssmrmh.ro · S.S.M.ROMÂNIA - Filiala Mehedinți 2016 6 REVISTA DE MATEMATICĂ MEHEDINȚEANĂ NR. 17 PRINCIPIUL INCLUDERII ȘI EXCLUDERII Daniel Stretcu

S.S.M.ROMÂNIA - Filiala Mehedinți 2016

72 REVISTA DE MATEMATICĂ MEHEDINȚEANĂ NR. 17

XI.31. Fie (푎 ) un șir de numere reale cu proprietatea că șirul (푏 ) este mărginit, unde 푏 = 푛 (푎 − sin푛) . Arătați că șirul (푎 ) este divergent.

Gabriela Bondoc

XI.32. Fie 퐴 ∈ 푀 (푅) cu proprietatea 퐴 ⋅ 퐴 =푎 + 1 0 0

1 푎 + 1 0푎 + 1 푎 + 2 1

, unde 푎 ∈ 푅. Calculați det퐴.

Gabriela Bondoc

XI.33 Fie 푃(푥) = 5푥 + 푥 + 푥 + 1 ∈ ℂ[푋] cu rădăcinile: 푥 , 푥 , … , 푥 ∈ ℂ.Să se calculeze:

Ω = (1 + 푥 ) ∙1

1 − 푥

Daniel Sitaru

XI.34. Fie 퐴,퐵,퐶 ∈ 0, . Să se arate că:

sin퐴sin퐵 sin퐶(sin퐴 + sin퐵 + sin 퐶) ≤

(1− sin퐴)(1 − sin퐵)(1 − sin 퐶)(3− sin퐴 − sin퐵 − sin 퐶)

Daniel Sitaru

XI.35. Fie x, y, z ∈ (0,∞). Să se arate că: x + y + z + + + ≥√

.

Daniel Sitaru

XI.36. Să se calculeze:

lim→

√푛 − √푛 − 1 + √푛 − 2 + ⋯+ (−1) √1√푛

Daniel Sitaru

XI.37.Să se arate că dacă în ∆ABC avem: 0 < A < B < C < atunci: + < + < + .

Daniel Sitaru

XI.38. Să se calculeze:

퐿 = lim→∞

sin ( )

cos cos

Daniel Sitaru

XI.39.Să se arate că dacă 푥,푦, 푧 ∈ (0, ∞); 푥 + 푦 + 푧 = 3푒 atunci: 푥 ∙ 푦 ∙ 푧 ≥ 푒 .

Daniel Sitaru

Page 73: SOCIETATEA DE ȘTIINȚE - ssmrmh.ro · S.S.M.ROMÂNIA - Filiala Mehedinți 2016 6 REVISTA DE MATEMATICĂ MEHEDINȚEANĂ NR. 17 PRINCIPIUL INCLUDERII ȘI EXCLUDERII Daniel Stretcu

S.S.M.ROMÂNIA - Filiala Mehedinți 2016

73 REVISTA DE MATEMATICĂ MEHEDINȚEANĂ NR. 17

XI.40. Să se arate că dacă 푥,푦, 푧 ∈ (0, ∞); 푥 + 푦 + 푧 = 3 atunci: arctg 푥 + arctg푦 + arctg 푧 ≤

Daniel Sitaru

XI.41. Să se arate că dacă 푥,푦, 푧 ∈ 푒√푒, ∞ și 푥 + 푦 + 푧 = 3푒 , atunci: 푥 ∙ 푦 ∙ 푧 ≥ 푒

Daniel Sitaru

XI.42. Să se arate că dacă 푥,푦, 푧 ∈ (0,2);푥 + 푦 + 푧 = 1 atunci:

1 + 푦 + 푧 + 1 + 푥 + 푦 + 1 + 푦+ 푧 ≤ √15

Daniel Sitaru

XI.43. Să se calculeze:

퐿 = lim→

1푘 (푘 + 3)

Daniel Sitaru

XI.44. Să se calculeze:

퐿 = lim→∞

12

tg휋

3 ∙ 2

Daniel Sitaru

XI.45. Să se arate că în ∆퐴퐵퐶 ascuțitunghic este valabilă relația:

cos퐴 + cos퐵 + cos 퐶 + ln(퐴 + 1)(퐵 + 1)(퐶 + 1) ≤ 휋 + 3

Daniel Sitaru

XI.46. Aflați funcțiile derivabile 푓:ℝ → ℝ cu proprietatea că ∀푛 ∈ ℕ∗,∀푥 ∈ ℝ

푓 (푥) + 푛 = ( ) ( ) Dan Nedeianu(„CătălinȚigaeru”- Contest-2016)

XI.47. Fie 푓: (0,∞) → (0,∞) o funcție monoton crescătoare, convexă și derivabilă cu proprietatea că 푥푓 (푥) ≤ 푓(푥),∀푥 > 0. Să se arate că ∀푎, 푏, 푐,푑 ∈ (0,∞) avem:

푓+

푓≤

푏 + 푑

Leonard Giugiuc,Daniel Sitaru(“Laurențiu Duican”-Contest-2016)

Clasa a XII-a XII.1. Demonstrați că există 푐 ∈ 0,√3 pentru care are loc relația:

Page 74: SOCIETATEA DE ȘTIINȚE - ssmrmh.ro · S.S.M.ROMÂNIA - Filiala Mehedinți 2016 6 REVISTA DE MATEMATICĂ MEHEDINȚEANĂ NR. 17 PRINCIPIUL INCLUDERII ȘI EXCLUDERII Daniel Stretcu

S.S.M.ROMÂNIA - Filiala Mehedinți 2016

74 REVISTA DE MATEMATICĂ MEHEDINȚEANĂ NR. 17

푐 + 푥 푒√ 푑푥 =푐3

+ 푒√ 푑푥

Leonard Giugiuc XII.2. Considerăm funcția continuă și crescătoare 푓: 0, → [0,∞). Arătați că există 푚 ∈ 0,

pentru care ∫ 푓(푥) ⋅ sin푥 푑푥 = ∫ 푓(푥) ⋅ cos 푥 푑푥 . Leonard Giugiuc

XII.3. Considerăm mulțimea de funcții ℱ = {푓:ℝ → ℝ|푓(푥푦) = 푓(푥)푓(푦)∀푥,푦 ∈ ℝ și 푓 este continuă pe (0,∞)} Să se determine numărul elementelor lui ℱ a căror imagine este mulțime finită. Leonard Giugiuc XII.4. Fie 푛 un număr natural, 푛 ≥ 2. Demonstrați că numărul 푐푡푔 este irațional.

Leonard Giugiuc XII.5. Fie numerele reale 훼 și 푡, cu 훼 > 1 și 푡 ∈ , . Demonstrați că :

푒 ⋅ sin푥 푑푥 < 푒 cos 푥 푑푥.

Leonard Giugiuc XII.6. Să se calculeze: ∫ − 2∫ Diana Trăilescu XII.7. Să se calculeze:

ln 푥 sin(푥 )푑푥 + 2 푥 cos(푥 ) ln 푥 푑푥 + sin(푥 ) 푑푥

Dan Nănuți XII.8. Să se calculeze:

∫ √

√푑푥; 푥 ∈ (0,1) Mihai Octavian Ungureanu

XII.9. Dacă 푎 ∈ 0, ; 푓:ℝ → ℝ este o funcție continuă și impară să se calculeze:

(푥 + 푥 + 80) arccos(sin푓(푥)) 푑푥

D. M. Bătinețu – Giurgiu

XII.10. Fie 푓(푥) = 푎푥 + 푏푥 + 푏푥 + 푎; 푎, 푏 ∈ ℝ,푎 + 푏 = 0.Să se arate că (푥 − 1) divide 푓. Mihai Octavian Ungureanu

XII.11. Să se calculeze: (2푒 + 1) sin 푥 − cos 푥 − 1

푒 + (cos푥 − sin푥 + 2)푒 + cos푥 − sin푥 cos 푥 − sin 푥 + 1푑푥

Mihaly Bencze XII.12. Fie 푓,푔: [푎, 푏] → ℝ funcții continue și

푓(푥) 푑푥 = 푔(푥) 푑푥

Să se arate că:

Page 75: SOCIETATEA DE ȘTIINȚE - ssmrmh.ro · S.S.M.ROMÂNIA - Filiala Mehedinți 2016 6 REVISTA DE MATEMATICĂ MEHEDINȚEANĂ NR. 17 PRINCIPIUL INCLUDERII ȘI EXCLUDERII Daniel Stretcu

S.S.M.ROMÂNIA - Filiala Mehedinți 2016

75 REVISTA DE MATEMATICĂ MEHEDINȚEANĂ NR. 17

푔(푥) 푑푥 = 푔(푐) + 푒 푓(푥)푑푥 − 푓(푐)

Mihaly Bencze XII.13. Să se arate că dacă 푥 ∈ (0,1); 푘 ∈ 1,푛;푛 ∈ ℕ; 푛 ≥ 2 atunci:

(1 + 푥 ) (1− 푥 ) 1 + 푥 ≥ 푒√ ∑ ∑

Mihaly Bencze XII.14. Se consideră șirul (퐼 ) , definit astfel : 퐼 = ∫ 푑푥,푛 ≥ 1. Calculați : lim → 퐼 . Ovidiu Ticuși

XII.15. Să se calculeze: 퐼 = ∫ ∙ 푑푥. Dana Paponiu

XII.16. Demonstrați că există 푡 ∈ (2,6) astfel ca : ∫ 푥 푒 푑푥 = √ Dan Nedeianu

XII.17. Fie 퐹[푥] = 푓 = |푎, 푏, 푐,푑 ∈ ℝ, 푐 + 푑 ≠ 0 și 퐹 = {푓 ∈ 퐹[푥]|푎푑 − 푏푐 ≠ 0}

Să se arate că (퐹,∘) este un grup care conține elemente de orice ordin. Gheorghe Căiniceanu

XII.18. Să se determine funcția 푓:ℝ → ℝ dată de: 푓(푥) = ∫ ln(1 + 푥 tg 푡) 푑푡

Manuela Prajea

XII.19. Fie 푓:ℝ → ℝ o funcție cu primitive mărginite și 훼 ∈ ℝ . Să se arate că există un șir crescător (푥 ) astfel încât: lim → 푛 푓(푥 ) = 0. Manuela Prajea

XII.20. Fie (퐺,⋅) un grup și 퐻 = {푥 ∈ 퐺|(∃)푛 ∈ ℤ∗;푥 = 1}. Să se arate că:

a. ℎ ∈ 퐻 ⇒ 푥ℎ ∈ 퐻, (∀)푥 ∈ 퐺, b. (∃)푥 ∈ 퐺 cu 푥ℎ푥 ∈ 퐻 ⇒ ℎ ∈ 퐻. Manuela Prajea

XII.21. Dacă (퐾, +,⋅) este un corp comutativ cu 16 elemente să se arate că

a. 푥 + 푥 = 0, (∀)푥 ∈ 퐾, b. ecuația 푥 + 푥 − 1 = 0 are exact patru rădăcini distincte în 퐾.

Manuela Prajea

XII.22. Calculați integrala următoare: ∫ 푥 tg 푥 푑푥. Gabriela Bondoc

XII.23. Fie funcția 푓: [−휋,휋] → 푅 prin, 푓(푥) = , 푥 ∈ [−휋,휋] − {0}훼,푥 = 0

. Determinați mulțimea

valorilor parametrului 훼 pentru care 푓 este integrabilă. Gabriela Bondoc

XII.24. Să se demonstreze că: ∫ sin푥 푑푥 − ∫ sin (푥 )푑푥 ≤

Daniel Sitaru

XII.25. Să se calculeze:

Page 76: SOCIETATEA DE ȘTIINȚE - ssmrmh.ro · S.S.M.ROMÂNIA - Filiala Mehedinți 2016 6 REVISTA DE MATEMATICĂ MEHEDINȚEANĂ NR. 17 PRINCIPIUL INCLUDERII ȘI EXCLUDERII Daniel Stretcu

S.S.M.ROMÂNIA - Filiala Mehedinți 2016

76 REVISTA DE MATEMATICĂ MEHEDINȚEANĂ NR. 17

퐿 = lim→

푛12

sin 푥 푑푥 −1푛

sin푖푛

sin푗푛

Daniel Sitaru

XII.26. Să se calculeze:

퐿 = lim→∞

1푛

arctg푖푛

arctg푗푛

Daniel Sitaru

XII.27. Să se calculeze:

퐿 = lim→

∫ 푑푡

∫ 푑푡

Daniel Sitaru

XII.28. Să se arate că:

ln푥 푑푥 ≥ ln푘 ln(푘 + 1) ;푛 ∈ ℕ; 푛 ≥ 3

Daniel Sitaru

XII.29. Să se arate că:

휋8≤ arctg 푥 푑푥 ≤ arctg

12

Daniel Sitaru

XII.30. Fie 푓: [0,∞) → [0,∞);푓(0) = 0 o funcție continuă, strict crescătoare.

Dacă 푎 ∈ [0,∞);푏 ∈ 퐼푚푓; 훼 ∈ ℝ ∖ {1} atunci:

푎 푏 ≤1

1− 훼푓(푥)푑푥 +

1훼

푓 (푦)푑푦

Aplicație:

11 − 푒

푒 푑푥 +1푒

ln푦 푑푦 ≥ 1

Daniel Sitaru

Page 77: SOCIETATEA DE ȘTIINȚE - ssmrmh.ro · S.S.M.ROMÂNIA - Filiala Mehedinți 2016 6 REVISTA DE MATEMATICĂ MEHEDINȚEANĂ NR. 17 PRINCIPIUL INCLUDERII ȘI EXCLUDERII Daniel Stretcu

S.S.M.ROMÂNIA - Filiala Mehedinți 2016

77 REVISTA DE MATEMATICĂ MEHEDINȚEANĂ NR. 17

XII.31. Fie 푡 ∈ , , 푓: 0, → [0,∞) f continuă, descrescătoare.Să se arate că:

푓(푥)푐표푠푥푑푥 ≥ 푓(푥)푠푖푛푥푑푥

Leonard Giugiuc,Diana Trăilescu(“Danubius”-Contest-2016) REZULTATE

OLIMPIADA DE MATEMATICĂ-ETAPA JUDEȚEANĂ-19.03.2016 Nr. Numele și prenumele Școala Clasa Premiul Profesor 1 Draga Tătucu Melissa C.N.”Țițeica” V I Draga Tătucu Mariana 2 Şalău Maria C.N.”Țițeica” V II Draga Tătucu Mariana 3 Mânjiană Bianca Alina C.N.”Țițeica” V III Draga Tătucu Mariana 4 Mîţoaică Elena C.N.”Țițeica” V M Draga Tătucu Mariana 5 Ţăruş Andrei Gabriel C.N.”Țițeica” V M Draga Tătucu Mariana 6 Armanca Alexia Emilia Șc.”Th.Costescu” V M Chican-Lăpădat Stela 7 Butaru Mihai Cătălin C.N.”Țițeica” V M Draga Tătucu Mariana 8 Bengulescu Vlad Alexandru C.N.”Țițeica” V M Vasilcanu Florentina 9 Marinescu Raluca Mihaela C.N.”Țițeica” V M Draga Tătucu Mariana

10 Videscu Eli Bianca C.N.”Țițeica” V M Draga Tătucu Mariana 11 Badea Maria Kallyopi C.N.”Țițeica” V M Draga Tătucu Mariana 12 Bostina David C.N.”St. Odobleja” V M Grecu Adela 13 Iliescu Elena C.N.”Țițeica” V M Draga Tătucu Mariana 14 Mazilu Oana C.N.”Traian” V M Vasile Tomiță 15 Mutu Răzvan Costin Iulian C.N.”Țițeica” V M Draga Tătucu Mariana 16 Chiritoiu Constantin L.T. “ V. Gomoiu” V M Osăin Victoria 17 Cîrciumărița Cristiana Ștefania Șc.”Th.Costescu” V M Chican-Lăpădat Stela 18 Drugă Briana Ștefania Șc.”A.Voinescu” V M Coadă Carmen 19 Giotina Daria C.N.”Traian” V M Vasile Tomiță 20 Panfiloiu Luigi David Șc.Gimnazială 6 V M Boșneagu Dina 21 Ploscaru Claudiu Lucian C.N.”Țițeica” V M Draga Tătucu Mariana 22 Anghel Alexandra Gabriela L.T. “ V. Gomoiu” V M Osăin Victoria 23 Căluţoiu Andreea Denisa C.N.”Țițeica” V M Vasilcanu Florentina 24 Ciulca Ramona C.N.”Traian” V M Vasile Tomiță 25 Luca Sfia Davide C.N.”Țițeica” V M Draga Tătucu Mariana 26 Lupsescu Eduard C.N.”Traian” V M Pupăză Ecaterina 27 Poganu Maria-Viviana C.N.”Traian” V M Vasile Tomiță 28 Sirbu Cretu Antonio Mihai L.T. “ V. Gomoiu” V M Osăin Victoria 29 Busuioc Daniel C.N.”Țițeica” V M Vasilcanu Florentina 30 Busuioc Patricia C.N.”Țițeica” V M Draga Tătucu Mariana 31 Popescu Călin C.N.”Țițeica” V M Draga Tătucu Mariana 32 Stîngă Bogdan Șc.Gimnazială 14 V M Palașcă Vladimira 33 Tufiș Denis C.N.”St. Odobleja” V M Bălu Nicoleta 1 Perpelea Iulia Lic.”Șt.Paulian” VI I Pătruțescu Sorin 2 Puiu Mihaela Maria C.N.”Traian” VI II Prajea Manuela 3 Ardeiu Andra-Teodora C.N.”Traian” VI III Prajea Manuela 4 Puiu Gabriela Mara C.N.”Traian” VI M Prajea Manuela 5 Nuică Mihai-Călin C.N.”Traian” VI M Prajea Manuela 6 Popescu Narcis-Șerban C.N.”Țițeica” VI M Stretcu Daniel 7 Sărăcin Iulia C.N.”Țițeica” VI M Stretcu Daniel 8 Bosoanca Raul Șc.Gimnazială 6 VI M Boșneagu Dina 9 Cioroianu Pavel-Rareș C.N.”Țițeica” VI M Stretcu Daniel

10 Diaconescu Christian Șc.Gimnazială 6 VI M Boșneagu Dina 11 Mitoaica Ana Maria Șc.”A.Voinescu” VI M Coadă Carmen 12 Jianu Elena-Alesandra C.N.”Traian” VI M Prajea Manuela 13 Mergea Erika-Maria C.N.”Traian” VI M Prajea Manuela

Page 78: SOCIETATEA DE ȘTIINȚE - ssmrmh.ro · S.S.M.ROMÂNIA - Filiala Mehedinți 2016 6 REVISTA DE MATEMATICĂ MEHEDINȚEANĂ NR. 17 PRINCIPIUL INCLUDERII ȘI EXCLUDERII Daniel Stretcu

S.S.M.ROMÂNIA - Filiala Mehedinți 2016

78 REVISTA DE MATEMATICĂ MEHEDINȚEANĂ NR. 17

14 Mihăescu Răzvan C.N.”Țițeica” VI M Stretcu Daniel 15 Chircu Erika Lic.”Șt.Paulian” VI M Pătruțescu Sorin 16 Negrea Andrei C.N.”Traian” VI M Prajea Manuela 17 Preda Dana-Amina C.N.”Traian” VI M Prajea Manuela 18 Zăuleț Vlad-Constantin C.N.”Țițeica” VI M Stretcu Daniel 19 Screciu Mara C.N.”Traian” VI M Prajea Manuela 20 Stoican Radu C.N.”Traian” VI M Grecu Vasile 21 Ţepeluş Elena C.N.”Traian” VI M Prajea Manuela 1 Vasile Marian Daniel C.N.”Odobleja” VII I Bondoc Gabriela 2 Şoşea Miruna C.N.”Traian” VII II Prajea Manuela 3 Apetri Anastasia C.N.”Traian” VII III Prajea Manuela 4 Semen Valentin Ion C.N.”Traian” VII M Prajea Manuela 5 Dorobanţu Andrei Christian C.N.”Traian” VII M Prajea Manuela 6 Lunguleasa Eugen Alexandru C.N.”Traian” VII M Prajea Manuela 7 Micşoniu Alexandru C.N.”Traian” VII M Prajea Manuela 8 Popa Alexandra Șc. Petre Sergescu VII M Mălineanu Gabriela 9 Voinea Andreea Alessandra C.N.”Traian” VII M Prajea Manuela

10 Răchitan Maria Alexandra C.N.”Traian” VII M Prajea Manuela 11 Popa Laura Denisa C.N.”Odobleja” VII M Bondoc Gabriela 12 Roşieţeanu Ana Maria C.N.”Traian” VII M Prajea Manuela 1 Crăciunescu Ion Emanuel Șc.”P.Dumitriu” VIII I Vasilcanu Tiberiu 2 Bușe Iasmina C.N.”Traian” VIII II Căiniceanu George 3 Grecu Bogdan C.N.”Traian” VIII III Căiniceanu George 4 Gardner Benjamin C.N.”Traian” VIII M Căiniceanu George 5 Baicu Bogdan Șc. “ M.Viteazul” VIII M Bozdog Ilie 6 Chereşdi Doru C.N.”Țițeica” VIII M Draga Tătucu Mariana 7 Ceausene Patricia C.N.”Traian” VIII M Căiniceanu George 8 Vlad Ariana Dalia Șc. “ M.Viteazul” VIII M Bozdog Ilie 9 Tîrîși Claudiu Șc.Gimnazială 14 VIII M Ionică Constantin

10 Jianu Diana Florentina C.N.”Țițeica” VIII M Draga Tătucu Mariana 11 Munteanu Cătălina Șc.Vînju Mare VIII M Șandru Daniela 12 Pîlșan Bianca Șc.”A.Voinescu” VIII M Coadă Carmen 13 Chirita Medeea C.N.”Traian” VIII M Căiniceanu George 14 Vițian Denis Șc.”A.Voinescu” VIII M Coadă Carmen 1 Șeitan Radu C.N.”Traian” IX I Paponiu Dana 2 Picior Cătălin C.N.”Traian” IX II Paponiu Dana 3 Toma Ioan Denis C.N.”Țițeica” IX III Vasilcanu Florentina 4 Florea Andrei-Bogdan C.N.”Țițeica” IX M Stretcu Daniel 5 Ghitan Bogdan Elvis C.N.”Țițeica” IX M Vasilcanu Florentina 6 Ofiteru Cristian Felix C.N.”Țițeica” IX M Stretcu Daniel 7 Fulga Fabian C.N.”Traian” IX M Paponiu Dana 8 Mihaita Alexandru C.N.”Traian” IX M Paponiu Dana 9 Stefan Bogdan C.N.”Traian” IX M Grecu Vasile 1 Marghescu Bogdan C.N.”Traian” X I Prajea Manuela 2 Bobîrsc Laura C.N.”Traian” X II Prajea Manuela 3 Burdescu Alexandru C.N.”Traian” X III Prajea Manuela 4 Andrei Eliza C.N.”Traian” X M Prajea Manuela 5 Antonescu Denis C.N.”Traian” X M Prajea Manuela 6 Ilioaia Andreea C.N.”Traian” X M Prajea Manuela 7 Iacobescu Floarea C.N.”Țițeica” X M Vasilcanu Florentina 8 Iordăchescu Anca C.N.”Traian” X M Prajea Manuela 9 Dinca Ana Maria C.N.”Țițeica” X M Căpraru Dorel

10 Mazilescu Andru C.N.”Traian” X M Prajea Manuela 11 Moraru Andrei C.N.”Traian” X M Prajea Manuela 12 Papava Patricia Lic.”T.Lalescu” X M Farago Alexandru 1 Lungu Vlad C.N.”Traian” XI I Căiniceanu George 2 Prepelita Diana C.N.”Odobleja” XI II Ticuși Ovidiu 3 Radu David Alexandru C.N.”Traian” XI M Căiniceanu George

Page 79: SOCIETATEA DE ȘTIINȚE - ssmrmh.ro · S.S.M.ROMÂNIA - Filiala Mehedinți 2016 6 REVISTA DE MATEMATICĂ MEHEDINȚEANĂ NR. 17 PRINCIPIUL INCLUDERII ȘI EXCLUDERII Daniel Stretcu

S.S.M.ROMÂNIA - Filiala Mehedinți 2016

79 REVISTA DE MATEMATICĂ MEHEDINȚEANĂ NR. 17

4 Bălă Andreea Marina C.N.”Țițeica” XI M Vasilcanu Florentina 5 Fleancu Ingrid C.N.”Traian” XI M Căiniceanu George 1 Meterez Alexandru C.N.”Țițeica” XII I Draga Tătucu M. 2 Firuți Bogdan C.N.”Traian” XII II Căiniceanu George 3 Antonie Ion Gabriel C.N.”Țițeica” XII III Căpraru Dorel 4 Iovițu Mircea George L.T. “ V. Gomoiu” XII III Mărăscu Pavel 5 Raceanu Monica C.N.”Traian” XII M Căiniceanu George

CONCURSUL NAȚIONAL DE MATEMATICĂ APLICATĂ „ADOLF HAIMOVICI” ETAPA JUDEȚEANĂ-19.03.2016

Nr. Numele și prenumele Școala Clasa Premiul Profesor 1 Saizu Elena C.N.”Costescu” IX M Sitaru Daniel 2 Bădeț Adelina C.N.”Țițeica” IX II Nedeianu Dan 3 Gheață Narcis C.N.”Traian” IX III Paponiu Dana 4 Trotea Daniela C.N.”Țițeica” IX III Nedeianu Dan 5 Mihăescu Gheorghe Daniel C.N.”Odobleja” IX M Preșneanu Doru 6 Tomescu Simina Lic.”T.Lalescu” IX M Gorun Sanda 7 Zăvadă Alexandra Lic.”T.Lalescu” IX M Gorun Sanda 8 Popa Delia C.N.”Odobleja” IX I Grecu Adela 9 Sandoniu Cristina C.N.”Odobleja” IX III Grecu Adela

10 Bărzuica Dara Maria C.N.”Odobleja” IX M Bălu Nicoleta 11 Degetaru Narcis C.N.”T.Costescu” IX II Lădaru Daniela 12 Masgras Aida L.T.”Decebal” X III Nițoiu Angela 13 Rolea Roxana C.N.”T.Costescu” X M Lădaru Daniela 14 Brebu Costinel C.N.”T.Costescu” X M Sitaru Daniel 15 Ciortan Bogdan C.N.”T.Costescu” X M Sitaru Daniel 16 Fluerașu Cosmin C.N.”T.Costescu” X M Sitaru Daniel 17 Giurescu Mălina C.N.”Traian” X II Gimoiu Iuliana 18 Videscu Andrada C.N.”Țițeica” X III Vasilcanu Florentina 19 Neo Ionuț Gabriel C.N.”Țițeica” X M Vasilcanu Florentina 20 Erena Elena C.N.”Țițeica” X M Vasilcanu Florentina 21 Ioniță Mihaela C.N.”Odobleja” X I Bălu Nicoleta 22 Cioplea Daniela C.N.”Odobleja” X II Ticuși Ovidiu 23 Voicu Livia Ionela C.N.”Odobleja” X III Bălu Nicoleta 24 Nastasiu Mona C.N.”Odobleja” X M Ticuși Ovidiu 25 Negrescu Maria Ionela C.N.”Odobleja” X M Bălu Nicoleta 26 Daicu Denisa C.N.”Odobleja” X M Ticuși Ovidiu 27 Mladen Maria C.N.”Odobleja” X M Ticuși Ovidiu 28 Cliseru Georgiana C.N.”T.Costescu” XI I Sitaru Daniel 29 Rostogol Maria Denisa C.N.”Traian” XI I Giugiuc Leonard 30 Seimeanu Andrei C.N.”Odobleja” XI II Ticuși Ovidiu 31 Gomoi Viviana C.N.”Odobleja” XI III Ticuși Ovidiu 32 Dăianu Oana C.N.”Traian” XI M Giugiuc Leonard 33 Costovici Adina C.N.”Odobleja” XI M Ticuși Ovidiu 34 Farcaș Loredana C.N.”Odobleja” XI M Bondoc Gabriela 35 Gabor Denisa C.N.”Traian” XI M Giugiuc Leonard 36 Dron Robert Sebastian C.T.”Dierna” XI I Vuc Ionelia 37 Pătășanu Petre C.T.”Dierna” XI II Vuc Ionelia 38 Prundeanu Adelina C.N.”Odobleja” XI II Preșneanu Doru 39 Roșoga Ana Letiția C.N.”Odobleja” XI III Bondoc Gabriela 40 Șocareva Nicoleta C.N.”Odobleja” XI M Bondoc Gabriela 41 Al Niței Ana Mihaela C.N.”Odobleja” XI M Preșneanu Doru 42 Bîrlogeanu Andreea C.T.”Decebal” XI M Nițoiu Angela 43 Hornoiu Mirabela C.N.”Țițeica” XII I Nedeianu Dan 44 Dumitrașcu Ana Maria Lic.”T.Lalescu” XII III Gorun Sanda 45 Răulescu Andreea C.N.”Odobleja” XII I Ticuși Ovidiu 46 Ferlai Denisa Maria C.N.”Odobleja” XII II Bălu Nicoleta 47 Crașoveanu Cristina C.N.”Odobleja” XII II Ticuși Ovidiu

Page 80: SOCIETATEA DE ȘTIINȚE - ssmrmh.ro · S.S.M.ROMÂNIA - Filiala Mehedinți 2016 6 REVISTA DE MATEMATICĂ MEHEDINȚEANĂ NR. 17 PRINCIPIUL INCLUDERII ȘI EXCLUDERII Daniel Stretcu

S.S.M.ROMÂNIA - Filiala Mehedinți 2016

80 REVISTA DE MATEMATICĂ MEHEDINȚEANĂ NR. 17

48 Midan Andrei Angelo C.N.”Odobleja” XII M Bălu Nicoleta 49 Voicu Iuliana C.N.”Odobleja” XII M Ticuși Ovidiu 50 Ghiță Alina C.N.”Odobleja” XII M Ticuși Ovidiu 51 Pisdirca Sorin Andrei C.N.”Odobleja” XII M Bălu Nicoleta

OLIMPIADA NAȚIONALĂ DE MATEMATICĂ-2016 1 Crăciunescu Ion Emanuel Șc.”P.Dumitriu” VIII PREMIUL I

AUR-SSMR Vasilcanu Tiberiu

2 Șeitan Radu Cătălin C.N.”Traian” IX ARGINT-SSMR Paponiu Dana 3 Bușe Iasmina C.N.”Traian” VIII BRONZ-SSMR Căiniceanu George 4 Lungu Vlad C.N.”Traian” XI BRONZ-SSMR Căiniceanu George

CONCURSUL NAȚIONAL DE MATEMATICĂ APLICATĂ „ADOLF HAIMOVICI” ETAPA NAȚIONALĂ-2016

1 Dron Robert Sebastian C.T.”Dierna” XI M Vuc Ionelia 2 Popa Delia C.N.”Odobleja” IX M Grecu Adela 3 Ioniță Mihaela C.N.”Odobleja” X M Bălu Nicoleta

CONCURSUL DE MATEMATICĂ „VALERIU ALACI”

1 Ghilicică Vlad C.N.”Traian” XII II Căiniceanu George 2 Șeitan Radu Cătălin C.N.”Traian” IX III Paponiu Dana

CONCURSUL DE MATEMATICĂ „DISCIPOLII LUI PITAGORA”

1 Vasile Marian Daniel C.N.”Odobleja” VII I Bondoc Gabriela 2 Mergea Erika Maria C.N.”Traian” VI M Prajea Manuela

CONCURSUL DE MATEMATICĂ „LUMINA-MATH”

1 Grecu Octavian Bogdan C.N.”Traian” VIII II Căiniceanu George 2 Mânjiană Bianca Alina C.N.”Țițeica” V III Draga Tătucu Mariana 3 Nuică Mihai Călin C.N.”Traian” VI III Prajea Manuela 4 Vasile Marian Daniel C.N.”Traian” VII III Prajea Manuela 5 Ciulca Vlădaia Rianna C.N.”Traian” V III Vasile Tomiță 6 Bușe Iasmina C.N.”Traian” VIII III Căiniceanu George 7 Șoșea Miruna C.N.”Traian” VII M Prajea Manuela 8 Semen Valentin Ion C.N.”Traian” VII M Prajea Manuela 9 Păcală Cătălin Gabriel C.N.”Țițeica” V M Draga Tătucu Mariana

10 Cioroianu Pavel Rareș C.N.”Țițeica” VI M Stretcu Daniel 11 Mihăescu Răzvan C.N.”Țițeica” VI M Stretcu Daniel

CONCURSUL DE MATEMATICĂ „LAURENȚIU PANAITOPOL”

1 Bușe Iasmina C.N.”Traian” VIII M Căiniceanu George CONCURSUL DE MATEMATICĂ „DAN BARBILIAN”

1 Bușe Iasmina C.N.”Traian” VIII M Căiniceanu George CONCURSUL DE MATEMATICĂ „VIITORI OLIMPICI”

1 Bușe Iasmina C.N.”Traian” VIII II Căiniceanu George 2 Grecu Bogdan C.N.”Traian” VIII II Căiniceanu George 3 Vasile Marian Daniel C.N.”Traian” VII II Prajea Manuela

CONCURSUL DE MATEMATICĂ „PURPLE COMET”

1 Gardner Benjamin C.N.”Traian” VIII M Căiniceanu George 2 Grecu Bogdan C.N.”Traian” VIII M Căiniceanu George 3 Chiriță Medeea C.N.”Traian” VIII M Căiniceanu George 4 Ceaușene Patricia C.N.”Traian” VIII M Căiniceanu George 5 Guran Sarah C.N.”Traian” VIII M Căiniceanu George 6 Tudor Bogdan C.N.”Traian” VIII M Căiniceanu George 7 Dop Alexandra C.N.”Țițeica” VI M Stretcu Daniel

Page 81: SOCIETATEA DE ȘTIINȚE - ssmrmh.ro · S.S.M.ROMÂNIA - Filiala Mehedinți 2016 6 REVISTA DE MATEMATICĂ MEHEDINȚEANĂ NR. 17 PRINCIPIUL INCLUDERII ȘI EXCLUDERII Daniel Stretcu

S.S.M.ROMÂNIA - Filiala Mehedinți 2016

81 REVISTA DE MATEMATICĂ MEHEDINȚEANĂ NR. 17

8 Lazăr Anne-Marie C.N.”Țițeica” VI M Stretcu Daniel 9 Mănescu David C.N.”Țițeica” VI M Stretcu Daniel

10 Chisăliță Andreea C.N.”Țițeica” VI M Stretcu Daniel 11 Mangu Georgiana C.N.”Țițeica” VI M Stretcu Daniel 12 Săndulescu Alexandra C.N.”Țițeica” VI M Stretcu Daniel

CONCURSUL DE MATEMATICĂ „COMPER-2015-2016”

1 Mânjiană Bianca Alina C.N.”Țițeica” V II,III Draga Tătucu Mariana 2 Videscu Eli Bianca C.N.”Țițeica” V II,M Draga Tătucu Mariana 3 Băcan Claudia Alexia C.N.”Țițeica” V III Draga Tătucu Mariana 4 Boștină Arminovici C.N.”Țițeica” V III Draga Tătucu Mariana 5 Șerban Andrei Octavian C.N.”Țițeica” V III,III Draga Tătucu Mariana 6 Păcală Cătălin Gabriel C.N.”Țițeica” V III Draga Tătucu Mariana 7 Erena Sabina C.N.”Țițeica” V III,III Draga Tătucu Mariana 8 Gîrleanu Alexandra C.N.”Țițeica” V III,M Draga Tătucu Mariana 9 Iliescu Elena C.N.”Țițeica” V III,M Draga Tătucu Mariana

10 Lachstadter David C.N.”Țițeica” V III,III Draga Tătucu Mariana 11 Marinescu Raluca Mihaela C.N.”Țițeica” V III Draga Tătucu Mariana 12 Paralescu Alexandra C.N.”Țițeica” V III Draga Tătucu Mariana 13 Șalău Maria C.N.”Țițeica” V III Draga Tătucu Mariana 14 Butaru Mihai Cătălin C.N.”Țițeica” V III Draga Tătucu Mariana 15 Busuioc Patricia C.N.”Țițeica” V III,M Draga Tătucu Mariana 16 Draga Tătucu Mellisa C.N.”Țițeica” V III,III Draga Tătucu Mariana 17 Duicu Ștefania Elena C.N.”Țițeica” V III,M Draga Tătucu Mariana 18 Iacob Ruxandra Maria C.N.”Țițeica” V III Draga Tătucu Mariana 19 Popescu Călin C.N.”Țițeica” V III Draga Tătucu Mariana 20 Cîrstea Alexia Mădălina C.N.”Țițeica” V M Draga Tătucu Mariana 21 Mutu Răzvan Costin C.N.”Țițeica” V M,M Draga Tătucu Mariana 22 Chiriță Mihai C.N.”Țițeica” V M Draga Tătucu Mariana 23 Luca Sfia Davide C.N.”Țițeica” V M Draga Tătucu Mariana 24 Mițoaica Elena C.N.”Țițeica” V M Draga Tătucu Mariana 25 Dragotoniu Ionuț C.N.”Țițeica” V M Draga Tătucu Mariana 26 Chereșdi Doru C.N.”Țițeica” VIII III Draga Tătucu Mariana 27 Jianu Diana C.N.”Țițeica” VIII III,M Draga Tătucu Mariana 28 Lazăr Alexandru C.N.”Țițeica” VIII M,M Draga Tătucu Mariana 29 Pîndici Andreea Denisa C.N.”Țițeica” VIII M,M Draga Tătucu Mariana 30 Pîrvu Denisa C.N.”Țițeica” VIII M,M Draga Tătucu Mariana 31 Mihăescu Răzvan C.N.”Țițeica” VI III,II Stretcu Daniel 32 Guină Alexandru C.N.”Țițeica” VI III Stretcu Daniel 33 Popescu Șerban C.N.”Țițeica” VI III,II Stretcu Daniel 34 Calu Andrei Daniel C.N.”Țițeica” VI M,III Stretcu Daniel 35 Giurcanu Costinel C.N.”Țițeica” VI M,M Stretcu Daniel 36 Albei Ema Raisa C.N.”Țițeica” VI M Stretcu Daniel 37 Cioroianu Pavel Rareș C.N.”Țițeica” VI M Stretcu Daniel 38 Leuștean Diana C.N.”Țițeica” VI M,M Stretcu Daniel 39 Bereczki Iulia C.N.”Țițeica” V M Draga Tătucu Mariana 40 Sărăcin Răzvan C.N.”Țițeica” VIII III Draga Tătucu Mariana 41 Brehui Ana Maria C.N.”Țițeica” VIII III Draga Tătucu Mariana 42 Costache Patricia C.N.”Țițeica” VIII M Draga Tătucu Mariana 43 Ciochia Andrei Toni C.N.”Țițeica” VIII M Draga Tătucu Mariana 44 Bora Ana Maria C.N.”Țițeica” VIII M Draga Tătucu Mariana 45 Combei David C.N.”Țițeica” VIII M Draga Tătucu Mariana 46 Jenaru Constantin C.N.”Țițeica” VIII M Draga Tătucu Mariana 47 Ispir Răzvan C.N.”Țițeica” VIII M Draga Tătucu Mariana 48 Bărbuț Denis C.N.”Țițeica” VIII M Draga Tătucu Mariana 49 Măgureanu Raoul Gabriel C.N.”Țițeica” VIII M Ungureanu Mihai Octavian 50 Lupițu Adrian C.N.”Țițeica” VI III Stretcu Daniel 51 Firu Elena Claudia C.N.”Țițeica” VI III Stretcu Daniel

Page 82: SOCIETATEA DE ȘTIINȚE - ssmrmh.ro · S.S.M.ROMÂNIA - Filiala Mehedinți 2016 6 REVISTA DE MATEMATICĂ MEHEDINȚEANĂ NR. 17 PRINCIPIUL INCLUDERII ȘI EXCLUDERII Daniel Stretcu

S.S.M.ROMÂNIA - Filiala Mehedinți 2016

82 REVISTA DE MATEMATICĂ MEHEDINȚEANĂ NR. 17

52 Bîrzan Dalia Andrada C.N.”Țițeica” VI M Stretcu Daniel 53 Săndulescu Alexandra C.N.”Țițeica” VI M Stretcu Daniel 54 Bălășoiu Robert C.N.”Țițeica” VI M Stretcu Daniel 55 Bibu Ștefan C.N.”Țițeica” VI M Stretcu Daniel 56 Bătineanu Robert C.N.”Țițeica” VI M Stretcu Daniel 57 Giurcanu Gabriel C.N.”Țițeica” VI M Stretcu Daniel 58 Roșoga Eduard C.N.”Țițeica” VI M Stretcu Daniel 59 Tomescu Adriana C.N.”Țițeica” VI M Stretcu Daniel 60 Leuștean Roxana C.N.”Țițeica” VI M Stretcu Daniel

OIM „FORMULA OF UNITY/THE THIRD MILLENIUM”

1 Marghescu Bogdan C.N.”Traian” X III Prajea Manuela CONCURSUL DE MATEMATICĂ „MINIMATH”

1 Paralescu Alexandra C.N.”Țițeica” V I Draga Tătucu Mariana 2 Iliescu Elena C.N.”Țițeica” V I Draga Tătucu Mariana 3 Draga Tătucu Melissa C.N.”Țițeica” V I Draga Tătucu Mariana 4 Șalău Maria C.N.”Țițeica” V I Draga Tătucu Mariana 5 Marinescu Raluca C.N.”Țițeica” V II Draga Tătucu Mariana 6 Băcan Claudia C.N.”Țițeica” V III Draga Tătucu Mariana 7 Duicu Ștefania C.N.”Țițeica” V M Draga Tătucu Mariana 8 Chiriță Mihai C.N.”Țițeica” V M Draga Tătucu Mariana 9 Badea Maria C.N.”Țițeica” V M Draga Tătucu Mariana

10 Erena Sabina C.N.”Țițeica” V M Draga Tătucu Mariana 11 Popescu Narcis C.N.”Țițeica” VI I Stretcu Daniel 12 Bibu Ștefan C.N.”Țițeica” VI II Stretcu Daniel 13 Zăuleț Vlad C.N.”Țițeica” VI III Stretcu Daniel 14 Roșoga Eduard C.N.”Țițeica” VI M Stretcu Daniel 15 Chisăliță Andreea C.N.”Țițeica” VIII I Stretcu Daniel

CONCURSUL REZOLVITORILOR GMB 04/2015-03/2016

1 Mihăescu Răzvan Andrei C.N.”Țițeica” VI 1540

PROFESORI CU MINIM 5

APARIȚII DE NUME ÎN GMB:

04/2015-03/2016

Prajea Manuela Stretcu Daniel

Căiniceanu Gheorghe Draga Tătucu Mariana

Coadă Carmen Paponiu Dana Dan Nedeianu

Leonard Giugiuc Ștefan Marica Daniel Sitaru

2 Vasile Marian Daniel C.N.”Odobleja” VII 1520 3 Săndulescu Alexandra C.N.”Țițeica” VI 1400 4 Pătuleanu Ioana Maria C.N.”Țițeica” IX 1140 5 Bălășoiu Robert Alin C.N.”Țițeica” VI 1000 6 Ganea Andreea Denisa C.N.”Țițeica” VI 930 7 Dan Ela Șc.Gen.nr.6 V 920 8 Andrița Miruna C.N.”Traian” VIII 640 9 Draga Tătucu Melisa C.N.”Țițeica” V 630

10 Mînjină Bianca Eliza C.N.”Țițeica” V 620 11 Nuică Mihai Călin C.N.”Traian” VI 600 12 Ghelner Alexandra C.N.”Țițeica” VI 500 13 Pârvuceanu Alexandru Șc.”A.Voinescu” VII 490 14 Alupoaie Radu C.N.”Traian” VIII 480 15 Roșoga Eduard C.N.”Țițeica” VI 450 16 Tomescu Adrian C.N.”Țițeica” VI 440 17 Popescu Narcisa C.N.”Țițeica” VI 440 18 Fleancu Robert C.N.”Traian” VIII 400 19 Ceaușene Patricia C.N.”Traian” VIII 390 20 Cojocaru Armand C.N.”Traian” VIII 380 21 Calinciuc Roberta C.N.”Țițeica” VI 350 22 Zăuleț Vlad C.N.”Odobleja” VI 330 23 Crețescu Maria Alexandra C.N.”Traian” VIII 320

Clasamentul complet al rezolvitorilor se găsește pe site-ul SSMRMH:www.ssmrmh.ro

Page 83: SOCIETATEA DE ȘTIINȚE - ssmrmh.ro · S.S.M.ROMÂNIA - Filiala Mehedinți 2016 6 REVISTA DE MATEMATICĂ MEHEDINȚEANĂ NR. 17 PRINCIPIUL INCLUDERII ȘI EXCLUDERII Daniel Stretcu

S.S.M.ROMÂNIA - Filiala Mehedinți 2016

83 REVISTA DE MATEMATICĂ MEHEDINȚEANĂ NR. 17

AUTORII MATERIALELOR DIN R.M.M.-17

Nr.crt. Numele și prenumele Nr.crt. Numele și prenumele 1 DANIEL SITARU 30 VAN LE 2 GHEORGHE CĂINICEANU 31 AUGUSTINI MORARU 3 CLAUDIA NĂNUȚI 32 IMAD ZAK 4 DIANA TRĂILESCU 33 RAHIM SHABAZOV 5 MIHAI OCTAVIAN UNGUREANU 34 ROBET KOSOVA 6 DAN NĂNUȚI 35 KUNIHIKO CHIKAIA 7 LEONARD GIUGIUC 36 SK REJUAN 8 EMILIA RĂDUCAN 37 GREGOIRE NICOLLIER 9 OVIDIU TICUȘI 38 MICHALOS NIKOLAU

10 MANUELA PRAJEA 39 GEORGE APOSTOLOPOULOS 11 ELENA RÎMNICEANU 40 ARKADY ALT 12 DAN NEDEIANU 41 ANGEL PLAZA 13 DANIEL STRETCU 42 MIQUEL OCHOA SANCHEZ 14 MIHALY BENCZE 43 RUBEN DARIO 15 D.M.BĂTINEȚU-GIURGIU 44 ALEXANDER BOGOMOLNY 16 NECULAI STANCIU 45 NGUYEN VIET HUNG 17 GHEORGHE CALAFETEANU 46 OLEG FAYNSHTEYN 18 DORU PREȘNEANU 47 KUNIHIKO CHIKAYA 19 IULIANA TRAȘCĂ 48 ABNUSHAV MISHRA 20 DANA PAPONIU 49 PHAM HUANG NGUYEN 21 ION NĂNUȚI 50 DAO THANH OAI 22 LĂDARU DANIELA 51 FRANCISCO JAVIER GARCIA 23 CARMEN-VICTORIȚA CHIRFOT 52 RADU CĂTĂLIN ȘEITAN 24 MARIA UNGUREANU 53 VICTOR SĂCEANU 25 MARIANA CORNEA 54 TITU ZVONARU 26 GHEORGHIȚA TUFIȘ 55 MANUELA PARIS 27 VALERIU DRULĂ 56 LUCIAN GEORGES LĂDUNCĂ 28 GABRIELA BONDOC 57 MARIAN DINCĂ 29 DOREL CĂPRARU 58 DANIEL LIUMINATI

NOTĂ: Website-ul SSMR-Filiala Mehedinți are adresa http//:www.ssmrmh.ro Pe acest site găsiți conținutul numerelor anterioare ale RMM, subiectele de la OLM, OJM,ONM,baraje,OBM,OIM, Concursul Național de Matematică Aplicată”Adolf Haimovici”, baza de date cu probleme rezolvate, note și articole matematice, rezolvările problemelor din RMM precedente, clasamentul rezolvitorilor GMB și galeria de onoare cuprinzând elevii medaliați la olimpiadele naționale și internaționale pe care suntem mândri că i-am cunoscut. Pentru a publica probleme propuse, articole și note matematice în RMM sau pe site puteți trimite materialele pe mailul: [email protected]